You are on page 1of 223

PEDIATRICS

EXAMINATION REVIEW

Questions , Answers And Teaching nots for each question


Dr. kamel hassan
C.E.S DE PEDIATRIE, C.P.B. OF PEDIATRICS Medical Services Directorate

PEDIATRICS EXAMINATION REVIEW

This book acknowledges those who anti cipate the need to use this book and those who made it possible. It is dedicated to the Palestine medical council and all my colleagues.

PEDIATRICS EXAMINATION REVIEW

INTRODUCTION
This is a test book, not a text book. This test book will be a learning experience because the questions and answers are structured to give you experience in preparation for any examination. In addition ,the questions and answers are structured to teach a great deal about problems, disease, differenti al diagnosis, process, and therapy. It will test your medical skills and approach to diagnosis, laboratory confirmation, therapeutic planning, and anticipated outcome. Pediatrics has been substantially revised and updated to keep in step with current trends in medical education and the continuing expansion of scientific knowledge. The questions themselves are organized in broad categories to give you a representative sampling of the material covered in course work and are modeled as closely as possible after those used by the board. The answers and teaching notes follow each questions. Using this test book, you may identify the areas of strength and weakness in your command of the subject.

PEDIATRICS EXAMINATION REVIEW

Questions
(PART 1)

TRUE OR FALSE

PEDIATRICS EXAMINATION REVIEW

1- Regarding hepatitis B virus (HBV):


1. HBV is an RNA virus 2. HBV only replicates in the liver 3. HBsAg is the first serological marker to appear 4. The presence of HbsAg indicates acute infection only 5. The persistence of HbeAg indicates infectivity

2- The following statements are true regarding superantigens:


1. They are very strongly antigenic bacterial polysaccharides 2. They bind to B cell receptors 3. Lead to excess stimulation of T cell proliferation and interleukin-2 production 4. They are responsible for gram-negative toxic shock 5. Lead to stimulation of production of TNFa and other cytokines

3- Causes of pulmonary fibrosis include: 1. Sulfasalazine 2. Chlorpromazine 3. Penicillamine 4. Methysergide 5. Nitrofurantoin

4- Cerebral blood flow:


1. Is increased in an acutely compensated metabolic acidosis. 2. Is decreased at 72 hours in a normal person who is made to hyperventilate for 72 hours. 3. Shows a right shift in the autoregulation curve in patients with chronic hypertension. 4. Shows a linear increase with PaO2. 5. Is unaffected by temperature.

5- The following are clinical associations of Fallots tetralogy:


1. Single second heart sound. 2. Cerebral abscess. 3. Digital clubbing. 4. Right-sided aortic arch. 5. Aortic ejection click.

6- Muscle relaxation during surgery:


1. Allows improved intra-abdominal access. 2. Is not necessary in ventilated patients. 3. Is maintained using a depolarising agent such as suxamethonium. 4. Block acetyl-choline receptors on muscle end plates. 5. Side effects include histamine release.

PEDIATRICS EXAMINATION REVIEW

7-In hypogammaglobulinaemia, whi ch of the followings are true?


1. In nephrotic syndrome, the loss of immunoglobulin in urine is the only cause of predisposition to infection 2. In common variable immunodeficiency, there is an increased incidence of lymphoma 3. Selective IgA deficiency is the commonest inherited form 4. In patients over 40 years of age, a chest radiograph is mandatory to exclude thymoma 5. The genetic defect in X-linked agammaglobulinaemia (XLA) is in Brutons tyrosine kinase (Btk)

8- ACE inhibitors:
1. They cause hypotension via a drop in systemic vascular resistance 2. May lead to a side effect of dry cough, due to an increase in bradykinin levels 3. In the normal kidney, will cause an increase in renal blood flow 4. They are highly protein bound drugs 5. Indications include congestive cardiac failure in a patient with renal artery stenosis

9-Macrocephaly :1. Is defined as Head circumference > 1 SD above mean 2. Is associated with high intelligence 3. Is associated with Cornelia de Lange syndrome 4. Can be inherited 5. Is associated with neurofibromatosis

10-Regarding normal renal physiology:


1. The GFR of a child reaches adult values (corrected for weight) by the age of one year 2. The fluid entering the DCT is iso-osmolar 3. Under the influence of ADH, there is increased permeability to water in the DCT 4. In the proximal tubules, there is near -complete absorption of glucose and amino acids 5. There are approximately one million nephrons in each kidney

11-The following conditions occur due to the presence of circulating antibodies:


1. Graves disease. 2. Hashimoto thyroiditis. 3. Reidel's thyroiditis. 4. Myasthenia gravis. 5. Motor neurone disease.

12-The following can cause card iomyopathy in HIV disease: 1. Mycobacterium tuberculosis 2. Lymphoma 3. Kaposi Sarcoma 4. CMV infection 5. HIV of the heart

PEDIATRICS EXAMINATION REVIEW

13- Gastric emptying is slowed by:


1. Supine posture 2. Gastric distention 3. Domperidone 4. Ranitidine 5. Cisapride

14- The following is true regarding changes in the fetal cardiovascular system after birth:
1. There is normally immediate closure of the ductus arteriosus 2. Hypoxia-induced vasoconstriction is the mechanism of closure of the ductus arteriosus 3. The ligamentum teres is the remnant of the umbilical vein 4. Regression of right ventricular hypertrophy occurs postnatally 5. Inferior vena caval pressure falls after birth

15- X-linked hyper IgM syndrome :1. Causes defective B cell development 2. Is associated with normal levels of IgG 3. Can present with pneumocystis carinii infection 4. Is due to a defective CD40 ligand 5. Is due to a failure of Ig isotype switching

16- Control of absorption: Which of the following statements are true?


1. Baroreceptors are the principal sensors for controlling sodium secretion and reabsorption. 2. In reduced sodium states, both increased activity of renal sympathetic nerves (leading to constricting afferent arterioles) and reduced glomerular capillary pressure, decrease Na+ and water excretion. 3. Changes in aldosterone levels allow control of the re -uptake of 50% of sodium originally present in the glomerular filtrate. 4. Lungs secrete angiotensinogen, kidneys secrete renin, liver cells contain angiotensin -converting enzyme. 5. Renin secretion is controlled by the macula densa.

17- A cataract may be induced accidentally by the following treatments:


1. Systemic steroids 2. Prolonged general anaesthesia 3. Massive blood transfusion 4. Total body irradiation 5. Multiple MRI scans

18- Regarding surfactant:1. Production begins at 30 weeks 2. It is produced by Type II pneumocytes in the walls of the bronchi 3. Testosterone stimulates surfactant production 4. Production is increased during a stressful event like hypothermia 5. Betamathasone given to the mother improves surfactant production in the premature baby

PEDIATRICS EXAMINATION REVIEW

19- The following syndromes commonly have associated renal manifestations: 1. Zellwegers syndrome 2. Drash syndrome 3. Meckel Gruber syndrome 4. Noonan's syndrome 5. Chromosome 11p deletion

20- Bile:
1. Is secreted by the bile canaliculi 2. Contains NaHCO3 3. Is released from the gall bladder in response to the hormone gastrin 4. Contains salts that are only water soluble 5. Salts are mostly reabsorbed in the large bowel

21- The following are true regarding vaccine administration in children: 1. Live vaccines are contraindicted within 3 weeks of another live vaccine administration 2. MMR can be given a week after administration of IV immunoglobulins 3. Live vaccines are not given to children on prolonged steroid therapy 4. Severe febrile illness is a contraindication for DPT 5. Severe malnutrition is a contraindication for BCG

22- Wide splitting of second heart sound may be evident in:


1. Normal children 2. Volume overload due to partial anomalous venous drainage (PAPVD) 3. Severe aortic defect 4. Atrial septal defect 5. Pulmonary hypertension

23- Regarding hyperkalaemia : 1. Tall P waves are seen on ECG 2. Continuous salbutamol nebulisers should be started immediately on diagnosis 3. IV calcium resonium should be commenced 4. Calcium gluconate exchanges for potassium and enables its excretion 5. If acidosis is corrected, potassium shifts back into the intracellular fluid

24- T lymphocytes:1. Are derived from a precursor cell in the bone marrow 2. 95% die in the thymus 3. All have the alpha-beta heterodimeric T cell receptor 4. Are usually activated by intact protein antigens 5. Are prominent in type II hypersensitivity reactions

PEDIATRICS EXAMINATION REVIEW

25- Post-traumatic stress disorder : 1. Is an immediate response to a traumatic event 2. Often involves flashbacks 3. May lead to avoidance 4. Can sometimes be treated by play therapy 5. Is associated with feelings of detachment 26- Refsums Disease: 1. Is inherited as an autosomal dominant disorder 2. Is associated with icthyosis 3. Is caused by the same enzyme as Hurlers disease 4. Is one of the causes of retinitis pigmentosa 5. Is affected adversely by radical weight change 27- The following are true of adult polycystic kidney disease: 1. It can present in the newborn. 2. It may be associated with aortic stenosis. 3. The genetic mutation of type I has been linked to chromosome 16. 4. Liver cysts are seen in approximately 10%. 5. The incidence is about 1:10,000. 28-The following statements are true regarding Hypertrophic obstructive cardiomyopathy:1. The prognosis is related to the degree of left ventricular outflow gradient 2. Prognosis is worse in children 3. Runs of Non sustained ventricular tachycardia on Holter have a high predictive value for sudden cardiac death 4. Progresses to dilated cardiomyopathy in 10% cases 5. Strenuous activity is proscribed in all patients 29- Disseminated intravascular coagulation: 1. Is usually associated with a high fibrinogen level 2. Is usually associated with a normal thrombin time 3. May be associated with cytomegalovirus or varicella zoster infections 4. May occur in liver disease 5. May be associated with formation of microthrombi, so should be treated with intravenous heparin 30-Cardiac tamponade: 1. Occurs only after penetrating injuries to the heart. 2. Is characterised by arterial hypertension. 3. The central venous pressure is raised. 4. Should be treated with aspiration using a needle inserted via the second intercostal space, mid-clavicular line. 5. Is characterised by increase ECG voltage across all leads.

PEDIATRICS EXAMINATION REVIEW

31-Regarding glycogen and its metaboli sm: 1. Glycogenolysis in the liver releases glucose-6-phosphate into the blood stream 2. Branches in the glycogen molecule are produced by 1,4 -glycosidic linkages 3. Adrenaline is an important stimulus for glycogenolysis in the liver during exercis e 4. Deficiency of muscle phosphorylase, may result in an impaired exercise tolerance 5. McArdles Disease results in fasting hypoglycaemia and hepatomegaly 32-Concerning Hepatitis C Virus: 1. 1% of the worlds population is infected 2. The proportion developing chronic infection and hepatocellular carcinoma are similar to HBV infection 3. Neonatal infection is more likely if the mother has concomitant HIV infection as well as HCV 4. Breast feeding should be discouraged in the HCV +ve mother 5. Liver transplantation is usually required due to the inadequacy of medical therapies 33-Which of the following conditions would be suitable for somatic gene therapy: 1. Adenosine deaminase deficiency 2. Thalassaemia 3. Diabetes mellitus 4. Haemoplilia A 5. Sickle cell disease 34-In paroxysmal nocturnal haemoglobinuria: 1. The defect is a congenital abnormality of RBC production by the bone marrow 2. Hamoglobinuria is a characteristic finding 3. Venous thrombosis is a recognised complication 4. The diagnostic test is Hams test 5. Transforms into acute leukaemia in approx. 5% 35-Phosphodiesterase inhibitors: 1. Have an anti-diuretic effect 2. Cause bronchodilation 3. Cause peripheral dilatation 4. Cause relaxation of the diaphragm 5. Decreases the pulmonary capillary wedge pressure 36-Regarding mumps infection: 1. The incubation period is 7-14 days 2. It is often asymptomatic 3. Children are not infectious after the development of parotitis 4. Infertility is a frequent complication of the orchitis associated with mumps 5. It is caused by a DNA virus

10

PEDIATRICS EXAMINATION REVIEW

37-The following are known functions of T helper cells : 1. Production of T cell growth factors such as IL-2 2. Promotion of immunoglobulin isotype class switching 3. Production of perforin 4. Production of suppressive cytokines such as IL -10, TGF-beta 5. Induction of the maturation of immature dendritic cells 38-Regarding immunisation in the UK: 1. The first dose of vaccines is given at a corrected age of 2 months in case of premature delivery 2. Family history of febrile convulsion following immunisation is a relative contraindication

3. Measles, mumps, rubella and polio are contraindicated in HIV +ve children 4. Live vaccines should be avoided within 6 months of termination of lymphoma treatment 5. Prolonged inconsolable crying to previous immunisation is a definite contraindication. 39-Regarding paediatric liver transplantation:1. The most common indication is failed Kasai procedure 2. 5-yr survival rates for children after orthotopic liver transplantation are around 30% 3. FK506 has superceded tacrolimus as an immunosuppressant 4. Signs of chronic liver disease persist despite improvement in liver function 5. Organ rejection is an early post operative complication
40-The following statements are correct regarding Horners syndrome:

1. It may be caused by injury to the superior cervical sympathetic ganglion. 2. Ptosis occurs because of loss of innervation to the smooth muscle portion of the levator palpebrae superioris. 3. Pupillary dilatation occurs because of loss of innervation to the iris sphincter muscle. 4. May be associated with ophthalmoplegia in the territory of the IIIrd cranial nerve. 5. May be associated with loss of sweating on the ipsilateral arm. 41-In HIE or Jobs syndrome : 1. There is a markedly raised IgE level 2. There is an association with coarse facies, kyphoscoliosis, osteoporosis, and eczema 3. There is a low incidence of atopy due to defective IgE function 4. The condition is X-linked 5. Cold abscesses occur 42-The following increases the pulmonary vascular resistanc e 1. Maximal inspiration 2. Maximal expiration 3. Anemia 4. Salbutamol 5. Prostacyclin

11

PEDIATRICS EXAMINATION REVIEW

43-Chronic lymphocytic leukaemia : 1. Is associated with smear or smudge cells in the peripheral blood 2. Is asymptomatic at diagnosis in les s than 10% of patients 3. Should be treated aggressively in the early stages of the disease 4. Has a median survival of less than 5 years 5. Is associated with splenomegaly at diagnosis in about 50% of patients

44-The following agents have good activity against Bacteroides fragilis :


1. Benzylpenicillin 2. Piperacillin / tazobactam 3. Cefoxitin 4. Metronidazole 5. Chloramphenicol

45-Digestion and absorption: Which of the following statements are true?


1. The glucose-containing polysaccharides starch, cellulose and glycogen are broken down for absorption by salivary and pancreatic amylases. 2. Trypsin, chymotrypsin, carboxypeptidases and amino peptidases are responsible for the breakdown of proteins for absorption. 3. Amphipathic phospholipids and bile salts convert micelles into lipid droplets ready for absorption. 4. The three known fat soluble vitamins A, D and E are taken up by fat absorbing processes in the gut. 5. About 9 litres of water enters the gut each day.

46-In asthma:1. The eosinophils to the lung are recruited from the bone marrow 2. Steroids are contra-indicated if the patient is a diabetic 3. Blocking interleukin-5 maybe therapeutic 4. Leukotriene inhibitors are associated with the development of Churg-Strauss . 5. Bronchial hyper-responsiveness (BHR) does not have to be present.

47-Regarding overdose, haemodialysis would be effective in removing drug in: 1. Digoxin 2. Pethidine 3. Amitriptyline 4. Propranolol 5. Salicylate

48-Heat inputs and outputs: Which of the following statements are true?
1. Changes in muscle activity are the main source of heat production for temperature control. 2. The skin is a good thermal insulator whose effectiveness is controlled through blood flow. 3. The blood flow through skin is controlled through vasodilator sympathetic nerves whose firing rate is increased in response to heat. 4. Sweating can increase body evaporative water loss from 0.6 to over 4.5 litres per day and eliminate 2.4 Mcal heat from the body. 5. Local humidity and air movement do not significantly influence heat loss.

12

PEDIATRICS EXAMINATION REVIEW

49-In gene regulation: 1. Chromatin plays no role. 2. Transcription factors control expression by binding to specific DNA sequen ces. 3. Transcription levels determine protein levels. 4. DNase I hypersensitivity sites are usually indicative of regions of active chromatin. 5. The stability of mRNA is entirely dependent on its length.

50-Sarah is eight years old and she wa s referred by her G.P with a velvety, pigmented rash on her neck and axilla. It is not itchy and she is not disturbed by it. She is the only child of her parents. Her weight is 49 kg and her G.P was concerned about this and has addressed this i ssue with her and her parents. Which one of the following investigations is the most relevant for her future management?
1. Fasting serum cholesterol/triglyceride 2. Fasting serum insulin/glucose ratio 3. Serum adrenocorticotrophic hormone (ACTH)/cortisol 4. Plasma leptin 5. Serum thyroid-stimulating hormone (TSH) 51-Congenital CMV infection: 1. Only 10% of affected pregnancies have resulting long term sequele at birth 2. Diagnosis is by viral isolation from the urine 3. Hearing loss can deveolp gradually over the first 5 years 4. The affected newborn should be treated with ganciclovir 5. Intracranial calcifications are seen in a periventricular distribution

52-In systemic lupus erythematosis (SLE)


1. Exacerbations may be associated with lymphadenopathy 2. There is an increased liability to infections 3. Intravenous immunoglobulins are commonly used in severe disease 4. Raynauds phenomenon is the commonest presenting symptom 5. Is associated with an increased risk of ischaemic heart disease

53-In scleroderma (systemic sclerosis): 1. Skin tightness may improve with time 2. Females are affected three times more commonly than men 3. Calcinosis only occurs in CREST form of the disease 4. Anti-Scl 70 antibodies occur in 50-60% of cases 5. Penicillamine may delay the development of internal organ complications

54-Regarding multicystic dysplastic kidney disease:


1. It is a rare anomaly that presents as an abdominal mass in the neonate 2. It is treated b y routine nephrectomy 3. UnilateraI lesions have an incidence of 1:2500 -1:4000 live births 4. There is usually 0% differential function on DMSA/ MAG3 imaging 5. Hypertension is a common complication of MCDK that warrants nephrectomy

13

PEDIATRICS EXAMINATION REVIEW

55-Regarding schistosomiasis:
1. Freshwater snails are the definitive host 2. Cercariae penetrate the skin and may lead to an itchy papular rash 3. Fever, urticaria and eosinophilia may be seen in the initial illness due to antigen -antibody complex formation 4. Schistosoma haematobium infection may lead to rectal bleeding 5. Schistosoma haematobium infection may lead to an obstructive uropathy

56-Regarding platelets:
1. They have a life span of approximately 10 days 2. Platelet production is controlled by specific cytokines 3. Platelets should be stored between 2 and 6 degrees C 4. Platelets can be stored for a maximum of 48 hours 5. A pool of platelets transfused can produce an increment of 50 000 / ml if platelet consumption is not an issue

57-Concerning Salmonella infection:


1. Results in a non-invasive enteric infection 2. Infection invariably occurs following oral ingestion of contaminated food products 3. Statutory notification is required 4. Antibiotic treatment is required for Salmonellosis 5. With typhoid 15-20% become chronic carriers

58-Carbon dioxide:
1. Does not affect plasma HCO3 concentration 2. Is transported in the blood as phosphate esters 3. Is transported in the blood in simple solution 4. Arterial CO2 rises during sleep 5. Arterial CO2 rises when living at an altitude of 9000 meters

59-Infantile hypertrophic pyloric stenosis:


1. Is more common in male babies than female babies 2. Is more common in the Afro-Caribbean population 3. Presents with projectile, bilious vomiting 4. Results in biochemical changes including low serum chloride, potassium and bicarbonate 5. Is surgically treated by excision of the pylorus muscle

60-The following are recognised causes of dr ug-induced immune thrombocytopenia:1. Chloramphenicol 2. Quinine 3. Penicillamine 4. Rifampicin 5. Heparin

14

PEDIATRICS EXAMINATION REVIEW

61-The following statements are correct regarding urinary analysis in acute renal failure:
1. There is a good correlation between functional renal impairment and the findings on urinalysis. 2. Pigmented 'muddy brown' granular casts suggest acute tubular necrosis. 3. Transparent hyaline casts are nonspecific and may be present in any case of ARF unrelated to the specific aetiology. 4. RBC casts suggest primarily renal arteriolar diseases such as vasculitis. 5. Presence of proteinuria argues against acute tubular necrosis and suggests a primary gomerular pathology.

62-Concerning meningococcal disease:


1. Is readily distinguished on clinical grounds from other infectious agents by the spreading purpura 2. Presents in seasonal epidemics 3. Frequently co-presents with meningitis, making LP necessary at presentation 4. Mannose binding protein deficiency is a known risk factor for meningococcaemia 5. Ciprofloxacin is used for routine contact prophylaxis

63-Features of the systemic inflammatory response syndrome (SIRS) include:


1. Core temperature <35 deg.cent. 2. Respiratory rate >16/min. 3. Hypercapnia with PCO2 >6 kPa. 4. Tachycardia >90 bpm. 5. Metabolic alkalosis.

64-The stiff -man syndrome :1. Is a syndrome associated with chronic inflammation of peripheral nerves. 2. Is a predominantly autoimmune syndrome. 3. Is uniquely associated with GAD antibodies. 4. Is associated with fibrillation and myokimeia. 5. Is best treated by benzodiazepenes and baclofen.

65-The following conditions will present with cyanosis in the first week of life:
1. Aortic stenosis 2. Transposition of the great vessels 3. Hypoplastic left heart syndrome 4. Fallot's tetralogy 5. Fallot's pentalogy.

66-Sturge-Weber syndrome :1. Occurs in 50% of children with port wine stains in the ophthalmic division of the trigeminal nerve 2. Develop hemiplegia on the ipsilateral side of the vascular lesion 3. Are associated with cataracts 4. Typically have slit like ventricles on CT 5. Skull radiograph shows 'pepperpot' skull

15

PEDIATRICS EXAMINATION REVIEW

67-Hypochromic microcytic anaemia is seen in:


1. Alpha thalassemia 2. Sickle cell anemia 3. Autoimmune hemolytic anemia 4. Folate deficiency 5. Imerslund syndrome

68-Which of the following is associated with common variable immunodeficiency?


1. Inflammatory bowel disease 2. Autoimmune disease 3. Normal levels of IgA 4. Granulomatous lesions 5. Susceptibility to recurrent parasitic infections

69-A 9 yr old girl is seen in clinic. Her teacher at school has reported that she has numerous episodes throughout the day of abrupt ly staring into space midconversation for a few seconds with rapid return of awareness. Which of the following features suggest complex partial rather than absence seizures as the cause of her problem : 1. Reporting unusual smells prior to the episodes 2. Provocation of the episodes by hyperventilation 3. Abnormal features on MRI scanning of the brain 4. A good response to anti-epileptic drugs 5. A normal EEG

70-The following statements are reagrding asthma:


1. Leukotriene antagonists are a first line treatment in asthma 2. The diagnosis of asthma involves blood tests 3. Treatment can be divided into those that prevent and those that relieve symptoms 4. Inhaled glucocorticoids improve lung function in asthmatics 5. Peak flow monitoring can be used in most children aged 5 years and above

71-Regarding cytokines:1. IL-1 causes pyrexia and induces protein synthesis in the liver 2. Interferon gamma causes increased expression of MHC class II molecules on antigen presenting cells 3. Chemokines can induce the transmigration of T cells into inflamed tissues 4. Can be used clinically to help treat infection 5. IL-2 s the main cytokine produced by nave T cells

72-15-year-old teenager presented with few haehorrha gic spots, agitation and confusion having been to school. Temp 39, pulse 110 and BP 120/80. The first management step would be : 1. Oxygen 2. Intravenous fluid 3. Throat swab 4. Intravenous antibiotic 5. Lumbar puncture

16

PEDIATRICS EXAMINATION REVIEW

73-The following congenital lesions are associated with cyanosis: 1. Aortopulmonary window 2. Total anomalous pulmonary venous drainage (TAPVD) 3. Coarctation of the aorta 4. Tetralogy of Fallot 5. Eisenmenger's syndrome

74-Causes of hypoadrenalism include:


1. Cushing's disease. 2. Severe sepsis. 3. Tuberculosis. 4. Amyloidosis. 5. Metastatic cancer.

75-In Wiskott -Aldrich syndrome:1. The immunodeficiency is restricted to T cells only 2. Platelets are of normal size, but are few in number 3. There is an increased risk of malignancy 4. IgA and IgE levels are often low 5. Inheritance is autosomal recessive

76-The following are consistent with myxedema:


1. Bradycardia 2. Intolerance to heat 3. Dry skin 4. Myopathy 5. Decreased sensitivity to opioids

77-The following tumour markers are strongly associated with the tumours shown:
1. Alpha-FP and hepatocellular carcinoma. 2. Beta-HCG and choriocarcinoma. 3. CA-19.9 and ovarian carcinoma. 4. CA-125 and pancreatic carcinoma. 5. CEA and colorectal carcinoma.

78-Arterial thromboses are associated with: 1. Behcets syndrome 2. Atrial fibrillation 3. Factor V Leiden mutation 4. Protein C deficiency 5. Polycythaemia

17

PEDIATRICS EXAMINATION REVIEW

79-Salbutamol:
1. Can cause an increase in PEFR 2. Has no effect on the FEV1 3. Has no effect on hypoxic-pulmonary vasoconstriction 4. May cause a rise in insulin release 5. Causes hypokalemia after prolonged use

80-Regarding hereditary angioede ma:


1. It is due to deficiency of complement component C1 2. It can present with abdominal pain and vomiting 3. Attacks can be triggered by exercise 4. C4 levels tend to be low during an attack 5. Danazol is used for treatment

81-Regarding Tanner stages of puberty:


1. Boys' genital development is described by 5 stages 2. Breast development stage 3 refers to breast buds 3. Pubic hair in the prepubertal state is described as Stage 0 4. Asymmetrical breast development is common in girls during puberty 5. Menarche is occurring in increasingly younger girls over the past three decades

82-Hereditary angioedema is:1. Inherited in an autosomal recessive fashion 2. A consequence of C2 kinin production 3. A cause of spontaneous life-threatening laryngeal oedema 4. Sometimes treated with the anabolic steroid danazol 5. Treated with regular C1 inhibitor infusions

83-Regarding the meninges:


1. The dura is adherent to the bone 2. Subdural hematomas form a lentiform shape 3. Arachnoid folds project inwards to form the falx cerebri and falx cerebelli 4. Pia stimulation during surgery is very stimulating 5. Pia is responsible for production of CSF

84-Pectus excavatum is associated with:


1. Marfan's syndrome 2. Rickets 3. Upper airway obstruction 4. Exercise limitation 5. Rightward shift of the heart

18

PEDIATRICS EXAMINATION REVIEW

85-Renal ultrasound:
1. Is a reliable means of looking for vesico ureteric reflux (VUR). 2. Has a false negative result of 10 - 20% when looking for renal scarring. 3. Has a false negative result of 15 - 45% when looking for renal scarring. 4. Has a false positive rate of 25 - 50% when looking for VUR. 5. Is a good means of detecting distal ureteric dilation.

86-Regarding P.Carinii prophylaxis:


1. It is recommended in all infants born to HIV infected mothers 2. Intravenous Pentamidine is recommended 3. Previous P. Carinii infection warrants prophylaxis 4. Recommended when CD4 count drops below 1000microlitre 5. Oral candidiasis in HIV patients warrants Prophylaxis

87-The following are true about acne: 1. Females with acne commonly have elevated blood androgens 2. Oral steroids are a recognised cause 3. Interleukin 1 alpha promotes comedogenesis 4. Oral isotretinoin causes hyperlipidaemia 5. The enzyme Type 1 5 alpha reductase is expressed in sebaceous gland ducts

88-The following criteria must exist before a diagnosis of brain death can be made:
1. Fixed, dilated pupils unresponsive to light. 2. Absent gag reflex with pharyngeal stimulation. 3. Neuromuscular drugs should have been stopped for at least twelve hours. 4. No eye movement when 20 mls cold water is perfused into the external auditory meatus. 5. No motor response fr om painful stimuli.

89-The following are true regarding parathyroid hormone actions:


1. Increases calcium and decreases phosphate reabsorption in kidney 2. Increases osteoclastic activity 3. Increases 1,25 dihyroxy vitamin D3 production in the kidney 4. Decreases calcium and increases phosphate reabsorption in kidney 5. The overall effect is to increase calcium and phosphate level in the blood

90-Selective IgA deficiency: 1. Is treated with iv sandoglobulin 2. is very rare (incidence less than 1 in 10000) 3. May present with autoimmune disease 4. IgG2 subclass deficiency may also be present with more severe infective complications 5. is associated with HLA-DR3

19

PEDIATRICS EXAMINATION REVIEW

91-The following increase vasopressin secretion:


1. Vomiting. 2. Alcohol. 3. Haemorrhage. 4. Exercise. 5. Morphine.

92-Regarding ketamine:
1. Is contra-indicated in reversible airways obstruction 2. Is more emetogenic than thiopentone 3. Has anti-sialogogue properties 4. May be used safely in patients with suspected raised intra -cranial pressure 5. Is a potent respiratory depressant

93-Sickle cell disease : 1. Is caused by a mutation on chromosome 16 2. Will give a positive sickle test result at birth 3. Is associated with strokes in 25% of patients by the age of 45 years 4. Is an indication for autologous haemopoietic stem cell transplant 5. Can be co-inherited with alpha or beta -thalassaemia

94-The femoral artery:


1. Runs as a single trunk from the inguinal ligament to the lower border of the popliteus 2. Is a direct continuation of the internal iliac 3. In the upper third of the thigh the femoral artery is contained in the femoral triangle (Scarpas triangle) 4. In the lower third of the thigh, lies in the adductor canal (Hunters canal) 5. The superficial epigastric artery is a branch of this vessel

95-Coarctation of the aorta: 1. Is typical of Marfans Syndrome 2. Is typical of Noonan syndrome 3. Is associated with notching of the anterior ribs 4. Renal failure may arise, especially in the neonatal period. 5. Radiofemoral delay is the diagnostic sign

96-Anti-diuretic hormone (ADH):


1. Is a decapeptide 2. Has a half-life in the plasma of approximately 24 hours 3. Acts on the distal tubule of the kidney to increase reabsorption of water 4. Secretion can be suppressed by morphine 5. Secretion is increased by pain .

20

PEDIATRICS EXAMINATION REVIEW

97-In recurrent abdominal pain in children, the following suggest an underlying organic cause:
1. presentation at age 10 2. pain localising close to the umbilicus 3. family history of migraine 4. pain during daytime only 5. vomiting

98-The following are inherited in an autosomal dominant manner


1. Neurofibromatosis type I 2. Hereditary spherocytosis 3. Congenital adrenal hyperplasia 4. Galactosaemia 5. G6PD deficiency

99-The following skin conditions are by definition associated with epithelial dysplasia:
1. Xeroderma pigmentosum 2. Bowens disease of skin 3. Basal cell carcinoma 4. Actinic keratosis 5. Seborrhoeic keratosis

100-Pulmonary surfactant
1. Is partly recycled by endocytosis into the synthesizing cell 2. Is produced by type 1 alveolar cells 3. Reduction in pulmonary flow can cause a decrease in surfactant production 4. Synthesis is inhibited by thyroxine 5. Synthesis is stimulated by glucocorticoids

101-Regarding sleep disturbance:


1. Sleep terrors occur in non -REM sleep 2. Nightmares are associated with appre ciable body movements 3. Sleep terrors last at least twenty minutes 4. Sleep terrors are associated with intense vocalisation 5. Nightmares are associated with intense vocalisation

102-Nitric Oxide (NO):


1. Is critical in increasing endothe lin (ET-1) levels. 2. Is important in the maintenance of pulmonary vascular flow. 3. Increases cAMP. 4. Operates by inhibiting PDEs. 5. Is the same substance as EDHF.

21

PEDIATRICS EXAMINATION REVIEW

103-Factors associated with poor wound healing include:


1. Vitamin A deficiency 2. Vitamin B1 deficiency 3. Vitamin C deficiency 4. Folate deficiency 5. Zinc deficiency

104-The following are true of COPD exacerbations: 1. All patients should be given antibiotics 2. Are associated both with rhinovirus and chlamydia pneumoniae infection 3. There is a positive correlation in mortality and the number of smoking pack years 4. pH less than 7.35 is an indication for antibiotics according to the BTS guidelines 5. Moraxhella catarrhalis is the most freque ntly implicated organism in exacerbation

105-The following statements are correct regarding the cardiac anatomy:


1. The left atrium is larger in size as compared to the right atrium. 2. Right coronary artery is dominant in over three -fourth of cases. 3. There is one pulmonary vein from each lung that enters the posterior wall of the left atrium. 4. The AV node is supplied by the left anterior descending artery in most of the cases. 5. Most of the venous drainage of the heart occurs via the the Basian system of veins.

106-Mitral valve prolapse is associated with: 1. Cyanosis 2. Bacterial endocarditis 3. Chest pain 4. A systolic 'click' on auscultation 5. Foetal arrythmias

107-Normal arterial pH is :
1. Equal to [H+] of 34-46 nmoles/L 2. Lower than intracellular pH 3. Not essential to maintain ion distribution 4. Measured rather than calculated in a pH probe 5. Decreased in pregnancy

108-A 16 year old girl is admitted with progressive weight loss for whi ch no organic cause can be found. The following factors would be against a diagnosis of anorexia nervosa: 1. A history of laxative abuse 2. The delusion that someone is trying to control her eating patterns 3. An excess of physical exercise 4. Self-induced vomiting 5. Low plasma potassium

22

PEDIATRICS EXAMINATION REVIEW

109-Defective Fas-FasL interaction leads to impairment of : 1. CTL-mediated lysis 2. Helper T-lymphocyte activation 3. NK cell killing 4. Oxidative burst within activated macroph ages 5. Recruitment of neutrophils

110-The following poisons are matched to the appropriate therapy: 1. Carbon monoxide - hyperbaric oxygen 2. Organophosphates - atropine and pralidoxime 3. Beta-blocker - phentolamine 4. Methanol - ethanol 5. Tricyclic antidepressants - phenytoin

111-The following are true:


1. The uptake of iodide into the thyroid follicular cell is by active transport 2. T4 has a half-life of 24 hours 3. About 99% of circulating T4 is protein boun d 4. T3 is roughly 10 times more potent than T4 5. All steps in thyroid hormone synthesis are stimulated by thyroid peroxidase

112-Restriction fragment length polymorphisms (RFLPs): 1. Are found every 100 bases 2. Are useful in diagnosis of multifactorial diseases 3. Are useful in linked-gene diseases 4. Are generated by digestion of DNA with bacterial enzymes 5. Are detected by Northern blotting

113-Which of the following statements are true:


1. Data may be described as qualitative or quantitative 2. Discrete data may include number of children 3. Discrete data may include serum glucose values 4. Ordinal data does not include tumour staging 5. Nominal data may include blood group type

114-In an athymic child:1. Macrophage function is impaired 2. The development of gamma/ delta -T-lymphocytes is unaffected 3. The production of antibody is unaffected 4. The function of helper T-lymphocytes (Th) is affected while cytotoxic T -lymphocytes (Tc) activity is only moderately impaired 5. Lymphadenopathy is common because of the over -expansion of B-lymphocytes

23

PEDIATRICS EXAMINATION REVIEW

115-The following statements are true about the mitral valve


1. It is bicuspid 2. It lies posterior to the sternum at the level of the fourth costal cartilage 3. The associated papillary muscles are smaller than those of the tricuspid valve 4. Its' opening is associated with the first heart sound 5. It can be regurgitant even if structurally normal

116-Causes of vesicular rash in children include: 1. Roseola Infantum 2. Dermatitis herpetiformis 3. Hand Foot and Mouth disease 4. Kawasaki disease 5. Molluscum contagiosum

117-The following are causes of lymphadenopathy:


1. Rubella 2. Amyloidosis 3. SLE 4. Rheumatoid arthritis 5. Thyrotoxicosis

118-The following statements are true for the Renin Angiotensin System: 1. Renin converts Angiotensin I to Angiotensin II in the kidney 2. Angiotensin II promotes renal arteriolar dilatation 3. Angiotensin II inhibits aldosterone secretion by the adrenal glands 4. Angiotensin II receptors are blocked by spironolactone 5. Losartan inhibits formation of Angiotensin II

119-Which of the following are common symptoms associated with raised intracranial pressure in children?
1. Morning headache 2. Abdominal pain 3. Double vision 4. Ataxia 5. Seizure

120-The following is true regarding endotoxin: 1. It is a protein 2. It is heat-stable 3. It is produced by some Gram-positive bacteria 4. All endotoxin has the same physiological effect 5. It is secreted by bacteria during active infection

24

PEDIATRICS EXAMINATION REVIEW

121-The following are zoonoses:


1. Legionnaires disease 2. Hydatid disease 3. Lyme disease 4. Poliomyelitis 5. Salmonellosis

122-Regarding Parvovirus infections in children: 1. It is the causative agent for exanthem subitum 2. Arthritis is a very common manifestation in children 3. Chronic anemia can be a manifestation in some children 4. Infection in pregnancy can result in fetal anemia 5. Aplastic crises due to parvovirus infection in sickle cell disease usually requires bone marrow transplant

123-Definition of status epilepticus:


1. Two or more fits within 30 minutes 2. Unresponsive patients who have been incontinent for 60 minutes 3. Mouth twitching lasting 60 minutes with reduced GCS 4. Tonic-clonic seizure lasting 30 minutes with incontinence 5. Rapidly repeating tonic -clonic seizure with not returning to full c onsiousness

124-Regarding Gross Motor Development, a normal child would be expected to: 1. Roll over by 2 months 2. Ride a tricycle at 2 years 3. Kick a ball at 2 years 4. Cruise by 1 year 5. Not run until 2 years

125-The functional consequences of moderately severe mitral stenosis include:


1. Reduced lung compliance. 2. Hypercapnia. 3. Reduced glomerular filtration rate. 4. Raised left atrial mean pressure. 5. Reduced pulmonary vascular resistance.

126-Evaluate the truth of the following descriptions of cytokines and their functional activities:1. IL-3 : growth and differentiation of bone-marrow stem cells 2. IL-2 : activation induced cell death 3. IL-10: activation of B-cells 4. IL-6: induction of antibody secretion by plasma cells 5. IL-12: induction of IFN-gamma secretion by activated T-cells

25

PEDIATRICS EXAMINATION REVIEW

127-Urine
1. normal pH is >5.5 2. normal Na content is <20mmol/L 3. normal osmolality is 300 -600 mosmol/kg 4. protein content of 500mg/day is within normal range 5. functional minimum urine output/day in 70kg male is 1400mls

128-In children with renal failure


1. In acute renal failure, dialysis is indicated with rapidly rising sodium 2. Anaemia of chronic renal failure is due to low plasma ferritin levels 3. Protein intake should be restricted in chronic renal failure. 4. In haemolytic uraemic syndrome, microangiopathic haemolytic anaemia is characteristic 5. Grade V vesico-ureteric reflux spontaneously resolves in only 10%

129-Lung surfactant
1. Decreases the surface tension within an alveolus 2. Causes an increase in chest wall compliance 3. Is a glycoprotein 4. Maintains the same surface tension for different sized alveoli 5. Appears only after the 1st week of life

130-With regards to bronchiolitis, the following are true : 1. It is linked to maternal smoking. 2. RSV immune globulin is recommended in infants with cyanotic congenital heart disease to prevent severe RSV infection. 3. Ribavarin is curative. 4. Lymphopenia is common. 5. CXR commonly shows hyperinflation.

131-The following concerning gastrointestinal hormones are true


1. Somatostatin reduces gastric motility 2. Gastric distension increased motility 3. Glucagons increases GI motility 4. Secretin inhibits pancreatic bicarbonate secretion 5. Pancreatic polypeptide inhibits pancreatic secretion

132-Normal arterial pH is:


1. Equal to [H+] of 34 - 46 nmoles/L. 2. Lower than intracellular pH. 3. Not essential to maintain ion distribution. 4. Measured rather than calculated in a pH probe. 5. Decreased in pregnancy.

26

PEDIATRICS EXAMINATION REVIEW

133-The following organisms cause conjunctivitis: 1. Epstein Barr virus 2. Chlamydia trachomatis 3. Adenovirus 4. Haemophilus influenzae 5. Neisseria gonorrhoeae

134-The following statements are true:


1. The vagus nerve is responsible for normal bronchial tone 2. There are 18 bronchopulmonary segments 3. Most of the alveoli in the adult lung are present at birth 4. The surface markings of the right middle lobe approximate to the axilla 5. The major component of surfactant is dipalmitoyl lecithin

135- In beta thalassaemia:


1. Beta thalassaemia major presents at birth with profound anaemia 2. The reticulocyte count is low 3. Diagnosis can be made by Haemoglobin electrophoresis 4. Anaemic patients should receive regular iron supplementation 5. Splenectomy is contraindicated due to extramedullary haemopoiesis

136-The following statements concerning CNS tumours are true:


1. Glioblastoma multiforme is a benign neoplasm. 2. Astrocytomas arise from the neuroglia. 3. Cerebral metastases comprise around 70% of all brain tumours. 4. Gastric carcinoma frequently metastasises to the brain. 5. Meningioma is usually benign.

137-Regarding lead poisoning:1. Urinary lead concentration is the most useful indicator of exposure 2. In asymptomatic children with increased lead levels DMSA therapy is recommended 3. Haemodialyisis effectively eliminates lead 4. The half-life of the deep bone compartment is months 5. Is associated with basophilic stippling in white cells

138-Benign rolandic epilepsy :1. Is a common form of partial epilepsy 2. Has a poor prognosis 3. Involves seizures that often occur at school 4. Can be successfully treated with carbamazepine 5. Has a typical EEG with a 3 per second spike and wave pattern

27

PEDIATRICS EXAMINATION REVIEW

139-Which of the following biological responses or consequences is mediated b y the release of tumour necrosis factor (TNF) alpha?
1. Production of acute phase proteins by hepatocytes 2. Mobilisation of B-cells from the bone marrow 3. Disseminated intravascular coagulation in systemic Gram -negative bacterial infection 4. Resetting of the temperature threshold in the hypothalamus resulting in pyrexia 5. Induction of killing by cytotoxic T lymphocytes (CTL)

140-The neck of the 1st rib is related to:


1. The thoracic duct on the right. 2. The superior intercostal ar tery. 3. Scalenus anterior. 4. Phrenic nerve. 5. Stellate ganglion.

141-Which of the following would cause an increase in the carbon monoxide transfer factor?
1. Extra-pulmonary restriction 2. Churg-Strauss 3. Goodpastures syndrome 4. Exercise 5. Anaemia

142-A patient develops low back pain which is progressive. He is found to a spinal schwannoma. It is resected, but division of the posterior spinal root occurs. Which of the following is likely to result from this?
1. Loss of motor function 2. Loss of tendon reflex 3. Loss of pain, temperature and preservation of light touch 4. Spinothalamic tract degeneration above the section level 5. Absence of sweating in the limb

143-Autism :1. Can be treated successfully with psychotherapy 2. Is sometimes related to poor parenting 3. Is related to Fragile X syndrome 4. Is related to Sandifer Syndrome 5. Is related to Rett syndrome

144-With regards the spleen:


1. The lymphoid function is carried ou t in the white pulp. 2. Following splenectomy, it is largely the kupffer cells in the liver that filter micro -organisms. 3. Following splenectomy, low dose penicillin & pneumovax are recommended. 4. The spleen is involved in the destruction of enc apsulated organisms. 5. Basophilic stippling of neutrophils is seen with asplenia.

28

PEDIATRICS EXAMINATION REVIEW

145-Chronic lymphocytic leukaemia :1. Is associated with smear or smudge cells in the peripheral blood 2. Is asymptomatic at diagnosis in less than 10% of pati ents 3. Should be treated aggressively in the early stages of the disease 4. Has a median survival of less than 5 years 5. Is associated with splenomegaly at diagnosis in about 50% of patients

146-In asthma:1. The eosinophils to the lung are recruited from the bone marrow 2. Steroids are contra-indicated if the patient is a diabetic 3. Blocking interleukin-5 maybe therapeutic 4. Leukotriene inhibitors are associated with the development of Churg -Strauss . 5. Bronchial hyper-responsiveness (BHR) does not have to be present.

147-With regards severe combined immunodeficiency: 1. Patients have defective T-cell dependent antibody responses and defective cell -mediated immunity 2. Patients are susceptible to infection with bacte ria and viruses 3. X-linked SCID is caused by a defect in the IL -2 receptor 4. ADA (adenosine deaminase) and PNP (purine nucleotide phosphorylase) deficiency cause accumulation of nucleotide metabolites that are toxic to developing T -cells resulting in T-cell and B-cell deficiencies and a SCID phenotype 5. SCID can be due to an autosomally inherited defect in DNA repair analogous to the genetic defect in the SCID mouse strain

148-Lower esophageal sphincter tone is decreased by:


1. Cisapride 2. Progesterone 3. Suxamethonium 4. Ganglion blockers 5. Ondansetron

149-A normally developing 18 -month-old child:


1. Should recognise his/her name 2. Can easily distinguish primary colours by name 3. Plays in parallel with his/her peers 4. Should imitate observed actions 5. Should be able to copy a vertical line

150-The following statements concerning congenital heart disease (CHD) are true: 1. VSDs account for 70% of all CHD. 2. ASDs account for 40% of all CHD 3. Approx. 13% of neonates with chromosomal abnormalities have associated CHD 4. Approx. 80% of children with Downs Syndrome have CHD 5. Approx. 40% of patients with Turners Syndrome have coarctation and -or aortic stenosis.

29

PEDIATRICS EXAMINATION REVIEW

151-The following are clues to the cause of an upper gastrointestinal (GI) bleed:
1. Purpuric rash suggestive of Henoch-Schonlein purpura (HSP) 2. Hypopigmented spots on the lips suggestive of Peutz -Jeghers syndrome 3. Webbed neck and low hairline 4. Perianal skin tags 5. Haematochezia due to peptic ulceration

152-The muscles of the soft palate are:


1. Tensor veli palatini 2. Palatoglossus 3. Levator veli palatini 4. Hyoglossus 5. Palatopharyngeus

153-Horners syndrome invariably produces :1. Ipsilateral ptosis 2. Contralateral mydriasis 3. Loss of sweating over the ipsilateral face 4. Heterochromia 5. Exophthalmos

154-An abnormal response to oral glucose loading is seen in: 1. Malabsorption 2. Acromegaly 3. Partial gastrectomy 4. Wilson's disease 5. Gestational diabetes

155-Metabolic influences on pH 2: Which of the following statements are true?


1. Metabolic alkalosis can be induced by vomiting from the stomach. 2. Metabolic alkalosis is the reduc tion in H+ concentration by any means other than decreased PCO2. 3. Ventilation is reflexly depressed by metabolic alkalosis. 4. Increased central chemoreceptor firing rates decrease ventilation rates. 5. Hyperventilation induced by metabolic alka losis returns arterial pH to the normal pre -set points.

155- With regard to the visual pathways:


1. A lesion of the optic chiasma will cause a homonymous hemianopia 2. A lesion of the left occipital lobe will result in a right homonymous hemianop ia 3. The pupillary light reflex is preserved in lesions of the optic nerve 4. Lesions of the optic nerve cause an enlargement of the blind spot 5. A pituitary adenoma may cause a bitemporal hemianopia

30

PEDIATRICS EXAMINATION REVIEW

156-The O2-Hb dissociation curve is displaced to the right in:


1. Fetal Hb 2. Methemoglobinemia 3. Increased lactic acid production 4. Chronic hypoxia due to cyanotic conditions 5. Increased 2,3 DPG

157-Intravenous immunoglobulin (IVIG) is used in the treatment of: 1. Guillain-Barre syndrome 2. IgA deficiency 3. Idiopathic thrombocytopenia purpura 4. Kawasaki disease 5. Metachromatic leukodystrophy

158-In the kidney


1. The filtration fraction is the ratio of GFR to renal blood flow 2. Normal filtration fraction is 0.1-0.2 3. Renal blood flow = plasma flow/(1 - HCT) 4. GFR can be measured using Doppler ultrasound 5. GFR can be measured using a thermodilution technique

159-In enuresis :1. In a 3 year old investigations must be carried out 2. 5% of 5 year old children are affected 3. Urine dipstick is a useful investigation 4. First line treatment is imipramine 5. Conditioning is usually successful

160-Endocrine disorders that result in diarrhoea include: 1. Cushings syndrome 2. Hyperparathyroidism 3. Thyrotoxicosis 4. Primary hyperaldosteronism 5. Carcinoid syndrome

161-Regarding complex partial epilepsy: 1. Absence seizures and generalised tonic -clonic seizures may occur 2. Consciousness is altered 3. Hyperventilation often provokes a complex partial seizure 4. EEG changes disappear in sleep 5. Ethosuximide is an affective treatment

31

PEDIATRICS EXAMINATION REVIEW

162-In osteogenesis imperfecta


1. There is an increased incidence of osteosarcoma 2. Blue sclera is diagnostic 3. Pamidronate is a recognised treatment in children with this condition 4. Hearing abnormalities are common 5. Diagnosis is usually made by bone biopsy

163-Ventricular septal defects:


1. Can present with heart failure in the neonatal period 2. Results in wide splitting of the second heart sound 3. May regress by incorporation of the tricuspid leaflet tissue 4. Tricuspid atresia is a recognised association 5. Cardiac bypass is not necessary for surgical correction

164-Neostigmine:
1. Has been used in the treatment of SVT 2. Prolongs the duration of action of suxamethonium 3. Has an oral bioavailability of 50% 4. Is metabolized by plasma esterases 5. Crosses the placenta

165-Recognized associations of systemic conditions and cataracts include:


1. Muscular dystrophy 2. Galactosemia 3. Hyperthyroidism 4. Relapsing polychondritis 5. Septicemia

166-The following statements are true: 1. A moro reflex with the fists remaining clenched is normal. 2. The plantar grasp persists longer than the palmar grasp 3. Deep tendon reflexes are difficult to elicit at birth 4. An extensor Babinski response is normal in the first year 5. A newborn baby from 32 weeks gestation will turn tow ards a light

167-Glucocorticoids:
1. Are secreted from the zona fasciculata within the adrenal cortex. 2. Molecules consist of 21 carbon atoms. 3. Cause retention of sodium. 4. Inhibit gluconeogenesis. 5. Reduce lymphocyte counts.

32

PEDIATRICS EXAMINATION REVIEW

168-In the treatment of attention deficit hyperactivity disorder : 1. Behavioural therapy is never useful 2. Pharmacological intervention includes the use of stimulant drugs such as amphetamines 3. Methylphenidate (Ritalin) should not be given late r than 4pm 4. Parenting techniques can be taught 5. Poor appetite is a common side affect of methylphenidate (Ritalin)

169-Concerning the synthesis of thyroid hormones:


1. TRH is released by the anterior pituitary. 2. Iodine is taken up passively by the thyroid gland. 3. Thyroid peroxidase catalyses the formation of thyroxine. 4. T4 is released bound to thryroglobulin. 5. T4 is released in greater quantities than T3.

170-The following diseases are inherited in an autosomal recess ive manner:1. Insulin dependant diabetes 2. Primary ciliary dyskinesia 3. Batten's disease 4. Congenital adrenal hyperplasia 5. Pyloric stenosis

171-Noradrenaline
1. Is a neurotransmitter at the pre-ganglionic sympathetic nervous syst em 2. Is produced by the adrenal cortex 3. May cause a reflex bradycardia when given by infusion 4. Causes a rise in metabolic rate 5. It acts predominantly via the beta adrenoceptor

172-In a healthy baby the transition from fetal to neon atal circulation involves:1. Functional closure of the foramen ovale in the first 24 hours 2. Blood flow in the ductus arteriosus continues from right to left until its closure 3. Decrease in pulmonary artery resistance following closure of the ductus arteriosus 4. The ductus arteriosus closes in response to decreased oxygen concentrations 5. The umbilical artery is a branch of the common iliac artery

173-The pancreas:
1. Is entirely retroperitoneal 2. Is supplied by branches of th e coeliac artery alone 3. Is crossed by the attachment of the transverse mesocolon 4. Is crossed by the left renal vein 5. Is related to the common bile duct posteriorly

33

PEDIATRICS EXAMINATION REVIEW

174-IgA deficiency:1. Is seen in Kartagener's syndrome 2. Is usually asymptomatic 3. May delay diagnosis of coeliac disease 4. Spontaneous recovery is a recognised feature 5. Affects both serum and secretory IgA levels

175-Abnormalities associated with foetal alcohol syndrome include: 1. Short palpebral fissures 2. Maxillary hyperplasia 3. Hyperactivity in childhood 4. Intra uterine growth retardation 5. Hypotonia

176-The seventh cranial nerve:


1. Has bilateral cortical representation. 2. Receives taste fibres from the palate. 3. When lesioned below the level of the stylomastoid foramen will cause hyperacusis. 4. Receives taste fibres from the anterior 2/3rd of the tongue via the greater petrosal nerve. 5. Has a sensory supply in the ear canal.

177-Strokes in childhood are associated with :1. Nephrotic syndrome 2. Activated protein C resistence 3. Systemic lupus erythematosis 4. Haemoglobin SS 5. Moya moya disease

178-The following statement(s) concerning inherited immunodeficiency syndromes are true:


1. The commonest inherited form of severe combined immune deficiency is x -linked 2. The incidence of lymphoid malignancy is increased in ataxia telangiectasia 3. The defect in Chediak-Higashi syndrome is in T-cell activation 4. Deficiency in glucose-6-phosphate dehydrogenase results in impaired motility of leukocytes 5. The molecular defect in X -linked hyper IgM syndrome is in the expression of CD40 on B -cells

179-The following diseases may be transmitted by needlestick injury:


1. Hepatitis C. 2. Hepatitis B. 3. Malaria. 4. Tuberculosis. 5. HIV.

34

PEDIATRICS EXAMINATION REVIEW

180-Leptospirosis: 1. Most recover within two weeks without treatment 2. May be diagnosed on urine culture 3. Is treated with tetracycline 4. May be complicated by thrombotic thrombocytopenic purpurea 5. Is more common around the New Forest

181-The lymphatic system:


1. Is a valveless system 2. Maintains a uni-directional flow of lymph towards the thoracic ducts 3. The thoracic ducts drain into the venous circulation at the junction of the sub -clavian veins and internal jugular veins 4. Has a total lymph drainage of 6-8 litres of fluid/24 hours from interstitial fluid 5. Contains lymph draining from the liver and intestine with a protein concentrati on of approximately 40 -60 g/l

182-The following have a much lower concentration in the cerebrospinal fluid (CSF) than in the cerebral capillary blood.
1. K+ 2. Na+ 3. Osmolarity 4. Protein 5. Sugar

183-Co-administration of the followin g pairs of drugs results in reduced absorption of the latter:1. Antacids and prednisolone 2. Cholestyramine and warfarin 3. Sucralfate and phenytoin 4. Amitriptyline and digoxin 5. Metoclopramide and paracetamol

184-Toe-walking in an 18 month old child :1. Is always abnormal 2. Spinal cord imaging may be indicated 3. Serum creatine kinase should be measured 4. Does not need to be investigated in a girl 5. May be a sign of cerebral palsy

185-Conn's syndrome:
1. Is more common in females. 2. Is due to atrophy of zona glomerulosa. 3. Causes muscle weakness. 4. Leads to sodium retention. 5. Leads to increased levels of plasma angiotensinogen 2.

35

PEDIATRICS EXAMINATION REVIEW

186-Haemochromatosis:1. Is associated with vitamin C deficiency 2. Is associated with Jaccoud's arthropathy 3. Is more common in men 4. Is associated with HLA B8 5. Is associated with a gene on chromosome 13

187-Phagocytic Cells :1. Have receptors that recognise bacterial cell wall constit uents 2. Require clonal expansion to mount an effective response 3. Release cytokines wich affect the adaptive immune system 4. Are important for the resolution of inflammation 5. Initiate firm attachment by binding to ICAM on the endothelium

188-With regards to aniridia: 1. Two thirds of the cases are sporadic. 2. It is usually bilateral. 3. Vision is commonly decreased. 4. It is associated with partial deletion of long arm of chromosome 11. 5. Wilm's tumour is associated with familial aniridia.

189- Immune complex deposition in the kidney occurs in:


1. Goodpasture's disease 2. Infective endocarditis 3. Systemic lupus 4. Polyarteritis nodosa 5. Plasmodium malariae infection

190-Regarding oxygen measurement:


1. Red light absorbs more oxyhemoglobin than deoxyhemoglobin 2. The isobestic point will vary with hemoglobin concentration 3. The readings from pulse oximeters are affected by anemia 4. The clark electrode uses a silver/silver chloride cath ode 5. Water vapor will cause inaccurate readings from a paramagnetic analyzer

191-The following syndromes/ associations are associated with congenital heart disease.
1. holt-Oram syndrome. 2. CHARG association. 3. Vacterl association. 4. Septo-optic dysplasia. 5. Goldenhar

36

PEDIATRICS EXAMINATION REVIEW

192- Regarding myasthenia gravis in childhood:


1. The neonatal form is always due to maternal antibodies. 2. The diagnostic test is subcutaneous endophonium. 3. EMG shows decremental response to repetitive motor nerve stimulation. 4. Thyroid dysfunction is a recognised association. 5. The disease can be confined to ocular muscles.

193- The following statements regrd the treatment of asthma:


1. All patients should be on inhaled steroids. 2. High dose regular short acting bronchodilators should be tried before inhaled steroids. 3. Either long acting beta-2 agonists (LABAs) and low dose steroid or high dose inhaled steroid can be used to the same clinical effect. 4. Patient education is emphasized. 5. Leukotrene inhibitors are recommended in step 4.

194- Regarding child development:


1. Sense of taste is present from birth. 2. 25% of children are left handed. 3. Left handedness is more common in exprem babies without cerebra l palsy. 4. Left handed children are more likely to have un intentional injuries. 5. Prevalence rate of thumb-sucking in children are universal across races.

195-Persistant, conjugated hyperbilirubinaemia may be caused by:


1. Alpha-1-antitrypsin deficiency. 2. Hypothyroidism. 3. Haemolytic disease. 4. Cytomegalovirus infection. 5. Cystic fibrosis.

196-Complete transaction of the oculomotor nerve results in :


1. Ptosis. 2. Convergent squint. 3. Constriction of the pupil. 4. Loss of accommodation and light reflex. 5. Diplopia.

197-The following involve deficiencies in T -cell immunity:


1. Di George syndrome. 2. Wiskott-Aldrich syndrome. 3. Chronic granulomatous disease. 4. Brutons agammaglobulinaemia. 5. Chronic mucocutaneous candidiasis.

37

PEDIATRICS EXAMINATION REVIEW

198-In young children.


1. Night terrors occur in the stages of deep sleep. 2. Sleep walking occurs during REM sleep. 3. Sleep disorders are never treated with drugs. 4. Breath holding attacks cause cyanosis. 5. Breath holding attacks should be investigated with CT scan.

199-The following maternal conditions are known to cause adverse effects on the neonate:
1. Diabetes insipidus. 2. Toxoplasmosis. 3. Chronic myeloid leukaemia. 4. SLE. 5. Hyperthyroidism.

200-en uptake by haemoglobin is characterized by: 1. Alkalosis. 2. Raised temperature. 3. Increased 2-3 DPG. 4. Increased red cell volume and MCH. 5. High altitude.

38

PEDIATRICS EXAMINATION REVIEW

Answer &Teaching Notes for Questions


(PART 1)

39

PEDIATRICS EXAMINATION REVIEW

1- Regarding hepatitis B virus (HBV):


1. HBV is an RNA virus 2. HBV only replicates in the liver 3. HBsAg is the first serological marker to appear 4. The presence of HbsAg indicates acute infection only 5. The persistence of HbeAg indicates infectivity Teaching Notes for Question 1 Theme: Hepatitis B HBV is a double stranded DNA virus (hepadenovirus) and only replicates in the liver. 1015% prevalence in Africa/Far East; across Europe HBV and HCV cause approximately 50% hepatocellular carcinomas. It is spread by blood Tx/IVDA/tatooists/sexually/vertically (most important worldwide). Incubation of virus: 6 weeks 6 months. MARKERS AG: * the first screening Ix is HBsAg, if found, a full profile is requested. HbsAg appears at approximately 610 wks indicating acute infection and then vanishes if virus has cleared, but persists in chronic infection (patient then becomes a carrier). * HbeAg appears at 6 wks and disappears at 10 wks as Anti -HBe rises ('seroconversion'). Persistence of HbeAg indicates increased severity and infectivity. MARKERS AB: * Anti-HBc (IgM) - First to appear at approximately 8 wks, until 12 wks. Indicates continued viral replication. * Anti-HBe appears next (10 wks), indicates reduced infectivity. * Anti-HBs appears last (12 wks), indicates immunity. Most sensitive test of continued viral replication is PCR for HBV DNA.

2- The following statements are true regarding superantigens: 1. They are very strongly antigenic bacterial polysaccharides 2. They bind to B cell receptors 3. Lead to excess stimulation of T cell proliferation and interleukin -2 production 4. They are responsible for gram-negative toxic shock 5. Lead to stimulation of production of TNFa and other cytokines Teaching Notes for Question 2 Theme: Superantigens Superantigens are an unusual type of bacterial toxin comprising of proteins which exert their effect by forming a bridge between the MHC complex of antigen -presenting cells and T cell receptors. Because the binding to T cell receptors is indiscriminate, excess stimulation of T cell proliferation occurs leading to abnormally high levels of IL-2, with symptoms of nausea, vomiting, fever and malaise. They have a role in Gram-positive septic shock, particularly toxic shock syndrome and staphylococcal food-borne disease. It is possible that they may also have a role in the pathogenesis and clinical features of Kawasaki disease.

40

PEDIATRICS EXAMINATION REVIEW

3- Causes of pulmonary fibrosis include:1. Sulfasalazine 2. Chlorpromazine 3. Penicillamine 4. Methysergide 5. Nitrofurantoin Teaching Notes for Question 3 Theme: Pulmonary fibrosis Causes of pulmonary fibrosis include amiodarone, nitrofurantoin, sulfasalzine, and cytotoxics (e.g. azathioprine, bleomycin, busulphan, bleomycin, chlorambucil, cyclophosphamide, melphalan, and methotrexate). NB: methysergide causes retroperitoneal, pleural, pericardial and subendocardial fibrosis, but not pulmonary fibrosis

4- Cerebral blood flow: 1. Is increased in an acutely compensated metabolic acidosis. 2. Is decreased at 72 hours in a normal person who is made to hyperventilate for 72 hours. 3. Shows a right shift in the autoreg ulation curve in patients with chronic hypertension. 4. Shows a linear increase with PaO2. 5. Is unaffected by temperature. Teaching Notes for Question 4 Theme: Cerebral blood flow The CBF is affected by PaO2, PaCO2, MAP, ICP, temp, cerebral metabolism, neurogenic factors and drugs. 1. A compensated metabolic acidosis is achieved via hyperventilation, which will decrease the PaCO2. As it is acute there will be no change in the CSF pH. 2. Studies have shown that although there is the initial cerebral vasoconstriction with the low PaCO2, this gradually normalizes within 72 hours despite the low PaCO2. 4. The CBF shows an exponential increase in the CBF at PaO2 levels of below 5.5 Kpa but is fairly constant at levels above that. 5. Every decrease in body temperature of 1 degree celcius will decrease the cerebral metabolism by 7%.

5- The following are clinical associations of Fallots tetralogy: 1. Single second heart sound. 2. Cerebral abscess. 3. Digital clubbing. 4. Right-sided aortic arch. 5. Aortic ejection click. Teaching Notes for Question 5

41

PEDIATRICS EXAMINATION REVIEW

Theme: Fallots tetralogy Most patients with tetralogy of Fallot have cya nosis from birth or beginning in the first year of life. In childhood, such patients may have sudden hypoxic spells, characterised by tachypnoea and hyperpnoea, followed by worsening cyanosis and, in some cases, loss of consciousness, seizures, cerebrovascular accidents, and even death. Such spells do not occur in adolescents or adults. Adults with tetralogy of Fallot have dyspnoea and limited tolerance of exercise. They may have complications of chronic cyanosis, including erythrocytosis, hyperviscosity, abnormalities of haemostasis, cerebral abscesses or stroke, and endocarditis. Without surgical intervention, most patients die in childhood: the rate of survival is 66 percent at 1 year of age, 40 percent at 3 years, 11 percent at 20 years, 6 percent at 30 years, and 3 percent at 40 years. Patients with tetralogy of Fallot have cyanosis and digital clubbing, the severity of which is determined by the degree of obstruction of the right ventricular outflow tract. The peripheral pulses are normal. A right ventricular lift or tap is palpable. In some patients, a systolic thrill (caused by turbulent flow across the right ventricular outflow tract) is palpable. The first heart sound is normal, but the second heart sound is single, since its pulmonary component is inaudible. An aortic ejection click (due to a dilated, overriding aorta) may be heard. A systolic ejection murmur, audible along the left sternal border, is caused by the obstruction of right ventricular outflow. The intensity and duration of the murmur are inversely related to the severity of the obstruction of right ventricular outflow; a soft, short murmur suggests that severe obstruction is present. The electrocardiogram shows right-axis deviation and right ventricular hypertrophy. On radiography, the size of the heart is normal or small, and lung markings are diminished. The heart is classically boot-shaped, with an upturned right ventricular apex and a concave main pulmonary arterial segment. A right-sided aortic arch may be present. Arterial oxygen desaturation is evident, as is compensatory erythrocytosis, the magnitude of which is proportional to the severity of the desaturation.

6- Muscle relaxation during surgery: 1. Allows improved intra-abdominal access. 2. Is not necessary in ventilated patients. 3. Is maintained using a depolarising agent such as suxamethonium. 4. Block acetyl-choline receptors on muscle end plates. 5. Side effects include histamine release. Teaching Notes for Question 6 Theme: muscle relaxants

Depolarisng agents(suxamethonium) are short acting. Non depolarising agents(atracurium,vecuronium,pancuronium) are longer lasting. Side effects include histamine release, bradycardia and rarely hyperpyrexia.

7-In hypogammaglobulinaemia, which of the followings are true? 1. In nephrotic syndrome, the loss of immunoglobulin in urine is the only cause of predisposition to infection 2. In common variable immunodeficiency, there is an increased incidence of lymphoma 3. Selective IgA deficiency is the commonest inherited form 4. In patients over 40 years of age, a chest radiograph is mandatory to exclude thymoma 5. The genetic defect in X-linked agammaglobulinaemia (XLA) is in Brutons tyrosine kinase (Btk) Teaching Notes for Question 7 Theme: Hypogammaglobulinaemia In nephrotic syndrome, the susceptibility to pneumococcal infection is multifactorial and includes decreased immunoglobulin levels, immunosuppressive treatment and in those patients with immune

42

PEDIATRICS EXAMINATION REVIEW

complex mediated glomerulonephritis -a major factor is decreased levels of complement. In XLA or Brutons agammaglobulinaemia, B-cell maturation is arrested at the pre -B-cell stage, suggesting that Btk may be required to couple the intracellular signaling events mediated by the pre -B cell receptor. Selective IgA deficiency is the commonest form of inherited Ig deficiency (Incidence about 1 in 800). Although usually asymptomatic, the incidence is higher in patients with chronic lung disease suggesting that IgA deficiency may be associ ated with increased respiratory infections. Selective IgA deficiency can also occur after phenytoin and barbiturate administration as well as post toxoplamosis, measles or other viral infections. Some patients with selective IgA deficiency may also have deficiencies in IgG2 and IgG4. The commonest form of immunoglobulin deficiency is however, transient hypogammaglobulinaemia of infancy. This presents between 6 -12 months when the maternal antibody levels wane and the infants own humoral system is not yet fully developed.

8- ACE inhibitors: 1. They cause hypotension via a drop in systemic vascular resistance 2. May lead to a side effect of dry cough, due to an increase in bradykinin levels 3. In the normal kidney, will cause an increase in renal blood flow 4. They are highly protein bound drugs 5. Indications include congestive cardiac failure in a patient with renal artery stenosis Teaching Notes for Question 8 Theme: ACE inhibitors * ACE inhibitors cause a fall in afterload or SVR, which leads to hypotension (especially on the first dose) but may increase cardiac output in those with heart failure. * They also prevent the breakdown of bradykinin, which leads to dry cough in many patients. * They are generally poorly protein bound and readily removed by hemodialysis. * Although in the normal kidney, they will increase renal blood flow and thus GFR, in a patient with renal artery stenosis, they reduce GFR and can precipitate acute renal failure and flash pulmonary edema; thus they are contraindicated in patients with renal artery stenosis. 9-Macrocephaly :1. Is defined as Head circumference > 1 SD ab ove mean 2. Is associated with high intelligence 3. Is associated with Cornelia de Lange syndrom e 4. Can be inherited 5. Is associated with neurofibromatosis Teaching Notes for Question 9 Theme: Microcephaly / Macrocephaly macrocephaly:Head circumference (HC) > 2SDs above mean microcephaly: HC > 2SDs below mean Causes of Macrocephaly : Causes of Microcephaly : Familial Familial Achondroplasia Congenital infection Canavans disease Fetal alcohol s yndrome Neurofibromatosis Cornelia de Lange syndrome Osteopetrosis Angelman syndrome Tay Sachs disease Edwards and Patau syndr ome Sotos syndrome Rubenstein Taybi syndrome.

43

PEDIATRICS EXAMINATION REVIEW

10-Regarding normal renal physiology: 1. The GFR of a child reaches adult values (corrected for weight) by the age of one year 2. The fluid entering the DCT is iso -osmolar 3. Under the influence of ADH, there is increased permeability to water in the DCT 4. In the proximal tubules, there is near -complete absorption of glucose and amino acids 5. There are approximately one million nephrons in each kidney Teaching Notes for Question 10 Theme: Normal renal physiology The GFR reaches adult levels by 3 years. GFR i s corrected for surface area, not weight. Glomerular filtration begins at 9 weeks of intrauterine life, but renal function is not necessary for normal intrauterine hemostasis. The glomerulus filters approximately 180 litres of fluid daily. The proximal tu bule isotonically reabsorbs two thirds of the glomerular filtrate, and during this process, the ultrafiltrate is modified: There is preferential absorption of sodium and bicarbonate and near complete absorption of glucose and amino acids in the proximal tu bule. Formation of nephrons is completed at birth, but functional maturation does not take place until later. 11-The following conditions occur due to the presence of circulating antibodies: 1. Graves disease. 2. Hashimoto thyroiditis. 3. Reidel's thyroiditis. 4. Myasthenia gravis. 5. Motor neurone disease. Teaching Notes for Question 11 Theme: Auto antibodies Reidel's thyroiditis is probably viral in origin. In myasthenia, there are circulating antibodies against acetylcholine receptors at the neuromuscular junction. The aetiology of motor neurone disease is not clear. 12-The following can cause cardiomyopathy in HIV disease:1. Mycobacterium tuberculosis 2. Lymphoma 3. Kaposi Sarcoma 4. CMV infection 5. HIV of the heart

44

PEDIATRICS EXAMINATION REVIEW

Teaching Notes for Question 12 Theme: Cardiomyopathy in HIV Cardiomyopathy can be caused by HIV itself, or other infections such as CMV, toxoplasma. However, although TB, Kaposi sarcoma, and lymphoma can cause an apparent cardiomegaly on a chest radiograph, this is due to pericardial effusions rather than cardiomyopathy. A pericardial effusion carries a poor prognosis in HIV. HIV cardiomyopathy is linked to CD4 count and HIV encephalopathy. 13- Gastric emptying is slowed by: 1. Supine posture 2. Gastric distention 3. Domperidone 4. Ranitidine 5. Cisapride Teaching Notes for Question 13 Theme: Gastric emptying 2,3,5 - Increases emptying. 4 - No effect 14- The following is true regarding changes in the fetal cardiovascular system after birth: 1. There is normally immediate closure of the ductus arteriosus 2. Hypoxia-induced vasoconstriction is the mechanism of closure of the ductus arteriosus 3. The ligamentum teres is the remnant of the umbilical vein 4. Regression of right ventricular hypertrophy occurs postnatally 5. Inferior vena caval pressure falls after birth Teaching Notes for Question 14 Theme: Cardiovascular changes after birth Occlusion of the umbilical cord removes this low resistance cap illary bed from the circulation; breathing results in a marked decrease in pulmonary vascular resistance , hence there is increased pulmonary blood flow returning to the left atrium raising the pressure in the left atrium causing the foramen ovale to close . As pressure in the systemic circulation rises, shunt through the ductus arteriosus reverses. As the pO2 rises, synthesis of bradykinin and prostacyclins is inhibited , thus causing closure of the ductus arteriosus . The ductus arteriosus can take up to 3 months to close in normal neonates. 15- X-linked hyper IgM syndrome :1. Causes defective B cell development 2. Is associated with normal levels of IgG 3. Can present with pneumocystis carinii infection 4. Is due to a defective CD40 ligand 5. Is due to a failure of Ig isotype switching Teaching Notes for Question 15 Theme: X-linked hyper IgM syndrome The hyper-IgM syndrome is a rare , inherited immune deficiency disorder resulting from defects in CD40 ligand/CD40 interaction. X-linked hyper-IgM (type 1) is caused by defects

45

PEDIATRICS EXAMINATION REVIEW in the CD40 ligand expressed on activated T-cells. Autosomal recessive hyper -IgM (type 2) is caused by defects in the CD40-activated RNA-editing enzyme- activation-induced cytidine deaminase. This enzyme is required for immunoglobulin isotype switching and somatic hypermutation in B -cells. In X-linked hyper-IgM, the loss of interaction between CD40 and its ligand results in impairment of T-cell, B-cell and macrophage function. The interaction of CD40L on T-cells and CD40 on B-cells is essential for B-cell activation and proliferation, isotype switching and formation of germinal centres (which are required for affinity maturation of immunoglobulin). Therefore, although there are high levels of serum IgM which can be produced without T-cell help, the T-dependent B-cell response is defective , and there is a lack of IgG, IgA or IgE production . CD40L/CD40 interaction is also necessary for optimal activation of macrophages by T -cells. The subsequent macrophage dysfunction contributes to the profound immunodeficiency seen in these patients . Patients are susceptible to infection with extracellular bac teria as well as intracellular pathogens . Boys present with recurrent sinopulmonary infections and chronic diarrhea often caused by Giardia , Campylobacter, rotavirus , or Cryptosporidium. These patients are particularly prone to opportunistic lung infection s caused by Pneumocystis carinii. There is an increased incidence of autoimmune diseases and malignancies including adenocarcinomas of the liver and biliary tract . Laboratory studies usually reveal normal B- and T-cell numbers, however neutropenia is common. IgM levels are elevated but IgG and IgA levels are low. Diagnosis is confirmed by identifying the defective gene . Treatment is supportive with IVIG and antibiotics . 16- Control of absorption: Which of the following statements are true? 1. Baroreceptors are the principal sensors for controlling sodium secretion and reabsorption. 2. In reduced sodium states, both increased activity of renal sympathetic nerves (leading to constricting afferent arterioles) and reduced glomerular capillary pressure, decrease Na+ and water excretion. 3. Changes in aldosterone levels allow control of the re -uptake of 50% of sodium originally present in the glomerular filtrate. 4. Lungs secrete angiotensinogen, kidneys secrete renin, liver cells contain angiotensin converting enzyme. 5. Renin secretion is controlled by the macula densa. Teaching Notes for Question 16 Theme: Renal system Angiotensinogen is from liver and the converting enzyme is in the lungs . Renal sympathetic nerves and intrarenal baroreceptor also control renin secretion . 17- A cataract may be induced accidentally by the following treatments: 1. Systemic steroids 2. Prolonged general anaesthesia 3. Massive blood transfusion 4. Total body irradiation 5. Multiple MRI scans Teaching Notes for Question 17 Theme: Iatrogenic cataracts A cataract is simply any loss of the transparency of the lens of the eye. Unfortunately, cataracts may form as a result of many medical treatments, including : Systemic steroid administra tion Irradiation (including multiple CT scans, theoretically) Chlorpromazine Thalidomide Busulphan Gold.

46

PEDIATRICS EXAMINATION REVIEW

18- Regarding surfactant:1. Production begins at 30 weeks 2. It is produced by Type II pneumocytes in the walls of the bronchi 3. Testosterone stimulates surfactant production 4. Production is increased during a stressful event like hypothermia 5. Betamathasone given to the mother improves surfactant production in the pre mature baby Teaching Notes for Question 18 Theme: Surfactant Surfactant production begins at 20-22 weeks. It is produced by Type II pneumocytes which are in the walls of the a lveoli. The hormones testosterone and insulin inhibit surfactant production; hence hyaline membrane disease is more common in males than females and more common in infants of diabetic mothers. Surfactant production is suppressed if the baby is hypothermic, hypoxic, acidotic or hypoglycemic. Although dexamethasone is more commonly used, betamethasone has an identical effect on lung maturation. 19- The following syndromes commonly have associated renal manifestations: 1. Zellwegers syndrome 2. Drash syndrome 3. Meckel Gruber syndrome 4. Noonan's syndrome 5. Chromosome 11p deletion Teaching Notes for Question 19 Theme: Syndromes with renal involvement Zellweger's syndrome - Also called the cerebro-hepato-renal syndrome. Often large ballotable kidneys with multiple cysts are present at birth. Most infants die in the first 6 months. Meckel -Gruber Syndrome - Occipital encephalocoele and renal cysts. Lethal. Drash Syndrome - Nephropathy, Wilm's tumour and ambiguous genitalia. Chromosome 11p deletion - Aniridia, Wilm's tumour and genitourinary malformations. 20- Bile: 1. Is secreted by the bile canaliculi 2. Contains NaHCO3 3. Is released from the gall bladder in response to the hormone gastrin 4. Contains salts that are only water soluble 5. Salts are mostly reabsorbed in the large bowel

47

PEDIATRICS EXAMINATION REVIEW Teaching Notes for Question 20 Theme: GI * Bile is produced by hepatocytes and secreted into biliary canaliculi. * Bile consists of; NaHCO3, bile salts, bile pigments, lecithin and chole sterol. It is stored in the gall bladder and gall bladder contraction is stimulated by cholecystokinin (CCK). * Bile salts are amphipathic, which means that they are both water and fat soluble. This allows them to stabilise fatty emulsions. * 80% of bile salts are reabsorbed, and this is done in the terminal ileum. 21- The following are true regarding vaccine administration in children: 1. Live vaccines are contraindicted within 3 weeks of another live vaccine administration 2. MMR can be given a week after administration of IV immunoglobulins 3. Live vaccines are not given to children on prolonged steroid therapy 4. Severe febrile illness is a contraindication for DPT 5. Severe malnutrition is a contraindication for BCG Teaching Notes for Question 21 Theme: Immunisation Live vaccines are contraindicated 3 weeks before and 6 weeks after administration of Immunoglobulins. Other contraindications to live vaccines include steroid administration / immunosuppressive therapy / severe egg allergy/ immunodeficient conditions.Contraindications to pertussis vaccination include hypersensitivity to the vaccine and a seve re reaction such as shock, persistent screaming, fever over 40.5 degrees C., or serious neurological symptoms. Malnutrition, Stable neurological conditions, minor illness , chronic diseases of heart, lung and kidney, antibiotic treatment are not contraindi cations to vaccines. 22- Wide splitting of second heart sound may be evident in: 1. Normal children 2. Volume overload due to partial anomalous venous drainage (PAPVD) 3. Severe aortic defect 4. Atrial septal defect 5. Pulmonary hypertension Teaching Notes for Question 22 Theme: Widely split 2nd heart sound Increased normal splitting (wider on inspiration ) of the 2nd heart sound occurs when there is: * any delay in RV emptying as in RBBB (delayed RV depolarization ), * pulmonary stenosis (delayed RV ejection), * VSD (increased RV volume overload), and * mitral incompetence (earlier aortic valve closure because of more rapid LV emptying). Fixed splitting (no respiratory variation) is caused by an ASD where equalization of volume loads between the two atria occurs through the defect . In the case of reverse splitting , P2 occurs first and splitting occurs in expiration. This can be due to: * delayed LV depolarization (LBBB), * delayed LV emptying (severe AS, coarctation of the aorta ) or * increased LV volume load (large PDA- though the machinery murmur tends to obscure S2). Splitting of the first sound is not usually detectable clinically; however, when it occurs, it is most often due to complete RBBB .

48

PEDIATRICS EXAMINATION REVIEW

23- Regarding hyperkalaemia :1. Tall P waves are seen on ECG 2. Continuous salbutamol nebulisers should be started immediately on diagnosis 3. IV calcium resonium should be commenced 4. Calcium gluconate exchanges for potassium and enables its excretion 5. If acidosis is corrected, potassium shifts back into the intracellular f luid Teaching Notes for Question 23 Theme: Hyperkalaemia ECG changes of hyperkalaemia are 1. Prolonged PR interval; 2. Peaked T waves; 3. Widening of the QRS complex; 4. ST depression; 5. Ventricular fibrillations; 6. ventricular tachycardia; Emergency treatment of hyperkalaemia 1. Continuous salbutamol nebulisers or IV salbutamol; 2. IV sodium bicarbonate; 3. IV calcium gluconate; 4. Glucose-insulin infusion; 5. Ion exchange resin; 6. Peritoneal or hemodialysis. Note- Calcium gluconate stabilizes the myocardium by antagonising the effect of potassium on the heart. It does not lower serum K+ levels. 24- T lymphocytes:1. Are derived from a precursor cell in the bone marrow 2. 95% die in the thymus 3. All have the alpha-beta heterodimeric T cell receptor 4. Are usually activated by intact protein antigens 5. Are prominent in type II hypersensitivity reactions Teaching Notes for Question 24 Theme: T lymphocytes T lymphocytes comprise 70-80% of the lymphocyte population and are haematopoietic cells. They are produced in bone marrow and paracortical regions of lymph nodes and spleen. Maturation occurs in the thymus , and involves a complicated series of positive and negative selection steps coordinated by the thymocytes and dendritic cells within the thymus, resulting in apoptosis of unwanted cells. The TCR is a heterodimer of two chains (alpha and beta, or gamma and delta) linked by disulphide bonds and is found at the cell surface associated with a complex of other polypeptides known collectively as CD 3. The two types of TCR are called TCR-1 (gamma-delta) and TCR-2 (alpha-beta). TCR-1 cells are thought to have a restricted repertoire and to be mainly non -MHC restricted. TCR-2 cells recognise peptide fragments associated with MHC molecules .

49

PEDIATRICS EXAMINATION REVIEW

25- Post-traumatic stress disorder :1. Is an immediate response to a traumatic event 2. Often involves flashbacks 3. May lead to avoidance 4. Can sometimes be treated by play therapy 5. Is associated with feelings of detachment
Teaching Notes for Question 25 Theme: Post-traumatic stress disorder Post traumatic stress disorder is characterised by a delayed or protracted response t o a stressful situation or event. Symptoms include reliving of the trauma or event in flashbacks or dreams, together with feelings of detachment or emotional numbness or blunting. Avoidance of people or situations associated with trauma. Lack of energy, panic attacks, anxiety, depression. Treatment: intensive individual therapy, sometimes using play, puppetry, storytelling, or dramatic reenactment. Family therapy sometimes useful. Medication also occasionally used, depending on symptoms encountered.

26- Refsums Disease: 1. Is inherited as an autosomal dominant disorder 2. Is associated with icthyosis 3. Is caused by the same enzyme as Hurlers disease 4. Is one of the causes of retinitis pigmentosa 5. Is affected adversely by radical weight change
Teaching Notes for Question 26 Theme: Refsums Disease Refsum's disease is an autosomal recessive disorder caused by defective peroxisomal oxidation of phytanic acid. The mutation is in the phytanoyl -CoA hydroxylase (PAHX) gene on chromosome 10p13. It is totally unrelated to Hurlers sy ndrome. Toxic accumulation of phytanic acid causes adolescent onset progressive retinitis pigmentosa, cataracts, anosmia, deafness, cerebellar ataxia, and sensory and hypertrophic interstitial neuropathy with preservation of intellect. The disorder is asso ciated with icthyosis (a desquamative skin disorder) and cardiac arrhythmias. By limiting intake of meat (animal fat) and green vegetables, plasma phytanic acid levels fall with an improvement in the neurological signs. This occurs because the mutations are not absolute in eliminating function and because omega-oxidation can also reduce phytanic acid levels. Cerebellar dysfunction also manifests more prominently with rapid changes in the patient's weight (particularly rapid weight loss) and hence it is reco mmended that a stable body weight is mantained. Plasmapheresis has been used if the patient's clinical condition rapidly worsens. Other causes of retinitis pigmentosa are:Freidrichs Ataxia, Kearns Sayre syndrome, Ushers syndrome, abetalipoproteinaemia , Alports syndrome and Laurence-Moon-Biedl sydrome.

50

PEDIATRICS EXAMINATION REVIEW

27- The following are true of adult polycystic kidney disease: 1. It can present in the newborn. 2. It may be associated with aortic stenosis. 3. The genetic mutation of type I has been linked to chromosome 16. 4. Liver cysts are seen in approximately 10%. 5. The incidence is about 1:10,000. Teaching Notes for Question 27 Theme: Adult polycystic kidney disease APKD is an autosomal dominant condition , affecting between 1:1,000 and 1:2,000 of the population. It can present at any time from early infancy. The genetic mutation of type 1 is on the short arm of chromosome 16 (PKD1 gene), and PKD2 has been located to chromosome 4. Clinical presentations include hypertension , haematuria, UTIs, abdo mass or pain and polycythaemia, as well as renal impairment. Over 50% have liver cysts , about 10% have pancreatic cysts and about 5% have Berry aneurysms . Other associations include mitral valve prolapse , aortic incompetence , diverticulitis and abdominal herniae . 28-The following statements are true regarding Hypertrophic obstructive cardiomyopathy:1. The prognosis is related to the degree of left ventricular outflow gradient 2. Prognosis is worse in children 3. Runs of Non sustained ventricular tachycardia on Holter have a high predictive value for sudden cardiac death 4. Progresses to dilated cardiomyopathy in 10% cas es 5. Strenuous activity is proscribed in all patients Teaching Notes for Question 28 Theme: Hypertrophic obstructive cardiomyopathy In hypertrophic cardiomyopathy , the degree of outflow obstruction and the risk of sudden death depend on the gene defect . There is no correlation betreen these two features however. Presentation in childhood signifie s worse prognosis. Runs of non-sustained VT are found in one-fourth of patients of HCM patients , although sustained VT is uncommon . In some, it is a harbinger of sudden cardiac death ; however, its overall predictive value in identifying patients at high ri sk of sudden death is quite limited . Treatment with beta-blockers and amiodarone may be of benefit as may implantation of a defibrillator . 29- Disseminated intravascular coagulation: 1. Is usually associated with a high fibrinogen level 2. Is usually associated with a normal thrombin time

51

PEDIATRICS EXAMINATION REVIEW

3. May be associated with cytomegalovirus or varicella zoster infections 4. May occur in liver disease 5. May be associated with formation of microthrom bi, so should be treated with intravenous heparin
Teaching Notes for Question 29 Theme: Disseminated intravascular coagulation * Disseminated intravascular coagulation (DIC) m ay be caused by bacterial infections, in particular gram-negative and meningococcal septicemia, by severe falciparum malaria infections and by viral infections including CMV, HIV and varicella zoster. * Liver disease can also cause DIC due to release of t hromboplastins from damaged liver cells and reduced concentrations of antithrombin and protein C. * Investigations show a prolonged thrombin time, prothrombin time and activated partial thromboplastin time (aptt), thrombocytopenia, a low fibrinogen and ra ised D-dimers or FDPs. * DIC may be associated with microthrombin causing skin lesions, renal failure, gangrene or cerebral ischemia but heparin therapy is usually not indicated as it can aggravate the bleeding tendency.

30-Cardiac tamponade: 1. Occurs only after penetrating injuries to the heart. 2. Is characterised by arterial hypertension. 3. The central venous pressure is raised. 4. Should be treated with aspiration using a needle inserted via the second intercostal space, mid-clavicular line. 5. Is characterised by increase ECG voltage across all leads. Teaching Notes for Question 30 Theme: Cardiac tamponade. Cardiac tamponade can occur after both penetrating and blunt injuries to the chest, and is characterised by hypotension , Kussmauls sign, raised JVP, muffled heart sounds and decreased voltages in all ECG leads . 31-Regarding glycogen and its metabo lism: 1. Glycogenolysis in the liver releases glucose-6-phosphate into the blood stream 2. Branches in the glycogen molecule are produced by 1,4 -glycosidic linkages 3. Adrenaline is an important stimulus for glycogenolysis in the liver during exercise 4. Deficiency of muscle phosphorylase, may result in an impaired exercise tolerance 5. McArdles Disease results in fasting hypoglycaemia and hepatomegaly Teaching Notes for Question 31 Theme: Glysogen metabolism
Glycogen, the storage form of carbohydrate, is found predominantly in liver and muscle as cytoplasmic granules. Glycogen is a large branched polymer of glu cose residues. Chains are formed by Alpha 1,4-glycosidic bonds, each residue being added from an activated donor molecule, UDP-glucose. Branches, at every 10 residues or so, are formed by 1,6 glycosidic bonds formed by a separate enzyme. Removal of glucose (glycogenolysis)

52

PEDIATRICS EXAMINATION REVIEW occurs by cleavage of alpha -1,4-glycosidic bonds by phosphorylase to produce glucose - 1phosphate and by cleavage of a -1,6glycosidic bonds by a separate debranching enzyme. Glucose-1-phosphate is then converted to glucose -6phosphate. In the liver, a glucose-6phosphatase allows passage of free glucose into the bloodstream. In muscle, there is no phosphatase trapping glucose -6-phosphate within the cell. Conditions that stimulate glycogenolysis inhibit glycogen synthesis, and vice versa. The rate of glycogenolysis is hormonally regulated by glycogenolysis (liver, starvation) and adrenaline (muscle, exercise). Glycogen storage diseases: type I (von Gierke's disease) results from glucose -6phosphatase deficiency leading to failure of release of glucose from the liver. Massive hepatomegaly and fasting hypoglycaemia develop; type II (McArdles disease) caused by a failure of muscle phosphorylase leading to normal blood glucose but impaired exercise tolerance.

32-Concerning Hepatitis C Virus: 1. 1% of the worlds population is infected 2. The proportion developing chronic infection and hepatocellular carcinoma are similar to HBV infection 3. Neonatal infection is more likely if the mother has concomitant HIV infection as well as HCV 4. Breast feeding should be discouraged in the HCV +ve mother 5. Liver transplantation is usually required due to the inadequacy of medical therapies Teaching Notes for Question 32 Theme: Hepatitis C Hepatitis C infects an estimated 500 million people. Unlike hepatitis B infection is not concentrated in developing countries and 0.7% of the UK population is affected. It is spread by blood contact especially via blood transfusion and IV drug abuse. Sexual and vertical transmission occurs but is much less common. Co infectin with HIV results in more rapid and severe progression to chronic liver disease. Hepatitis C can be treated with interferon alpha, sometimes in combination with ribavirin but response depends on viral genotype, age. alcohol intake and degree of liver fibrosis and inflammation seen on biopsy.
33-Which of the following conditions would be suitable for somatic g ene therapy:-

1. Adenosine deaminase deficiency 2. Thalassaemia 3. Diabetes mellitus 4. Haemoplilia A 5. Sickle cell disease Teaching Notes for Question 33 Theme: Somatic gene therapy

Somatic gene therapy (i.e. addition of a gene to the somatic cells or tissues rather than the germ line) is ideally suited for monogenic disorders caused by loss of gene function where even partial restoration of the protein will greatly ameliorate the symptoms, as this form of gene therapy is unlikely to restore the levels of the protein to normal. Adenosine deaminase deficiency, haemoglobinopathies in particular thalassaemia maj or, and haemophilia fit these criteria. Patients with diabetes mellitus require controlled release of insulin and gene therapy is unlikely to provide the answer. Islet cell transplantation into the portal vein is more promising as it preserves the microenv ironment of the beta cells of the islet and allows controlled temporal release of insulin. 34-In paroxysmal nocturnal haemoglobinuria:-

53

PEDIATRICS EXAMINATION REVIEW

1. The defect is a congenital abnormality of RBC production by the bone marro w 2. Hamoglobinuria is a characteristic finding 3. Venous thrombosis is a recognised complication 4. The diagnostic test is Hams test 5. Transforms into acute leukaemia in approx. 5% Teaching Notes for Question 34 Theme: Paroxysmal nocturnal haemoglobinuria

PNH is an acquired clonal defect of cell membranes that makes the cells more sensitive to lysis by comple ment. This results in intravascular haemolysis and haemoglobinuria. Crises of haemolysis can be triggered by infection, and chronic haemolysis may result in nephropathy. There is an increased risk of venous thromboses (esp. in unusual sites such as saggita l sinus or hepatic veins).The diagnostic test is Hams test (acidified serum lysis). The disease may transform into acute leukaemia (in approx 5%).

35-Phosphodiesterase inhibitors: 1. Have an anti-diuretic effect 2. Cause bronchodilation 3. Cause peripheral dilatation 4. Cause relaxation of the diaphragm 5. Decreases the pulmonary capillary wedge pressure Teaching Notes for Question 35 Theme: Inotropes and vasopressors 1. They cause a weak diuretic effect. Inhibition of tubular Na reabsorption leads to a naturesis and may precipitate hypokalemia. 2,3. They are ino -dilators. 4. They are bronchodilators, incre ase the sensitivity to CO2 and increase diaphragmatic contractility. 5. Both the LVEDP and PCWP decreases. 36-Regarding mumps infection: 1. The incubation period is 7-14 days 2. It is often asymptomatic 3. Children are not infectious after the development of parotitis 4. Infertility is a frequent complication of the orchitis associated with mumps 5. It is caused by a DNA virus Teaching Notes for Question 36 Theme: Mumps Mumps is caused by an RNA virus; incubation period is 14 -21 days. In 30-40% of cases the infection is asymptomatic. Children are infectious for a few days before until a week after the parotiti s. Infertility is not common after orchitis. The other important complications are CNS infection (aseptic meningitis) and pancreatitis.

54

PEDIATRICS EXAMINATION REVIEW

37-The following are known functions of T helper cells : 1. Production of T cell growth factors such as IL-2 2. Promotion of immunoglobulin isotype class switching 3. Production of perforin 4. Production of suppressive cytokines such as IL -10, TGF-beta 5. Induction of the maturation of immature dendritic cells
Teaching Notes for Question 37 Theme: Functions of T helper cells Perforin is produced by cytotoxic T lymphocytes (CTL) to kill target cells. Helper T lymphocyt es (HTL) are usually CD4+ T-cells. HTL can be sub-classified into TH1 and TH2 cells. More recently, further sub -populations of CD4+ T cells have been identified, termed Th3 and Tr1 cells. In general, Th1 cells secrete pro -inflammatory cytokines such as IL-2, IFN-g, TNF-alpha , TNF-beta and are important for cell mediated immunity. They also promote the production of opsonising antibodies. Th2 cells secrete IL -4, IL-5 and IL-10 and are important for humoral responses. Th3 and Tr1 are not yet well defined but the former secrete predominantly IL-10 while the latter secrete mainly TGF-beta It must be remembered that this differential cytokine expression is not absolute.

38-Regarding immunisation in the UK: 1. The first dose of vaccines is given at a correct ed age of 2 months in case of premature delivery 2. Family history of febrile convulsion following immunisation is a relative contraindication 3. Measles, mumps, rubella and polio are contraindicated in HIV +ve children 4. Live vaccines should be avoided within 6 months o f termination of lymphoma treatment 5. Prolonged inconsolable crying to previous immunisation is a definite contraindication. Teaching Notes for Question 38 Theme: Immunisations in childhood Prematurity, asthma, chronic lung disease, congenital heart disease, and Downs syndrome are associated with an increased risk of complications from infectious diseases and they should be immunised as a matter of priority. Premature babies should be immunised according to the recommended schedule from 2 months after birth, irrespective of prematurely. Studies have shown that antibody response and adverse events are not significantly different in preterm and term infants immunised 2,3, and 4 months afterbirth. Prolonged inconsolable crying or high pitched screaming more than 4 hours is classified as a severe reaction to immunisation and is a contraindication to sub sequent immunisation with same vaccine. Patients on active chemo or radiotherapy or within 6 months of termination of such treatment are contraindicated for live vaccines. They may not be able to produce a normal immune response to live vaccines and could potentially suffer from severe manifestation such as disseminated infection with BCG or paralytic polio from vaccine virus. No harmful effects have been reported with MMR and polio in HIV +ve individuals. However it should be noted that Polio is secreted for longer periods in HIV positive patients as compared to other people. Family history of febrile convulsion is not a contraindication to immunisation. 39-Regarding paediatric liver transplantation: 1. The most common indication is failed Kasai procedu re

55

PEDIATRICS EXAMINATION REVIEW

2. 5-yr survival rates for children after orthotopic liver transplantation are around 30% 3. FK506 has superceded tacrolimus as an immunosuppressant 4. Signs of chronic liver disease persist despite improvement in liver function 5. Organ rejection is an early post operative complication Teaching Notes for Question 39 Theme: Liver transplantation Paediatic liver transplantation is an uncommon procedure (about 500/year in US) but the most common reason is following a failed portoenterostomy (Kasai) procedure following extrahepatic biliary atresia. The next most common indications are metabolic liver disease and acu te hepatic necrosis. Orthotopic (same species) transplants are surprisingly successful especially with improved immunosuppression and better preservation of the organ (up to 18 hr ex vivo with <2% primary nonfunction). Most recent figures are 81% at 1 year and 77% at 5 years. As liver function improves, so does the childs growth stigmas of chronic liver disease resolve. Organ rejection tends to occur after the early phase along with infection. Early complications include fluid shifts, electrolyte imbalance , renal dysfunction, and hypertension. Vascular complications, such as thrombosis of graft vessels, also may be an ominous early problem. Tacrolimus and FK506 are the same drug. Steroids and either cyclosporine or tacrolimus are standard therapy to prevent rejection. Compared with cyclosporin, tacrolimus is associated with lower rates of acute and chronic rejection and reduced use of corticosteroids. 40-The following statements are correct regarding Horners syndrome: 1. It may be caused by injury to the superior cervical sympathetic ganglion. 2. Ptosis occurs because of loss of innervation to the smooth muscle portion of the levator palpebrae superioris. 3. Pupillary dilatation occurs because of loss of innervation to the iris sphincter muscle. 4. May be associated with ophthalmoplegia in the territory of the IIIrd cranial nerve. 5. May be associated with loss of sweating on the ipsilateral arm. Teaching Notes for Question 40 Theme: Horners syndrome. Horner's syndrome may be caused by any injury to the sympathetic chain in the neck. It is associated with ipsilateral ptosis (paralysis of levator palpebrae superioris), meiosis (paralysis of dilator pupillae muscle) and facial and upper limb anhidrosis. The somatic component of the IIIrd nerve is unaffected and thus ophthalmoplegia is not a feature

41-In HIE or Jobs syndrome : 1. There is a markedly raised IgE level 2. There is an association with coarse facies, kyphoscoliosis, osteoporosis, and eczema 3. There is a low incidence of atopy due to defective IgE function 4. The condition is X-linked 5. Cold abscesses occur

56

PEDIATRICS EXAMINATION REVIEW

Teaching Notes for Question 41 Theme: HIE or Jobs syndrome HIE (HyperIgE syndrome) or Jobs syndrome is an autosomal dominant condition characterised by coarse facies, kyphoscoliosis, osteoporosis, atopy and eczema. Patients develop recurrent sinopulmonary and cutaneous infections that tend to be much less inflamed than appropriate for the degree of infection (cold abscesses). The IgE level is mark edly raised in these patients 42-The following increases the pulmonary vascular resistance 1. Maximal inspiration 2. Maximal expiration 3. Anemia 4. Salbutamol 5. Prostacyclin Teaching Notes for Question 42 Theme: Pulmonary circulation The PVR is increased by * Hypoxia, hypercarbia, acidosis. * 5HT, histamine, vasoconstrictors. * Alpha-adrenergic drugs. The PVR is decreased by * Vasodilators, Ach. * Beta-adrenergic drugs. * Parasym nervous sys * Prostacyclin Effect of respiration: PVR varies with lung volume in a U -shaped curve. The lowest PVR is at the FRC
43-Chronic lymphocytic leukaemia : 1. Is associated with smear or smudge cells in the peripheral blood 2. Is asymptomatic at diagnosis in less than 10% of patients 3. Should be treated aggressively in the early stages of the disease 4. Has a median survival of less than 5 years 5. Is associated with splenomegaly at diagnosis in about 50% of patients Teaching Notes for Question 43 Theme: Chronic lymph atic leukaemia Chronic lymphatic leukaemia (CLL) is due to an accumulation of mature lymphocytes in the peripheral blood, bone marrow, spleen, liver and lymph nodes. Smear or smudge cells are often seen in the peripheral blood which represent the fragile m alignant cells which have been squashed as the blood film is spread. It is asymptomatic at diagnosis in over 30% of patients, and splenomegaly is seen in 50% at diagnosis. It is often a slowly progressive disease and should not be treated in the early stages. Median survival is 3-5 years, but one third of patients die of causes other than their CLL. 44-The following agents have good activity against Bacteroides fragilis 1. Benzylpenicillin

57

PEDIATRICS EXAMINATION REVIEW

2. Piperacillin / tazobactam 3. Cefoxitin 4. Metronidazole 5. Chloramphenicol Teaching Notes for Question 44 Theme: Anaerobic Infections B fragilis produces a -lactamase which is sensitive to inhibitors. It also produces a cephalosporinase, but it does not hydrolyse cefoxitin (cephalosporin with anaerobic activity). 45-Digestion and absorption: Which of the following statements are true? 1. The glucose-containing polysaccharides starch, cellulose and glycogen are broken down for absorption by salivary and pancreatic amylases. 2. Trypsin, chymotrypsin, carboxypeptidases and amino peptidases are responsible for the breakdown of proteins for absorption. 3. Amphipathic phospholipids and bile salts convert micelles into lipid droplets ready for absorption. 4. The three known fat soluble vitamins A, D and E are taken up by fat absorbing processes in the gut. 5. About 9 litres of water enters the gut each day. Teaching Notes for Question 45 Theme: Gastrointestinal system Amylases attack starch, cellulose is indigestible dietary fibre. Micelles are smaller structures made from lipid droplets by these agents. Vitamin K is also fat soluble and is also taken up by fat absorbing processes in the gut. 46-In asthma:1. The eosinophils to the lung are recruited from the bone marrow 2. Steroids are contra-indicated if the patient is a diabetic 3. Blocking interleukin-5 maybe therapeutic 4. Leukotriene inhibitors are associated with the development of Churg -Strauss . 5. Bronchial hyper-responsiveness (BHR) does not have to be present. Teaching Notes for Question 46 Theme: Asthma, pathophysiology In asthma, by definition bronchial hyper -responsiveness must be present. IL-5 is produced by T helper cells (TH-2) and recruits eosinophils from the marow. Blocking IL -5 will therefore be therapeutic. Leukotriene inhibitors are associated with Churg -Strauss syndrome and causality can not be excluded. 47-Regarding overdose, haemodialysis would be effective in removing drug in: 1. Digoxin 2. Pethidine 3. Amitriptyline 4. Propranolol

58

PEDIATRICS EXAMINATION REVIEW

5. Salicylate Teaching Notes for Question 47 Theme: Haemodialysis - drugs The volume of distribution (VD) of the drug determines how effective dialysis will be in removing it from the body. Drugs for which haemodialysis is effective include salicylates, phenobarbitone, methanol, ethylene glycol and lithium. Digoxin, opiates, tricyclics and lipid soluble beta-blockers have a very large VD. More effective supportive measures are antidigoxin antibody/atropine/correction of hypokalaemia for digo xin; naloxone for pethidine; betablockers for arrhythmias from tricyclics, pacing/glucagon/isoprenaline for propranolol. Haemoperfusion is effective for short and medium acting barbiturates, chloral hydrate, meprobamate, and theophylline. Activated charcoa l should be given for aspirin, carbamazepine, dapsone, phenobarbitone, quinine, and theophylline. 48-Heat inputs and outputs: Which of the following statements are true? 1. Changes in muscle activity are the main source of heat production for temperatu re control. 2. The skin is a good thermal insulator whose effectiveness is controlled through blood flow. 3. The blood flow through skin is controlled through vasodilator sympathetic nerves whose firing rate is increased in response to heat. 4. Sweating can increase body evaporative water loss from 0.6 to over 4.5 litres per day and eliminate 2.4 Mcal heat from the body. 5. Local humidity and air movement do not significantly influence heat loss. Teaching Notes for Question 48 Theme: Thermoregulation Skin blood flow is largely controlled by sympathetic vasoconstrictor nerves, which increase their firing rate in re sponse to cold and decrease with heat. Humidity controls evaporation and therefore the effectiveness of sweating. All convective losses increase with air movement. 49-In gene regulation:1. Chromatin plays no role. 2. Transcription factors control expression by binding to specific DNA sequences. 3. Transcription levels determine protein levels . 4. DNase I hypersensitivity sites are usually indicative of regions of active chromatin. 5. The stability of mRNA is entirely dependent on its length. Teaching Notes for Question 49 Theme: Gene regulation The association of double-stranded DNA with histones (and other non-histone) proteins forms chromatin. Nucleosomes consist of eight proteins - two copies each of H2A, H2B, H3 and H4 around which the DNA coils. A fifth protein, histone H1, helps pack the nucleosomes together to increase compaction. These nucleosome struc tures together with DNA form a repeat unit of approximately 200 nucleotide pairs and are very regularly arranged along the chromosome. Elaborate folding and protein histone interactions organise the chromatin fibre (made up of the nucleosomal DNA) into a h igher order structure with varying degrees of compaction. Condensation of chromatin varies at different stages of the cell cycle and with transcriptional activity. Regular nucleosomal spacing can be interrupted by transcription factors and other DNA bindin g proteins resulting in the creation of a nuclease hypersensitive site. DNase I hypersensitivity is a method used to analyse the state of chromatin at a particular site on a chromosome. DNA regions that are assembled into regularly spaced nucleosomes are m ore resistant to cleavage by this enzyme than are regions that lack nucleosomes due to the binding of proteins to the DNA. DNAse hypersensitive sites are often associated with gene control regions such as promoters and enhancers as the activating transcrip tion factors bound to

59

PEDIATRICS EXAMINATION REVIEW these regions displace nucleosomes or alter their structure. Thus DNase hypersensitive sites are often associated with active chromatin. However it is important to remember that this is not always the case. Some proteins that bind to DNA act as silencers, and in some cases transcription factors may still bind DNA when the gene is not expressed. The higher order structure of chromatin plays a major role in gene regulation. Chromatin must be open for transcription to take place to enabl e the various transcription factors and the components of RNA polymerase to gain access. This is achieved by various protein complexes which use energy to displace histones or which modify histone tails - e.g. acetylation of histone tails opens the DNA stru cture and aids in transcription factor binding. Some regions of DNA, the heterochromatic regions, consist of very compacted nucleosomes. These areas are refractory to gene expression, and are usually made up of DNA containing multiple repeat sequences. Euc hromatin refers to chromatin that is decondensed in interphase and is rich in actively transcribed genes. Transcription factors are proteins that bind to specific DNA target sequences and then act to enhance transcription, usually by stabilising the comp lex containing the RNA polymerase. They may not bind directly to the polymerase, but can interact via associated factors. They alter chromatin structure by displacing the nucleosome histones and 'opening up' the DNA, making it accessible to digestion by DN ase 1. They are known as trans-acting factors, because the protein is usually encoded on a different chromosome from the ones on which they operate. Trans means 'from elsewhere', while cis refers to the sequences that are immediately upstream of the gene i.e. are physically linked to the gene. A major factor in dictating the level of messenger RNA (mRNA) in the cell is obviously transcription. However, the half life of mRNA, together with the presence of alternatively spliced variants can affect the steady state levels of mRNA considerably. Secondly, once the mRNA has translocated to the cytoplasm and is translated, efficiency of translation will be another factor determining the level of protein produced. Finally, proteins can be stored in the cytoplasm for release; for instance, to the cell surface after an additional signal. In this case, expression on the cell surface would not be related to transcription at all, but to this secondary release. The role played by length of mRNA is unknown but it is proba bly not a major factor determining stability of the transcribed sequence. Specific sequences in the message seem to be involved in stabilisation. The best example is that of the AU rich element (ARE). The consensus sequence in the ARE is the AUUUA repeat e lement. This sequence is found in the 3' noncoding region of many eukaryotic mRNAs particularly those of transiently expressed proteins such as cytokines and growth factors. The ARE triggers destabilization of mRNA when bound by certain proteins. AREs have also been shown to play a role in the stabilization of mRNA - MK2 (a protein kinase induced by LPS) may inhibit the binding or activity of destabilising proteins associated with the AREs of proinflammatory cytokine mRNAs. The complex mechanisms of ARE -mediated post-transcriptional regulation are still being revealed.

50-Sarah is eight years old and she was referred by her G.P with a velvety, pigmented rash on her neck and axilla. It is not itchy and she is not disturbed by it. She is the only child of her parents. Her weight is 49 kg and her G.P was concerned about this and has addressed this issue with her and her parents. Which one of the following investigations is the most relevant for her future management? 1. Fasting serum cholesterol/triglycer ide 2. Fasting serum insulin/glucose ratio 3. Serum adrenocorticotrophic hormone (ACTH)/cortiso l 4. Plasma leptin 5. Serum thyroid-stimulating hormone (TSH) Teaching Notes for Question 50 Theme: Diabete mellitus Acanthosis nigrans is associated with insulin resistance or poor control and is indicated by a serum insulin/glucose ratio. 51-Congenital CMV infection: -

60

PEDIATRICS EXAMINATION REVIEW

1. Only 10% of affected pregnancies have resulting long term sequele at birth 2. Diagnosis is by viral isolation from the urine 3. Hearing loss can deveolp gradually over the first 5 years 4. The affected newborn should be treated with ganciclovir 5. Intracranial calcifications are seen in a periventricular distribution

Teaching Notes for Question 51 Theme: Congenital CMV. Congenital CMV occurs in approximately 1% of all live births and only 10% of these infections result in clinical symptoms. Severe clinical disease is associated with primary maternal infection in pregnancy. Infection in early gestation carrie s a far greater risk of severe fetal disease. In CMV intra-cranial calcifications are in a periventricular distribution. Ganciclovir is only used if there is CNS involvement, chorioretinitis or pneumonitis. 52-In systemic lupus erythematosis (SLE) 1. Exacerbations may be associated with lymphadenopathy 2. There is an increased liability to infections 3. Intravenous immunoglobulins are commonly used in severe disease 4. Raynauds phenomenon is the commonest presenting symptom 5. Is associated with an increased risk of ischaemic heart disease Teaching Notes for Question 52 Theme: SLE Kikuchis syndrome (histiocytic necrotising lymphadenitis) occurs in young females and presents with pyrexia and cervical lymphadenopathy and is associated with SLE; lymphadenopathy per se is commonly observed in flares. Cyclophosphamide and high dose steroid is the first line in life-threatening flares. Arthralgia and tiredness are almost universal. As with several chronic inflammatory diseases, there is an increased risk of IHD. 53-In scleroderma (systemic sclerosis):1. Skin tightness may improve with time 2. Females are affected three times more commonly than men 3. Calcinosis only occurs in CREST form of the disease 4. Anti-Scl 70 antibodies occur in 50-60% of cases 5. Penicillamine may delay the development of i nternal organ complications Teaching Notes for Question 53 Theme: Scleroderma Most patients' skin softens after 3 -10 years of disease, although progressive digital contractures may occur. Internal organ involvement is usually progressive. Calcinosis (deposits of BCP) is seen in about 50% of patients with limited SS (CREST) and in 5% of those with diffuse SS. The most commonly affected sites are the PIPs, elbows, fingertips and knees (but can occur anywhere). The female to male ratio is 3:1 (age 35 -65). Little race effect. Rare in children. Anti-topoisomerase (Scl-70) antibodies are only seen in 20-30% of patients with diffuse disease and in 10-15% of patients with CREST. Anti-centromere antibodies are seen in up to 90% of patients with the latter. The ANA is positive in about 95% of patients with either form. There is limited evidence that penicillamine delays the progression of cutaneous

61

PEDIATRICS EXAMINATION REVIEW disease and interstitial lung disease. Un like most connective tissue disorders, corticosteroids should probably be avoided in scleroderma as they may precipitate a renal crisis. 54-Regarding multicystic dysplastic kidney disease: 1. It is a rare anomaly that presents as an abdominal mass in th e neonate 2. It is treated by routine nephrectomy 3. UnilateraI lesions have an incidence of 1:2 500-1:4000 live births 4. There is usually 0% differential function on DMSA/ MAG3 imaging 5. Hypertension is a common complication of MCDK that warrants nephrectomy Teaching Notes for Question 54 Theme: Multicystic Dysplastic kidney disease Multicystic dysplastic kidney (MCDK) is a congenital disorder in which cysts of multiple sizes replace the normal renal cortex. MCDK is usually a unilateral disorder. Bilateral disease, which has an incidence of 1:20,000, is incompatible with life. Prior to the introduction of USS e xamination, CDK was thought to be an uncommon disorder that presented as an abdominal mass during childhood, and nephrectomy was routinely performed. However, it is now apparent that the condition is commoner than was previously supposed and unilateral dis ease often follows a clinically undetectable course. Evidence suggests that prophylactic nephrectomy in this condition is not indicated as complications are rare. If hypertension does develop -which is rare - then nephrectomy is warranted. 55-Regarding schistosomiasis: 1. Freshwater snails are the definitive host 2. Cercariae penetrate the skin and may lead to an itchy papular rash 3. Fever, urticaria and eosinophilia may be seen in the initial illness due to antigen -antibody complex formation 4. Schistosoma haem atobium infection may lead to rectal bleeding 5. Schistosoma haematobium infection may lead to an obstructive uropathy Teaching Notes for Question 55 Theme: Schistosomiasis Schistosomiasis is a trematode (fluke) infection. Intermediate host - snails Definitive host - humans S. Haematobium Urinary schistosomiasis (Africa, Middle East) S. Mansoni (Africa, SW Asia, Americas) S. Japonicum (China, Japan - more severe) S. Intercalatum (West/Central Africa) S. Mekongi (Mekong Delta) LIFE CYCLE: Eggs are shed in urine or stool into fresh water. Miracidia hatch and infect freshwater snails (intermediate hosts). 4 -8 weeks later cercariae emerge which penetrate skin of Worms reside in vesical plexus Eggs shed in urine Intestinal/liver schistosomiasis Worms reside in mesenteric veins Eggs shed in stool

62

PEDIATRICS EXAMINATION REVIEW humans and migrate via the lungs to t he liver as a schistosomule. Adult worms develop in 1-3 months in the portal veins and migrate to their final habitats (see above) where eggs are laid. PATHOLOGY is caused by: 1. Invasion of cercariae: * Swimmer's itch (papular dermatitis) 2. Katayama fever: * Acute infection * Fever, urticaria, eosinophilia, cough/wheeze, hepatosplenomegaly, diarrhoea * Can treat with steroids 3. Eggs cause most of the pathology: a) Eggs leaving gut/urinary tract cause bleeding b) Eggs remaining cause granulomata and calcify: * Urinary tract - obstructive uropathy, calcified bladder, * squamous cell carcinoma bladder, pseudopapillomas * GI tract - colonic polyposis leading to iron deficiency anaemia c) Eggs in the portal tract cause periportal fibrosis (Symmer s fibrosis) leading to portal hypertension (but normal liver function) d) Eggs can embolise to lungs (pulmonary hypertension, corpulmonale) or CNS (space occupying lesion, cord compression) 4. Immune complex disease: * Glomerulonephritis * Nephrotic syndrome DIAGNOSIS: 1. Direct microscopy of eggs in URINE (max at 12 noon, eggs have terminal spines remember terminal haematuria too) or STOOL (need to concentrate stools by formol -ether method; eggs have lateral spines) 2. Immunodiagnostic tests - problematic: no indication of severity of infection; do not tell past vs present infection; not species -specific; hi-tech. TREATMENT: Metriphonate - S. haematobium only; x3 doses Praziquantel - all species, single dose (sometimes repeated), no serious toxi cityct - colonic polyposis leading to iro 56-Regarding platelets: 1. They have a life span of approximately 10 days 2. Platelet production is controlled by specific cyto kines 3. Platelets should be stored between 2 and 6 degrees C 4. Platelets can be stored for a maximum of 48 hours 5. A pool of platelets transfused can produce an increment of 50 000 / ml if platelet consumption is not an issue Teaching Notes for Question 56 Theme: Platelets Platelets are produced from megakaryocytes, which are regulated by thrombopoietin, a specific cytokine. They have a lifespan of 10 -12 days thereafter being destroyed in the spleen. Platelet concentrates should be stored at around 20 degrees C and the pH kept between 6.2 and 7.8 - these conditions reduce the risk of a change in morphology of the platelets. They should also be continuously agitated to encourage gas exch ange. One pool of platelets gives on average, an increment of 10 000 / ml. 57-Concerning Salmonella infection: 1. Results in a non-invasive enteric infection 2. Infection invariably occurs following oral ingestion of contaminated food products 3. Statutory notification is required 4. Antibiotic treatment is required for Salmonellosis

63

PEDIATRICS EXAMINATION REVIEW

5. With typhoid 15-20% become chronic carriers Teaching Notes for Question 57 Theme: Salmonella infections The risk of infection following ingestion of contaminated food or water is higher in the immunosuppressed, elderly, sickle cell disease and those with reduced gastric acidity / motility e.g. postgastrectomy, hypochlorhydria. Salmonellae are highly invasive but most infections are mild and self limiting. Less than 5% of typhoid cases become chronic carriers 58-Carbon dioxide: 1. Does not affect plasma HCO3 concentration 2. Is transported in the blood as phosphate esters 3. Is transported in the blood in simple solution 4. Arterial CO2 rises during sleep 5. Arterial CO2 rises when living at an altitude of 9000 met ers Teaching Notes for Question 58 Theme: Carbon dioxide 1, 2, 3. CO2 is transported in blood in a variety of ways: - Hydrolyzed to carbonic acid in the red cells, which then dissociates to hydrogen ions and bicarbonate. Bicarbonate then diffuses into plasma in exchange for Cl ions. - Dissolved in simple solution in plasma and RBC. - As carbamino-coumpounds in combination with Hb and plasma proteins. 4. Small rise of 0.4 kPa during sleep. 5. The low PiO2 at altitude causes hyperventilation, which causes the lowering of the PaCO2. 59-Infantile hypertrophic pyloric stenosis: 1. Is more common in male babies than female babies 2. Is more common in the Afro-Caribbean population 3. Presents with projectile, bilious vomiting 4. Results in biochemical changes including low serum chloride, potassium and bicarbonate 5. Is surgically treated by excision of the pyloru s muscle Teaching Notes for Question 59 Theme: Infantile hypertrophic pyloric stenosis Pyloric stenosis occurs more commonly in males with a ratio of 4:1. It is more common in Europeans than in the Afro-Caribbean or Asian population. Vomiting normally commences at 2-3 weeks of age, and is of feed, not bilious. It becomes more frequent and projectile in nature. Typical biochemical findings are that of a hypochloraemic, hypokalaem ic metabolic alkalosis (raised serum bicarbonate levels) due to loss of gastric acid. On clinical examination during a feed, an olive shaped mass can be felt in the right hypochondrium. Ultrasound (US) examination confirms the diagnosis. Treatment is corre ction of the acidbase disturbance, followed by surgery. A Ramstedt's pyloromyotomy involves splitting of the muscle layers. 60-The following are recognised causes of drug -induced immune thrombocytopenia: 1. Chloramphenicol 2. Quinine 3. Penicillamine

64

PEDIATRICS EXAMINATION REVIEW

4. Rifampicin 5. Heparin Teaching Notes for Question60 Theme: Drug -induced immune thrombocytopenia. All the above drugs can cause thrombocytopenia. However chloramphenicol, penicillamine and phenylbutazone cause this by bone marrow suppression. The others produce an antibody-drug-plasma protein complex which deposits on the platelet surface leading to either lysis by complement or removal by reticuloendothelial cells. Patients present with acute purpura and platelets often less than 10x10 9/l. Heparin also causes thrombocytopenia through platelet aggregation. 61-The following statements are correct regarding ur inary analysis in acute renal failure: 1. There is a good correlation between functional renal impairment and the findings on urinalysis. 2. Pigmented 'muddy brown' granular casts suggest acute tubular necrosis. 3. Transparent hyaline casts are nonspecific and may be present in any case of ARF unrelated to the specific aetiology. 4. RBC casts suggest primarily renal arteriolar diseases such as vasculitis. 5. Presence of proteinuria argues against acute tubular necrosis and suggests a primary gomerular pathology. Teaching Notes for Question 61 Theme: Acute renal failure Pigmented 'muddy brown' granular casts are c haracteristic of acute tubular necrosis (ATN) and depict the sloughed out tubular epithelial cells. Hyaline casts (inactive/bland urinary sediment) are formed in concentrated urine (of any cause) by the Tamm -Horsfall protein that is secreted by the epithe lial cells of the loop of Henle. RBC casts ('active' urinary sediment) are characteristic of glomerulonephritis of any aetiology. Proteinuria of up to 1 gm per day is common in ATN; greater extent of proteinuria suggests glomerular pathology. 62-Concerning meningococcal disease: 1. Is readily distinguished on clinical grounds from other infectious agents by the spreading purpura 2. Presents in seasonal epidemics 3. Frequently co-presents with meningitis, making LP necessary at presentation 4. Mannose binding protein deficiency is a known risk factor for meningococcaemia 5. Ciprofloxacin is used for routine contact prophylaxis Teaching Notes for Question62 Theme: Meningococcal disease Other risk factors include splenectomy, terminal complement deficiencies (C6 -C9). Ciprofloxacin is used as an alternative prophylactic agent to rifampicin especially in pregnancy or those taking the oral contraceptive pill. 63-Features of the systemic inflammatory response syndrome (SIRS) include: 1. Core temperature <35 deg.cent. 2. Respiratory ra te >16/min.

65

PEDIATRICS EXAMINATION REVIEW

3. Hypercapnia with PCO2 >6 kPa. 4. Tachycardia >90 bpm. 5. Metabolic alkalosis. Teaching Notes for Question 63 Theme: Systemic inflammatory response syndrome. Sepsis is defined a s the development of a systemic inflammatory response syndrome. This may cause progression to multi-organ dysfunction syndrome. 64-The stiff-man syndrome :1. Is a syndrome associated with chronic inflammation of peripheral nerves. 2. Is a predominantly autoimmune syndrome. 3. Is uniquely associated with GAD antibodies. 4. Is associated with fibrillation and myokimeia. 5. Is best treated by benzodiazepenes and baclofen. Teaching Notes for Question 64 Theme: Stiff-man syndrome. The stiff-man syndrome (SMS) is an uncommon neurological disease, characterised by symmetrical rigidity and spasm of predominantly axial and proximal limb muscles. It may involve one limb only (stiff leg syndrome). The cause in many cases is the presence of antibodies to glutamic acid decarboxylase (GAD). Similar antibodies may be found in temporal lobe epilepsy and in diabetes. The autoim mune aetiology is confirmed by the finding of insulin-dependent diabetes in 40% of patients. The spontaneous muscle activity in SMS is of central origin not peripheral nerve dysfunction - and is related to release of polysynaptic spinal and brainstem reflexes from inhibition. If GAD function is significantly decreased, then the inhibitory neurotransmitter GABA is decreased and muscles become continuously stimulated by motor neurons. To further confuse matters, the most common pathologic correlate, anti glutamic acid decarboxylase (GAD) antibodies, have been associated with a wide range of human disease, including diabetes mellitus and seizures. Stiff-man syndrome can be treated with steroids and other immunosuppressants, but plasmapheresis or intravenous immunoglobulins are more effective. Benzodiazepines and baclofen are also helpful. 65-The following conditions will present with cyanosis in the first week of life: 1. Aortic stenosis 2. Transposition of the great vessels 3. Hypoplastic left heart syndrome 4. Fallot's tetralogy 5. Fallot's pentalogy Teaching Notes for Question 65 Theme: Cyanosis in the first week . Any cardiac lesion which allows a mixing of blood along with a right to left flow or any cardiac lesion wherein pulmonary perfusion is impaired results in cyanosis. Left heart problems or outflow tract obstructions present as cardiac failure. Fallot's pe ntalogy includes an ASD along with the tetrad of infundibular pulmonary stenosis, RVH, over -riding of the aorta and a VSD.Babies with tetralogy of Fallot usually have a patent ductus arteriosus at birth that provides additional pulmonary blood flow, so sev ere cyanosis is rare early after birth. As the ductus arteriosus closes, as it typically will in the first days of life, cyanosis can

66

PEDIATRICS EXAMINATION REVIEW develop or become more severe. The degree of cyanosis is proportional to lung blood flow and thus depends upon the degree of narrowing of the outflow tract to the pulmonary arteries. 66-Sturge-Weber syndrome :1. Occurs in 50% of children with port wine stains in the ophthalmic division of the trigeminal nerve 2. Develop hemiplegia on the ipsilateral side of the vascular lesion 3. Are associated with cataracts 4. Typically have slit like ventricles on CT 5. Skull radiograph shows 'pepperpot' skull

Teaching Notes for Question 66 Theme: Sturge-Weber syndrome Sturge-Weber is a rare (1 in 50000) sporadic combination of facial naevus and fits, hemiparesis and often low IQ. It occurs in 8% with unilateral lesion and 33% with bilateral lesion. Diagnosis is by MRI. Refractory focal tonic-clonic fits lead to a progressive hemiparesis contralaterally. Infantile glaucoma (bupthalmos) commonly occurs, ipsilaterally. CT shows unilateral dilated ventricle, calcification and cerebral atrophy. T he skull radiograph shows the intracranial calcification in the occipitoparietal lesions as tram -line or railroad track calcification.
67-Hypochromic microcytic anaemia is seen in: 1. Alpha thalassemia 2. Sickle cell anemia 3. Autoimmune hemolytic anemia 4. Folate deficiency 5. Imerslund syndrome Teaching Notes for Question 67 Theme: Hypochromic microcytic a nemia. Hypochromic microcytic anemia is usually associated with: a) Fe deficiency b) Sideroblastic anemia: - Primary X-linked recessive - Secondary alcohol - Malignancy - Inflammation In sideroblastic anemia there is Fe, but accumulation in the RBC mitochondria results in stippling on the peripheral smear and ring sideroblast formation in the bone marrow. Treatment is with pyridoxine (vitamin B6). c) Lead poisoning d) Beta-thalassemia (major or intermedia) e) Alpha-thalassemia f) Anemia of chronic disease g) Copper deficiency Sickle cell disease does not produce a microcytic anemia unless associated with thalassemia as in HbSb0. To differentiate between Fe deficiency and beta -thalassemia use: 'Discriminant function'(DF) = MCV(fl) - RBC (x109/l) - (5 x Hb[g/dl]) - 3.4 If DF is +ve = Fe deficiency If DF is -ve = beta-thalassemia trait

67

PEDIATRICS EXAMINATION REVIEW Beware; this is no use in hemodilution states (i.e. post hemorrhage or in pregnancy). Megaloblastic anemia on the other hand is usually due to: a) Folate deficiency b) Vitamin B12 deficiency c) Orotic aciduria d) Pyruvate kinase deficiency To discriminate between folate and B12 deficiency - think: -Decreased RBC folate -Hyper-segmented neutrophils -LDH All suggest folate deficiency. Imerslund syndrome is characterized by: a) Chronic mucocutaneous candidiasis b) Decreased PTH c) Proteinuria d) Decreased vitamin B12 absorption. 68-Which of the following is associated with common variable immunodeficiency? 1. Inflammatory bowel disease 2. Autoimmune disease 3. Normal levels of IgA 4. Granulomatous lesions 5. Susceptibility to recurrent parasitic infections Teaching Notes for Question 68 Theme: Common variable immunodeficiency - (CVI) CVID refers to a spectrum of disorders in which cell -mediated and antibody deficiency are combined in variable amounts. Immunological findings are variable, but it is the humora l immune deficits that usually present clinically. Some T -cell functional assays may be abnormal. Patients present with recurrent pyogenic infections, and in some cases susceptibility to infections with Pneumocystis carinii, viruses or Candida. Autoimmune disease occurs in about 10% of cases, usually immune thrombocytopenia, haemolytic anaemia or SLE. There is an increased incidence of malignancy. Granulomas are a special feature of CVID and do not occur in other primary lymphocyte disorders. Granulomatous lesions have been reported in lymphoid tissues, solid organs, and skin and share many clinical properties typical of sarcoidosis. In CVID, patients have marked reduction in serum IgG and IgA. IgM levels is also reduced in 50% of patients.

69-A 9 yr old girl is seen in clinic. Her teacher at school has reported that she has numerous episodes throughout the day of abruptly staring into space mid -conversation for a few seconds with rapid return of awareness. Which of the following features suggest complex partial rather than absence seizures as the cause of her problem : 1. Reporting unusual smells prior to the episodes 2. Provocation of the episodes by hyperventilation 3. Abnormal features on MRI scanning of the brain 4. A good response to anti-epileptic drugs 5. A normal EEG Teaching Notes for Question 69 Theme: Childhood seizures Complex partial seizures may present with brief periods of reduced awareness. Features which make complex partial seizures, rather than absence seizures likely, include: Later onset, longer duration, the presence of aura and post -ictal phenomena, focal

68

PEDIATRICS EXAMINATION REVIEW abnormalities or normal EEG (a bsences tend to have typical 3HZ spike pattern), associated MRI abnormalities (mesial temporal sclerosis, for example) and a poor response to anti epileptic therapies. 70-The following statements are reagrding asthma: 1. Leukotriene antagonists are a fi rst line treatment in asthma 2. The diagnosis of asthma involves blood tests 3. Treatment can be divided into those that prevent and those that relieve symptoms 4. Inhaled glucocorticoids improve lung function in asthmatics 5. Peak flow monitoring can be used in most children aged 5 years and above Teaching Notes for Question 70 Theme: Asthma Asthma remains a c linical diagnosis without a highly specific or sensitive test. The most reliable investigations are those of lung function that show reversibility of bronchospasm. Blood tests are not usually helpful. Peak flow monitoring is an easy way to monitor some aspects of lung function and can be used in most children >5 years and in some who are younger. Treatments for asthma continue to evolve, but the two main areas remain those of symptom relief and those that prevent attacks, bronchodilators and glucocorticoi ds respectively. Glucocorticoids can improve lung function with regular use. Leukotriene inhibitors have a useful role in some patients, but are not a first line treatment. 71-Regarding cytokines:1. IL-1 causes pyrexia and induces protein synthesis in the liver 2. Interferon gamma causes increased expression of MHC class II molecules on antigen presenting cells 3. Chemokines can induce the transmigration of T cells into inflamed tissues 4. Can be used clinically to help treat infection 5. IL-2 s the main cytokine produced by nave T cells Teaching Notes for Question 71 Theme: Cytokines IL1 is known as endogenous pyrogen. It is produced mainly by activated mononuclear phagocytes, but is also secreted by other cells e.g. epithelial cells and endothelial cells. At low concentration, IL1 mediates local inflammation. At high concentrations, it causes pyrexia and enhances acute phase protein synthesis. IFN -gamma is produces by activated CD4+, CD8+ and NK cells. It is a potent activator of macrophages and promotes the differentiation of Th1 cells. Chemokines are chemoattractant cytokines and recruit immune and other haemopoietic cells to sites of inflammation Cytokines are increasingly used in the therapeutic arena e.g. G -CSF is commonly used to treat neutropaenic sepsis; IFN -alpha is used as an anti-viral agent, and IFN-beta is used to treat multiple sclerosis. IFN-gamma is used to improve macrophage function in patients with chronic granulomatous disease. 72-15-year-old teenager presented with few haehorrhagic spots, agitation and confusion having been to school. Temp 39, pulse 110 and BP 120/80. The first manag ement step would be :1. Oxygen 2. Intravenous fluid

69

PEDIATRICS EXAMINATION REVIEW

3. Throat swab 4. Intravenous antibiotic 5. Lumbar puncture Teaching Notes for Question 72 Theme: Systemic Sepsis The history suggets meningococcal sepsis, IV antibiotics must be given immedietly. 73-The following congenital lesions are associated with cyanosis: 1. Aortopulmonary window 2. Total anomalous pulmonary venous drainage (TAPVD) 3. Coarctation of the aorta 4. Tetralogy of Fallot 5. Eisenmenger's syndrome Teaching Notes for Question 73 Theme: Cyanotic heart disease Congenital heart disease produces cyanosis when obstructio to RV outflow causes intracardiac right-to-left shunting, or when complex anatomic defects, unassociated with RV outflow obstruction, cause mixing of the pulmonary and systemic venous return. In addition, cyanosis may be caused by persistence of fetal pathways. The aortopulmonary window defect is a communication between the ascending aorta and the main pulmonary artery. The presence of pulmonary and aortic valves and an intact ventricular septum distinguishes this anomaly from truncus arteriosus. 74-Causes of hypoadrenalism include: 1. Cushing's disease. 2. Severe sepsis. 3. Tuberculosis. 4. Amyloidosis. 5. Metastatic cancer. Teaching Notes for Question 74 Theme: Hypoadrenalism Addison's disease leads to hypoadrenalism. Cushing's disease requires adrenalectomy for hyperfunction. Severe sepsis can cause haemorrhage in adrenals, also called as Waterhouse Friederichsen disease, which follows meningococcal septicaemia. 75-In Wiskott-Aldrich syndrome: 1. The immunodeficiency is restr icted to T cells only 2. Platelets are of normal size, but are few in number 3. There is an increased risk of malignancy 4. IgA and IgE levels are often low 5. Inheritance is autosomal recess ive

70

PEDIATRICS EXAMINATION REVIEW

Teaching Notes for Question 75 Theme: Wiskott-Aldrich syndrome Wiskott-Aldrich syndrome is an X-linked combined immunodeficiency. The defect is due to a deficiency in the WiscottAldrich protein- which plays a role in cytoskeletal organisation and possibly intracellular signalling. Microvilli on T-cells appear abnormal. Cardinal features are thrombocytopenia, eczema and immunodeficiency. As well as impaired cell mediated immunity (with lymphopenia) there are low levels of IgM, normal levels of IgG and high levels of IgA and IgE. Antibody responses to polysaccharide antigens are deficient. Platelets are not only few in number but are also small and these children can present w ith life-threatening bleeding. Patients also have recurrent respiratory infections and are susceptible to infections with organisms e.g. Pneumocystis carinii and herpes simplex virus. There is an increased risk of lymphoid malignancy. (mnemonic TIME: Thrombocytopenia, Immunodeficiency, Malignancy, Eczema).Bone marrow transplant is curative. 76-The following are consistent with myxedema: 1. Bradycardia 2. Intolerance to heat 3. Dry skin 4. Myopathy 5. Decreased sensitivity to opioids Teaching Notes for Question 76 Theme: Myxedema * Hypothyroidism (myxedema) can be due either to pituitary disease or autoimmine thyroiditis (Hashimoto's disease). * The classical features are lethargy, delayed deep tendon reflexes, weight gain, bradycardia, intolerance to cold, hoarse voice, greasy skin and a proximal myopathy. * Drug metabolism and excretion are reduced, so these p atients often exhibit increased sensitivity to certain drugs including opioids. 77-The following tumour markers are strongly associated with the tumours shown: 1. Alpha-FP and hepatocellular carcinoma. 2. Beta-HCG and choriocarcinoma. 3. CA-19.9 and ovarian carcinoma. 4. CA-125 and pancreatic carcinoma. 5. CEA and colorectal carcinoma. Teaching Notes for Question 77 Theme: Tumour markers The ideal tumour marker would be specific to a particular tumour, undetectable if the tumour was not present, and also sensitive, always being detectable if the tumour was present and present at a concentration directly propor tional to the volume of disease present. As no tumour marker fulfils these ideals, compromise must be sought, and they are usually more useful for monitoring response to treatment or disease recurrence than primary diagnosis. CEA is generally associated with colorectal cancer (also may be increased in other GI malignancies and in smokers). Alpha-FP is increased in hepatocellular carcinoma and in testicular germline tumours (esp. teratoma). Beta-HCG is raised in hydatidiform mole and choriocarcinoma and testicular tumours (esp. seminoma). CA-125 is associated with ovarian and CA-19.9 with pancreatic cancer.

71

PEDIATRICS EXAMINATION REVIEW

78-Arterial thromboses are associated with: 1. Behcets syndrome 2. Atrial fibrillation 3. Factor V Leiden mutation 4. Protein C deficiency 5. Polycythaemia Teaching Notes for Question 78 Theme: Arterial thromboses Arterial thromboses may be associated wi th Behcets syndrome, atrial fibrillation and polycythaemia. Factor V Leiden mutation and protein C deficiency are associated with venous thromboses. 79-Salbutamol: 1. Can cause an increase in PEFR 2. Has no effect on the FEV1 3. Has no effect on hypoxic-pulmonary vasoconstriction 4. May cause a rise in insulin release 5. Causes hypokalemia after prolonged use Teaching Notes for Question 79 Theme: Beta agonists 1,2 - Salbutamol causes bronchodilation and increases the PEFR and FEV1. 3 - It interferes with this mechanism (hypoxic pulmonary vasoconstriction) hence an adequate FiO2 is required during its admin. 4,5 - It causes a shift of potassium ions into cells. It may also increase the plasma concentration of free fatty acids and glucose, hence the rise in insulin. 80-Regarding hereditary angioedema: 1. It is due to deficiency of complement component C1 2. It can present with abdominal pain and vomiting 3. Attacks can be triggered by exercise 4. C4 levels tend to be low during an attack 5. Danazol is used for treatment Teaching Notes for Question 80 Theme: Hereditary angioedema Features of hereditary angioedema: Autosomal dominant Due to a deficiency of C1 esterase inhibitor. This glycoprotein regulates the clas sical complement pathway by dissociating C1r2s2 from C1q. In its absence, there is increased consumption of complement components C2 and C4 with the production of fragments that mediate oedema. Deficiency of C1 esterase inhibitor also results in increased formation of bradykinin which enhances oedema by acting on vascular endothelium. Clinical features include recurrent episodes of oedema of skin, gut causing abdominal pain,diarrhoea and vomiting and the upper airway. These episodes last hours to days. Stress, surgery or exercise can trigger attacks.

72

PEDIATRICS EXAMINATION REVIEW Treatment is with either danazol (boosts C1 esterase inhibitor levels) or tranexamic acid (inhibits plasmin, which consu mes

C1 esterase inhibitor).

81-Regarding Tanner stages of puberty: 1. Boys' genital development is described by 5 stages 2. Breast development stage 3 refers to breast buds 3. Pubic hair in the prepubertal state is described as Stage 0 4. Asymmetrical breast development is common in girls during puberty 5. Menarche is occurring in increasingly younger girls over the past three decades Teaching Notes for Question 81 Theme: Stages of puberty James Tanner devised stages in the 60s to describe male puberty by pubic hair and genital development and female puberty by pubic hair and breast development (as well as by menarche). * Pubic Hair: Stage I prepubertal II sparse, downy hair III IV V darker, curlier and spreading adult type hair but not on medial thighs yet adult

* Male: Genitals Stage I II III IV V prepubertal scrotum and testes grow and scrotal skin coarsens penis lengthens and testes grow penis broadens and glans develops. Testes/scrotum darken adult

* Female: Breasts Stage I prepubertal II III IV V breast bud breast and areola enlarge breast and areola enlarge to form mound above main breast adult - only nipple projects

* Menarche is either 0 or 1. 4: Asymmetrical bre ast development is a common phenomenon and just needs reassurance. 5: A recent BMJ paper looked at a sample of 100 girls to establish the average age of menarche. No data has been collected in a UK population since the 1950s. The median age of menarche was 12 years and 11 months. There was no difference related to social class or ethnicity. This suggests that there has been no decrease in the average menarcheal age in the past 3 decades. The much talked about drop was from late 19th century (14.75 years) to 1950s (12.8 years).

82-Hereditary angioedema is:1. Inherited in an autosomal recessive fashion

73

PEDIATRICS EXAMINATION REVIEW

2. A consequence of C2 kinin production 3. A cause of spontaneous life-threatening laryngeal oedema 4. Sometimes treated with the anabolic steroid danazol 5. Treated with regular C1 inhibitor infusions Teaching Notes for Question 82 Theme: Hereditary angioedema Hereditary angioedema is an autosomal dominant condition. It causes swelling (spontaneously or following trauma) of the following: i) skin -especially the face ii) intestine - resulting in abdominal pain, vomiting & diarrhoea but no point tenderness iii) airway - causing laryngeal oedema, choking and is potentially fatal. The inhibitor of C1 is grossly lacking and the resulting acute, circumscribed and non inflammatory oedema is mediated by the production of vasoactive C2 kinin. As the patients are heterozygotes they will produce very small amounts of C1 inhibitor, which can be stimulated with regular danazol (an anabolic steroid). This however, is far from popular with the girls, as it induces acne, weight gain and amenorrhoea. Purified C1 inhibitor can be given in acute severe episodes and should be given pre -operatively should one ever be required. Plasmin mediates the release of C2 kinin and therefore is amenable to blockade with e-aminocaproic acid as an alternative first line therapy. 83-Regarding the meninges: 1. The dura is adherent to the bone 2. Subdural hematomas form a lentiform shape 3. Arachnoid folds project inwards to form the falx cerebri and falx cerebelli 4. Pia stimulation during surgery is very stimulating 5. Pia is responsible for production of CSF Teaching Notes for Question 83 Theme: Meninges * The dura's rich arterial system supplie s the bone and dura. * Extradural hematomas form a lentiform shape; subdurals follow the form of the cerebral hemispheres. * The dura project inwards to form the falx. * Dural stimulation is very stimulating, and depth of anesthesia may need to be alter ed in response. 84-Pectus excavatum is associated with: 1. Marfan's syndrome 2. Rickets 3. Upper airway obstruction 4. Exercise limitation 5. Rightward shift of the heart Teaching Notes for Question 84 Theme: Pectus excavatum Midline narrowing of the thoracic cavity (pectus excavatum) is usually an isolated finding. But, it may be a manifestation of Marfans , rickets or upper/lower airway obstruction. A link has been suggested between pectus excavatum and exercise limitation, but it may be

74

PEDIATRICS EXAMINATION REVIEW habitual related to parental fears. In many children, the heart is shifted leftward. MVP and WPW syndrome appear to be ot her abnormalities associated with pectus excavatum. 85-Renal ultrasound: 1. Is a reliable means of looking for vesico ureteric reflux (VUR). 2. Has a false negative result of 10 - 20% when looking for renal scarring. 3. Has a false negative result of 15 - 45% when looking for renal scarring. 4. Has a false positive rate of 25 - 50% when looking for VUR. 5. Is a good means of detecting distal ureteric dilation. Teaching Notes for Question 85 Theme: Renal ultrasound scan. Whereas renal USS is good for detecting distal ureteric dilation, it is poor at detecting reflux (the false negative rate is 25 - 50%), as this is often not present at the time of scanning. It is also poor at detecting scarring. The broad ranges given relate to various studies, but also indicate the large amount of operator dependence found with all USS examinations 86-Regarding P.Carinii prophylaxis: 1. It is recommended in all infants born to HIV infected mothers 2. Intravenous Pentamidine is recommended 3. Previous P. Carinii infection warrants prophylaxis 4. Recommended when CD4 count drops below 1000microlitre 5. Oral candidiasis in HIV patients warrants Prophylaxis Teaching Notes for Question 86 Theme: Pneumocystis carinii prophylaxis Primary prophylaxis is warranted for all infants born to HIV mothers until infection is ruled out in the infant, CD4 count less than the cut off for the age or less than 200 in all ages, persistant oral candidiasis and progressive weight loss. Drugs used are cotrimo xazole and aerosolized pentamidine. Dapsone has also been tried. 87-The following are true about acne:1. Females with acne commonly have elevated blood androgens 2. Oral steroids are a recognised cause 3. Interleukin 1 alpha promotes comedogenesis 4. Oral isotretinoin causes hyperlipidaemia 5. The enzyme Type 1 5 alpha reductase is expressed in sebaceous gland ducts Teaching Notes for Question 87 Theme: Acne The four components of acne are (1) Seborrhoea (2) Comedogenesis (3) Colonisation of the duct with Propionibacterium acnes (P.acnes) and (4) Inflammation. Although more common in teenagers, 7% of in dividuals have acne until age 40. Androgens promote seborrhoea, but most females have normal levels of androgens - their sebaceous

75

PEDIATRICS EXAMINATION REVIEW glands respond excessively to normal levels of androgens (end -organ hypersensitivity). Apart from physiological acne, acne occurs in pathological conditions of congenital adrenal hyperplasia, and drugs such as oral steroids and anabolic steroids. Comedones represent hyperproliferation of the sebaceous follicles. IL-1 alpha promotes comedogenesis. Type 1 5 alpha reductase is an enzyme converts testosterone to dihydrotestosterone which binds to receptors in sebaceous glands increasing sebum production. Activity of this enzyme is high in sebaceous glands of acne -prone individuals. Treatment: Progress stepwise from (1) Topical treatments (2) Oral tetracyclines (3) Second-line antibiotics e,g, minocycline, doxycycline, erythromycin, trimethoprim (4) Dianette (cyproterone acetate and ethinyloestradiol) in women (5) Isotretinoin. Oral isotretinoin can cause hyperlipidaemia and der anged liver chemistry. It is a teratogen. Other side effects: headaches, muscle aches, dry eyes, dry lips and skin, hair thinning, mood alterations. 88-The following criteria must exist before a diagnosis of brain death can be made: 1. Fixed, dilated pupils unresponsive to light. 2. Absent gag reflex with pharyngeal stimulation. 3. Neuromuscular drugs should have been stopped for at least twelve hours. 4. No eye movement when 20 mls cold water is perfused into the external auditory meatus. 5. No motor response from painful stimuli. Teaching Notes for Question 88 Theme: Brain death An absent gag reflex must be assessed with bronchial stimulation via a tracheal catheter. Neuromuscular drugs should have been stopped for at least 24 hours before assesment can be made. 89-The following are true regarding parathyroid hormone actions: 1. Increases calcium and dec reases phosphate reabsorption in kidney 2. Increases osteoclastic activity 3. Increases 1,25 dihyroxy vitamin D3 production in the kidney 4. Decreases calcium and increases phosphate reabsorption in kidney 5. The overall effect is to increase calcium and phosphate level in the blood Teaching Notes for Question 89 Theme: Parathyroid The overall effect is to increase calcium amd decrease phosphate level in the blood.

90-Selective IgA deficiency:1. Is treated with iv sandoglobulin 2. is very rare (incidence less than 1 in 10000) 3. May present with autoimmune disease 4. IgG2 subclass deficiency may also be present with more severe infective complications 5. is associated with HLA-DR3

76

PEDIATRICS EXAMINATION REVIEW Teaching Notes for Question 90 Theme: Selective IgA deficiency Selective IgA Deficiency is the commonest immune deficiency (affects 1 in 500 -700 people). It is usually sporadic, but can be inherited in an autosomal dominant or autosomal recessive pattern. There is an association with MHC alleles HLA B8, DR3 and deletions of the C4A gene. Patients are often asymptomatic. If present, clinical features are usually mild recurrent sinusitis and respiratory infections (mild) or chronic diarrhoea. Autoimmune syndromes (e.g. rheumatoid arthritis , Sjogrens disease, pernicious anaemia, thyroiditis) are associated . Autoantibodies (organ -specific and non-specific) are often found even in the absence of clinical disease. Anti -IgA antibodies occur in 30 -40% and these may cause anaphylaxis if transfusi ons are administered. Serum IgA levels are usually < 50g/ml but serum IgG and IgM levels usually normal. Some patients may have additional IgG subclass deficiencies. The defect in this syndrome is a block in differentiation of IgA-expressing B cells to antibody - secreting plasma cells. Treatment is symptomatic antibiotics for infections 91-The following increase vasopressin secretion: 1. Vomiting. 2. Alcohol. 3. Haemorrhage. 4. Exercise. 5. Morphine. Teaching Notes for Question 91 Theme: Vasopressin The following stimulate vasopressin secretion: * Increased plasma osmolality, * Decreased ECF volume, * Pain, stress, exercise and standing, * Nausea and vomiting, * Angiotensin II, clofibrate, morphine and carbamazepine. The following decrease vasopressin secretion: * Decreased plasma osmolality, * Increased ECF volume, * Alcohol. Alcohol also reduces the secretion of oxytocin. 92-Regarding ketamine: 1. Is contra -indicated in reversible airways obstruction 2. Is more emetogenic than thiopentone 3. Has anti-sialogogue properties 4. May be used safely in patients with suspected raised intra -cranial pressure 5. Is a potent respiratory depressant Teaching Notes for Question 92 Theme: Intravenous anesthetic agents * Ketamine is a mild respiratory stimulant and a potent bronchodilator, making it a good agent to be used in asthmatics, either for induction or as a 3rd line treatment of an acute attack in a ventilated patient. * It causes more post-op nausea and vomit ing than thio and propfol (but less than etomidate) and cholinergic stimulation leads to increased saliva production. * It causes increased cerebral blood flow, O2 consumption and ICP and frequently leads to emergence phenomena. 93-Sickle cell disease :-

77

PEDIATRICS EXAMINATION REVIEW

1. Is caused by a mutation on chromosome 16 2. Will give a positive sickle test result at birth 3. Is associated with strokes in 25% of patients by the age of 45 years 4. Is an indication for autologous haemopoietic stem cell transplant 5. Can be co-inherited with alpha or beta -thalassaemia Teaching Notes for Question 93 Theme: Sickle cell disease Sickle cell disease is an autosomal recessive condition caused by a single amino acid mutation in the Beta globin gene on chromosome 11. The alpha globin genes are coded on chromosome 16. It can be co-inherited with alpha or beta thalassaemia. Clinical effects do not become apparent until 6 to 12 months of age when haemoglobin switching occurs and the amount of HbF (fetal haemoglobin) decreases and the amount of HbS (sickle haemoglobin) increases. This is why the sickle test, which detects the presence of sickle haemoglobin, is often not positive until 6-9 months of age and cannot be guaranteed until one year of age. Clinical problems include strokes and these are common, occuring in 25% of patients with homozygous sickle disease by the age of 45 years. Patients with se vere clinical manifestations of sickle disease have been treated with allogeneic bone marrow transplantation from an HLA -identical sibling. Autologous bone marrow transplantation is where the patients own bone marrow is replaced and would not be of benefit in this condition. 94-The femoral artery: 1. Runs as a single trunk from the inguinal ligament to the lower border of the popliteus 2. Is a direct continuation of the inte rnal iliac 3. In the upper third of the thigh the femoral artery is contained in the femoral triangle (Scarpas triangle) 4. In the lower third of the thigh, lies in the adductor canal (Hunters canal) 5. The superficial epigastric artery is a branch of this vessel Teaching Notes for Question 94 Theme: Vascular anatomy - femoral artery The femoral artery is a direct continuation of the external iliac artery. The branches of the femoral artery are: - Superficial epigastric - Deep external pudendal - Superficial iliac circumflex - Muscular - Superficial external pudendal - Profunda femoris - Highest genicular

95-Coarctation of the aorta:1. Is typical of Marfans Syndrome 2. Is typical of Noonan syndrome 3. Is associated with notching of the anterior ribs 4. Renal failure may arise, especially in the neonatal period. 5. Radiofemoral delay is the diagnostic sign

78

PEDIATRICS EXAMINATION REVIEW Teaching Notes for Question 95 Theme: Coarctation of the aorta Constrictions of the aorta of varying degrees may occur at any point from the arch to the iliac bifurcation, but 98% occur just distal to the origin of the left subclavian artery at the origin of the ductus arteriosus. The anomaly is commoner in males (2:1, M:F). It is a feature of Turners syndrome and is associated with a bicuspid aortic valve in more than 70% of patients. Mitral valve abnormalities (a supravalvular ring or prolapse) and subaortic stenosis are associated lesions. This group of left -sided obstructive lesion occurring together is termed the Shone complex. Notching of the inferior border of the ribs from pre ssure erosion by enlarged collateral vessels is common by late childhood seen in posterior one third of the ribs. Radiofemoral delay is due to the blood flow to the descending aorta being dependent on collaterals. 96-Anti-diuretic hormone (ADH): 1. Is a decapeptide 2. Has a half-life in the plasma of approximately 24 hours 3. Acts on the distal tubule of the kidney to increase reabsorption of water 4. Secretion can be suppressed by morphine 5. Secretion is increased by pain Teaching Notes for Question 96 Theme: Anti -diuretic hormone * ADH (vasopressin) is a nonapeptide, having a half -life of 5 minutes in vivo - it is metabolised by the liver and kidneys. * It acts on V1 receptors in the vascular smooth muscle giving it vasoconstrictor properties. * Activation of V2 receptors in the distal tubule and collecting duct leads to increased water permeability via cAMP . * Pain, stress, exercise, sleep and morphine all increase ADH as does a rise in the osmolality of the extracellular fluid and hemorrhage. *Alcohol is a well-known inhibitor of ADH. 97-In recurrent abdominal pain in children, the following suggest an u nderlying organic cause: 1. presentation at age 10 2. pain localising close to the umbilicus 3. family history of migraine 4. pain during daytime only 5. vomiting Teaching Notes for Question 97 Theme: Recurrent abdominal pain. Recurrent abdominal pain is very common and seldom has an organic cause. Certain features should, however, alert you to this possibility. These include presentation at less than 4 or over 15 years and pain that wakes the child at night. Abdominal migraine is one of the variants of migraine headache. It is also known by other terms including 'periodic syndrome'. This variant most typically occurs in children. They usually have a family history of migraine and go on to develop typical migraine later in their life. It is said that the further the pain is from the umbilicus, the more likely the presence of an organic cause. Obviously, altered bowel habit, vomiting, dysuria, PR bleeding and constitutional symptoms are all worrying signs, and non -organic causes can only be suggested after the exclusion of the organic ones!

79

PEDIATRICS EXAMINATION REVIEW

98-The following are inherited in an autosomal dominant manner 1. Neurofibromatosis type I 2. Hereditary spherocytosis 3. Congenital adrenal hyperplasia 4. Galactosaemia 5. G6PD deficiency Teaching Notes for Question 98 Theme: Autosomal dominant inheritance Neurofibromatosis type I is inherited as an autosomal dominant disorder. There is however a high rate of spontaneous mutation and approximately 50% of affected individuals represent new mutations. Congenital adrenal hyperplasia and galactosaemia are inherited in an autosomal recessive manner. Hereditary spherocytosis is a dominantly inherited disorder with variable expression. G6PD deficiency is X -linked recessive. 99-The following skin conditions are by definition associated with epithelial dysplasia: 1. Xeroderma pigmentosum 2. Bowens disease of skin 3. Basal cell carcinoma 4. Actinic keratosis 5. Seborrhoeic keratosis Teaching Notes for Question 99 Theme: Dysplasia in the skin. * Xeroderma pigmentosum is a condition of defective DNA repair. Although later on in the condition dysplastic lesions such as actinic keratosis may occur, the condition itself is not necessarily associated with dysplastic changes, at least in the early stages. * Bowens disease of the skin is by definition associated with carcinoma in situ of the epidermis. * Basal cell carcinoma is an invasive condition. * Actinic keratosis is a condition in which epidermal dysplasia is induced by sunlight. * Seborrhoeic keratosis is a common skin lesion, which is not associated with dysplasia. 100-Pulmonary surfactant 1. Is partly recycled by endocytosis into the synthesizing cell 2. Is produced by type 1 alveolar cells 3. Reduction in pulmonary flow can cause a decrease in surfactant production 4. Synthesis is inhibited by thyroxine 5. Synthesis is stimulated by glucocorticoids Teaching Notes for Question 100 Theme: Surfactant Dipalmityl-phosphotidyl choline is the mai n component of surfactant and is produced by Type-11 alveolar cells (granular pneumocytes). Its half -life is 14 hours and its main function is to reduce the surface tension of the alveoli. 101-Regarding sleep disturbance:

80

PEDIATRICS EXAMINATION REVIEW

1. Sleep terrors occur in non -REM sleep 2. Nightmares are associated with appreciable body movements 3. Sleep terrors last at least twenty minutes 4. Sleep terrors are associated with intense vocalisation 5. Nightmares are associated with intense vocalisation Teaching Notes for Question 101 Theme: Sleep disturbance Sleep terrors or night terrors are nocturnal episodes of extreme terror and pan ic associated with intense vocalisation, motility, and high levels of autonomic discharges. Nightmares are dream experiences loaded with anxiety or fear, of which the individual has very little recall. The dream experiences are extremely vivid and usually include themes involving threats to survival, security, or self esteem. Often there is recurrence of the same or similar frightening nightmare themes. -Sleep terrors * Occurs in non REM sleep * May be remembered * Considerable autonomic discharge * Appreciable vocalisation * Appreciable body motility * Last no more than 110 minutes -Nightmares * Occur in REM sleep * Usually vivid * Some autonomic discharge * No appreciable vocalisation * No appreciable body motility * Can continue into the second half of sleep In a typical sleep terror, child sits up or gets up with a panicky scream, often rushing to the door as if trying to escape. 102-Nitric Oxide (NO): 1. Is critical in increasing endothelin (ET -1) levels. 2. Is important in the maintenance of pulmonary vascular flow. 3. Increases cAMP. 4. Operates by inhibiting PDEs. 5. Is the same substance as EDHF. Teaching Notes for Question 102 Theme: Nitric Oxide (NO): Nitric Oxide is a critical molecule involved in processes as diverse as vasodilation, angiogenesis, cell signalling, neurotransmission, immune defence and apoptosis etc. Nitric oxide is a short-lived molecule (with a half-life of a few seconds). Its synthesis is catalysed by enzymes known as nitric oxide synthases (NOS). These enzymes convert arginine into citrulline, producing NO in the process. Oxygen and NADPH are co -factors in this reaction. There are three isoforms of nitric oxide synthase (NOS): neuronal NOS (nNOS), endothelial NOS (eNOS) and inducible NOS (iNOS). All three isoforms can be found in a variety of tissues and cell types. nNOS and eNOS are constitutively expressed in mamma lian cells and synthesise NO in response to increases in intracellular calcium and thus calmodulin levels. iNOS activity does not depend on intracellular calcium levels, but like the other NOS enzymes, the binding of calmodulin is essential for iNOS activi ty. In most cell types iNOS protein levels are very low or undetectable. However, stimulation of these cells with cytokines or growth factors, can increase transcription of the iNOS gene, with subsequent production of high concentrations of NO. The biological effects of NO are mediated through its reaction with a number of targets

81

PEDIATRICS EXAMINATION REVIEW such as haem groups, sulfhydryl groups -to produce nitrosothiols- and iron and zinc clusters. Such a diverse range of potential targets for NO explains the large number of systems that utilize it as a regulatory molecule. As a consequence of this, abnormal regulation or control of NO synthesis is capable of affecting a number of important biological processes and has been implicated in a variety of diseases. Insufficiency is believed to contribute to the pathogenesis of diseases such as atherosclerosis, hypertension and myocardial infarction. NO is also called endothelium-derived relaxing factor (EDRF) - it mediates the relaxation of vascular smooth muscle. The NO formed in vascular endothelium diffuses to smooth muscle cells, and activates guanylyl cyclase producing cGMP, which mediates smooth muscle relaxation. Importantly, NO modulates endothelin and maintains a low pulmonary vascular resistance. NO is produced by a number of immune cells. In particular, activated macrophages produce high concentrations of NO in response to LPS and interferon -gamma. NO has been shown to kill cells by disrupting enzymes involved in the Kreb's cycle, DNA synthesis and mitochondria. NO may act as a mediator of inflammatory processes. It enhances the effect of cyclooxygenases and stimulates the production of pro -inflammatory eiconosoids. Nitric oxide acts as a neurotransmitter in the central and peripheral nervous systems. It is also involved in regulating apoptosis in neurons. NO is now known to be the physiological mediator of penile erections. As part of its plethora of functions, it modulates firefly lighting! 103-Factors associated with poor wound healing include: 1. Vitamin A deficiency 2. Vitamin B1 deficiency 3. Vitamin C deficiency 4. Folate deficiency 5. Zinc deficiency Teaching Notes for Question 103 Theme: Wound healing Patients who are malnourished have poorer outcomes because of impairment and eventual failure of physiological protein -dependent functions. These may be manifest by increased infection rates (e.g. chest, urinary, wound), slo wer wound healing, wound breakdown and dehiscence and death. Trace element deficiencies often go hand in hand with malnourishment. 104-The following are true of COPD exacerbations: 1. All patients should be given antibiotics 2. Are associated both with rhinovirus and chlamydia pneumoniae infection 3. There is a positive correlation in mortality and the number of smoking pack years 4. pH less than 7.35 is an indication for antibiotics according to the BTS guidelines 5. Moraxhella catarrhalis is the most frequently implicated organism in exacerbation Teaching Notes for Question 104 Theme: Exacerbations of COPD Haemophilus influenzae is the most frequent cause, followed by S.pneumoniae and Moraxhella catarrhalis in causing bacterial exacerbations. According to the BTS guidelines if a patient has two out of the following criteria; increased shortness of breath, in creased sputum volume and increased sputum purulence, there should be given antibiotics. In addition with acute or chronic respiratory failure (pH less than 7.35) should receive antibiotics. COPD exacerbations are frequently associated with influenza and r hinovirus infection. Thus all patients with moderate or severe disease should be offered influenza vaccination seasonally.The following features are associated with increasing mortality with

82

PEDIATRICS EXAMINATION REVIEW COPB exacerbations. These are greater age, low FEV1 chronic sputu m production and the number of total pack years. 105-The following statements are correct regarding the cardiac anatomy: 1. The left atrium is larger in size as compared to the right atrium. 2. Right coronary artery is dominant in over three -fourth of cases. 3. There is one pulmonary vein from each lung that enters the posterior wall of the left atrium. 4. The AV node is supplied by the left anterior descending artery in most of the cases. 5. Most of the venous drainage of the heart occurs via the the Basian system of veins. Teaching Notes for Question 105 Theme: Cardiac anatomy The right atrium is larger in size than the left atrium in normal people. The dominant coronary artery (CA) is the one that supplies the posterior part of the interventricular septum. This is supplied by the right CA (RCA) in 85% cases (right dominant circulation) and by the left circumflex (LCx) in the remaining (left dominant circulation). The sinoatrial node is almost always supplied by the RCA. Venous drainage from the heart is mainly into epicardial cardiac veins, draining into the coronary sinus and then into the right atrium. The 4 pulmonary veins drain into the left atrium. 106-Mitral valve prolapse is associated with:1. Cyanosis 2. Bacterial endocarditis 3. Chest pain 4. A systolic 'click' on auscultation 5. Foetal arrythmias Teaching Notes for Question 106 Theme: Mitral valve prolapse. Mitral valve prolapse is a common syndrome and the vast majority of patients are asymptomatic. A variety of non-specific symptoms may be present including palpitations, postural hypotension, chest discomfort, and when MR is severe, symptoms of diminished cardiac reserve predominate. Autonomic dysfunction occurs commonly in MVP and this has been suggested as the cause for the chest discomfort. The unique physical finding in MVP is a systolic click at least 0.14sec after S1. The click is often followed by a mid - to latecrescendo systolic murmur that continues to A2. The outlook for MVP in children is excellent, a large majority remaining asymptomatic for many years. There is an increased risk of infective endocarditis, although the incidence appears to be extremely low in patients with a midsystolic click only; the incidence rises in patients with a systolic murmur. Acute hemiplegia, TIAs, cerebellar infarcts, amaurosis fugax and retinal arteriolar occlusions all appear to occur more frequently. 107-Normal arterial pH is : 1. Equal to [H+] of 34-46 nmoles/L 2. Lower than intracellular pH 3. Not essential to maintain ion distribution 4. Measured rather than calculated in a pH probe 5. Decreased in pregnancy

83

PEDIATRICS EXAMINATION REVIEW

Teaching Notes for Question 107 Theme: Acid/base balance A. pH 7.35-7.45 = [H+] 36-45nmol/L B. Intracellular pH = extracellular pH - they use this question a lot C. Normal pH absolutely essential for enzyme function/ion distribution/protein stability D. pH measured in pH probe E. In pregnancy, pH increases because of excretion of bicarbonate 108-A 16 year old girl is admitted with progressive weight loss for which no organic cause can be found. The following factors would be against a diagnosis of an orexia nervosa:1. A history of laxative abuse 2. The delusion that someone is trying to control her eating patterns 3. An excess of physical exercise 4. Self-induced vomiting 5. Low plasma potassium Teaching Notes for Question 108 Theme: Anorexia Anorexia Nervosa most commonly occurs in young women. It is characterised by an obsessive refusal to eat and altered body image. Laxative abuse and self-induced vomitting may occur in the context of anorexia. It has a mortality of up to 5%. Examination findings include emaciation, fine lanugo hair, brittle hair and nails and dry skin. A low plasma potassium may result from laxative use or self-induced vomitting. A delusion of outside control implies psychosis, this should be excluded when making a diagnosis of anorexia. 109-Defective Fas-FasL interaction leads to impairment of : 1. CTL-mediated lysis 2. Helper T-lymphocyte activation 3. NK cell killing 4. Oxidative burst within activated macrophages 5. Recruitment of neutrophils Teaching Notes for Question 109 Theme: Fas - FasL Fas-FasL interaction leads to apoptosis of the Fas expressing cells. CTL kill target cells by Fas ligand-mediated apoptosis as well as by perforin -mediated cytolysis. Natural killer cell s destroy cells by the release of cytoplasmic granules containing perforin and granzymes. They have abundant Fc gamma receptors on their surface (FcgammaRIII, CD16) which bind to the constant region of IgG, and are responsible for the destruction of antibo dycoated target cells, a process termed antibody -dependent cell-mediated cytotoxicity (ADCC). The oxidative burst refers to the process of generation of toxic products such as hydrogen peroxide, superoxide anion and nitric oxide within activated macrophag es and neutrophils upon phagocytosis. This process is catalysed by an enzyme complex NADPH oxidase and is important in killing the engulfed microorganisms. Defects in the NAPDH oxidase pathway leads to susceptibility to bacterial and fungal infections, esp ecially in infancy and results in chronic granulomatous disease. 110-The following poisons are matched to the appropriate therapy: 1. Carbon monoxide - hyperbaric oxygen 2. Organophosphates - atropine and pralidoxime

84

PEDIATRICS EXAMINATION REVIEW

3. Beta-blocker - phentolamine 4. Methanol - ethanol 5. Tricyclic antidepressants - phenytoin Teaching Notes for Question 110 Theme: Antidotes For beta-blocker overdose, try atropine, glucagon infusion, and temporary pacing. Tricyclic antidepressant overdose may require intravenous neostigmine to counteract the anticholinergic effects, and a beta -blocker for treatment of SVTs. Other antidotes include desferrioxamine for iron, calcium EDTA dimercaprol for lead poisoning, dimercaprol for heavy metal poisoning, ethanol for ethylene glycol, dicobalt edetate for cyanide, digoxin specific antibody for digoxin, naloxone for opiates, N -acetylcysteine for paracetamol, Fuller's earth for paraquat, and vitamin K for warfarin. 111-The following are true: 1. The uptake of iodide into the thyroid follicular cell is by active transport 2. T4 has a half-life of 24 hours 3. About 99% of circulating T4 is protein bound 4. T3 is roughly 10 times more potent than T4 5. All steps in thyroid hormone synthesis are stimulated by thyroid peroxidase Teaching Notes for Question 111 Theme: Thyroid hormones The main steps are as follows: 1. Active uptake of iodide, which is concentrated to 20 times that of plasma. 2. Oxidation of iodide to iodine, which combines with ty rosine residues to give monoiodotyrosine (MIT) and di-iodotyrosine (DIT). 3. Coupling of 2 DIT residues gives T4 while addition of MIT and DIT gives T3. *Both T4 and T3 are highly bound to thyroid binding globulin or albumin. *T4 has a half-life of 7 days T3 is 1 day. *T3 is 3 to 5 times more potent than T4. *TSH stimulates all steps in thyroid hormone synthesis - thyroid peroxidase catalyses the 2nd and 3rd steps. 112-Restriction fragment length polymorphisms (RFLPs): 1. Are found every 100 bases 2. Are useful in diagnosis of multifactorial diseases 3. Are useful in linked-gene diseases 4. Are generated by digestion of DNA with bacterial enzymes 5. Are detected by Northern blotting Teaching Notes for Question 112 Theme: Restriction fragment length polymorphisms The term restriction fragment length polymorphisms (RFLP) refers to the different patterns seen when DNA from different persons is digested by bacterial enzymes (restriction enzymes). These enzymes are produces by bacteris (probably as a defence against viruses) and cut DNA at specific sequences, sequences not found in the bacterium's own DNA). W hen human DNA is subjected to digestion, the DNA is cut only where the particular sequence occurs producing fragments of DNA. These can be resolved according to size by gel electrophoresis, and then the patterns of different fragment sizes compared from on e sample to another. If a mutation changes the DNA sequence, the enzyme will not cut and

85

PEDIATRICS EXAMINATION REVIEW so the sizes of the DNA fragments will be different. RFLPs can be used to link phenotype (disease) to genotype (sequence), but further detailed analysis of the exact m utation will still be necessary to identify the gene responsible. Northern blotting is used for the analysis of RNA not DNA. 113-Which of the following statements are true: 1. Data may be described as qualitative or quantitative 2. Discrete data may include number of children 3. Discrete data may include serum glucose values 4. Ordinal data does not include tumour staging 5. Nominal data may include blood group type Teaching Notes for Question 113 Theme: Types of data Data may be described as: * Qualitative only a quality can be applied and not an exact number. This can be either nominal (a naming scale) e.g. blood group type or ordinal (data can be ranked in some way) e.g. tumour staging. * Quantitative which is data to which a certain number can be ascribed. This can be either discrete (only discrete values are assumed) e.g. number of children or cont inuous (assuming any value) e.g. plasma glucose. 114-In an athymic child:1. Macrophage function is impaired 2. The development of gamma/ delta -T-lymphocytes is unaffected 3. The production of antibody is unaffected 4. The function of helper T-lymphocytes (Th) is affected while cytotoxic T -lymphocytes (Tc) activity is only moderately impaired 5. Lymphadenopathy is common because of the over -expansion of B-lymphocytes Teaching Notes for Question 114 Theme: In the athymic child The thymus is the major organ of development of both helper and cytotoxic T -cells. The absence of thymus affects the development of both alpha/beta and gamma/delta T -cells. In humans <5%of T-cells express the gamma/delta T-cell receptor. These cells tend to be CD4 and CD8 negative. Much of what is known about gamma/delta T -cells is derived from studies in mouse. Although these cells a re thymus dependent, there is some evidence that they are able to develop in athymic mice possibly in the intestinal mucosa. Gamma/ delta T-cells in mice are found predominantly in epithelial tissues. In humans, 10% of intestinal T-cells express the gamma/ delta receptor. T-cells play a vital role in macrophage function. T-helper cells activate macrophages by directly interacting with these cells via costimulatory molecules (CD40L, TNF -alpha, TNF-beta) and by secreting cytokines e.g. IFN-gamma - increasing macrophage antimicrobial activity. Humoral responses are integrally dependent on T -cell help, and therefore antibody production and B -cell function are affected when the thymus is absent. In DiGeorge, the thymus fails to develop and is associated with par athyroid agenesis (hence tetany) and congenital defects of the heart and great vessels. This is due to the failure of development of the 3rd and 4th pharyngeal pouches. Patients with residual thymic tissue may have some T-cell function which tends to improve with age; many have no T-cell defects by adolescence. 115-The following statements are true about the mitral valve 1. It is bicuspid

86

PEDIATRICS EXAMINATION REVIEW

2. It lies posterior to the sternum at the level of the fourth costal cartilage 3. The associated papillary muscles are smaller than those of the tricuspid valve 4. Its' opening is associated with the first heart sound 5. It can be regurgitant even if structurally normal Teaching Notes for Question 115 Theme: Mitral valve anatomy and physiology The mitral Valve has two cusps and large papillary muscles to hold the cusps firmly. These muscles are larger than those for the tricus pid valve as the LV pressure is higher. Closure of the mitral valve is associated with the first heart sound. Opening is only audible in mitral stenosis (as the opening snap). Mitral regurgitation can occur with dilatation of the left ventricle, even wit h a structurally normal valve. 116-Causes of vesicular rash in children include:1. Roseola Infantum 2. Dermatitis herpetiformis 3. Hand Foot and Mouth disease 4. Kawasaki disease 5. Molluscum contagiosum Teaching Notes for Question 116 Theme: Vesicular Rash Other causes of vesicular rash in children include Varicella and Herpes zoster, Herpes simplex type I, poxviral infections and Impetigo. Measles, Rubella , Kawasaki disease, scarlet fever, erythema infectiosum, roseola infantum and other viral infections present with maculopapular rash 117-The following are causes of lymphadenopathy: 1. Rubella 2. Amyloidosis 3. SLE 4. Rheumatoid arthritis 5. Thyrotoxicosis Teaching Notes for Question 117 Theme: Lymphadenopathy Rubella, SLE, Rheumatoid arthritis and thyrotoxicosis are all causes of lymphadenopathy. Sarcoidosis but not Amyloidosis is also a cause of lymphadenopathy. 118-The following statements are true for the Renin Angiotensin System: 1. Renin converts Angiotensin I to Angiotensin II in the kidney 2. Angiotensin II promotes renal arteriolar dilatation 3. Angiotensin II inhibits aldosterone secretion by the adrenal glands

87

PEDIATRICS EXAMINATION REVIEW

4. Angiotensin II receptors are blocked by spironolactone 5. Losartan inhibits formation of Angiotensin II Teaching Notes for Question 118 Theme: Rennin Angiotensin System Renin converts angiotensinogen (produced in the liver) to Angiotensin I, which is converted to Angiotensin II by the Angiotensin Converting enzyme (ACE) in the lungs. Angiotensin II promotes renal arteriolar constriction, esp of the efferent arteriole. Angiotensin II also stimulates aldosterone secretion leading to Na and water absorption. Aldosterone receptors are blocked by Spironolactone Captopril inhibits ACE. Losartan is a competitive antagonist of the A II receptor. 119-Which of the following are common sympt oms associated with raised intracranial pressure in children? 1. Morning headache 2. Abdominal pain 3. Double vision 4. Ataxia 5. Seizure Teaching Notes for Question 119 Theme: Benign intracranial hypertension Benign intracranial hypertension is characterised by increased intracranial pressure in the absence of a mass lesion or hydrocephalus. It is idiopathic i n 50%. Secondary causes include: * Drugs (corticosteroids, tetracycline, vitamin A and oral contraceptives); * SLE; * Head trauma; * Otitis media; * Sinusitis; * Adrenal insufficiency; * Hyperthyroidism; * Hypoparathyroidism; * Hyperadrenalism; * Leukaemia; * Polycythemia; * Guillain-Barr syndrome; * Iron deficiency anaemia. Headache is the commonest feature; others include diplopia, caused by 6th nerve palsy (abducens). Less commonly, some may have nausea, vomiting, paraesthaesia, neck stiffness, and ataxia. Papilloedema is a common early finding. If not treated, the papilloedema will progress and cause optic nerve atrophy. Cranial CT or MRI is essential with ophthalmology review. Lumbar puncture will help to relieve headache and preserve the vision. If the pressure is high, it should be reduced by 40% - the normal pressure lies between 12-16 mmHg. Acetazolamide can be started if patient cannot tolerate second or third lumbar puncture when symptoms recur. In spite of all these, if a child is still symptomatic with progressive loss of vision, dexamethasone can be given for two weeks or/and consider optic nerve fenestration or lumboperitoneal shunt. 120-The following is true regarding endotoxin: 1. It is a protein 2. It is heat-stable 3. It is produced by some Gram -positive bacteria 4. All endotoxin has the same physiological effect

88

PEDIATRICS EXAMINATION REVIEW

5. It is secreted by bacteria during active infection Teaching Notes for Question 120 Theme: Endotoxin Endotoxin is composed of lipopolysaccharide in the Gram -negative cell wall. Unlike exotoxin, the effects of endotoxin are all non -specific. Whereas exotoxin is secreted by viable bacteria, endotoxin is released only following bacterial cell lysis or death.

121-The following are zoonoses: 1. Legionnaires disease 2. Hydatid disease 3. Lyme disease 4. Poliomyelitis 5. Salmonellosis Teaching Notes for Question 121 Theme: Microbiology - zoonotic infections. A zoonosis is an infection that is naturally transmissible between vert ebrate animals and man. Organisms may be bacteria, viruses, protozoa, chlamydiae, etc. Important zoonoses include: Disease Animal source Campylobacter Cattle, poultry, sheep Cryptosporidiosis Sheep, cattle, puppies, kittens Salmonellosis Q fever (Coxiella) Poultry, cattle, sheep, pigs Sheep, cattle, goats

Psittacosis Birds Hydatid disease Dogs, sheep (Echinococcus granulosus) Leptospirosis (Weils) Lyme disease Rabies Toxocariasis Toxoplasmosis Rats Deer, wild rodents (via tick bites) Dogs, foxes, bats Dogs, puppies, cats Cats

Legionnaires disease (organism Legionella pneumophila) exists in water systems, cooling towers, jacuzzis etc. Poliomyelitis is found only in humans. Hydatid disease may present to a surgeon as liver cysts. 122-Regarding Parvovirus infections in children: 1. It is the causative agent for exanthem subitum 2. Arthritis is a very common manifestation in children 3. Chronic anemia can be a manifestation in some children 4. Infection in pregnancy can result in fetal anemia 5. Aplastic crises due to parvovirus infection in sickle cell disease usually requires bone marrow transplant Teaching Notes for Question 122 Theme: Parvoviris B19 Parvovirus B19 infection may be asymptomatic or cause a condition called erythema infectiosum(Fifth disease/Slapped cheek disease).It may present with fever/ malaise/rash

89

PEDIATRICS EXAMINATION REVIEW over face followed by rash over trunk. Can cause transient arthritis. It specifically affects erythrocyte precursors and in conditions with shortened life span like Sickle cell disease/Thallasemia/Hereditary spherocytosis and other hemolytic anemias it can cause aplastic crises which resolves spontaneously in 2 to 3 weeks. Usually blood transfusions are all that is needed. In pregnancy it can cause fetal hydrops and fetal anemia. In children with leukemia it can cause prolonged anemia. 123-Definition of status epilepticus: 1. Two or more fits within 30 minutes 2. Unresponsive patients who have been incontinent for 60 minutes 3. Mouth twitching lasting 60 minutes with reduced GCS 4. Tonic-clonic seizure lasting 30 minutes with incontinence 5. Rapidly repeating tonic -clonic seizure with not returning to full consiousness Teaching Notes for Question 123 Theme: Epilepsy * Status epilepticus is d efined as a condition in which continuous or rapidly repeating tonic-clonic seizures persist for 30 minutes or more. * It is often associated with incontinence but this is required for the diagnosis. * As the seizures persist they often become localized, mouth twitching is a common sign of late status, but there are many other diagnosis in a patient with mouth twitching and reduced GCS 124-Regarding Gross Motor Development, a normal child would be expected to: 1. Roll over by 2 months 2. Ride a tricycle at 2 years 3. Kick a ball at 2 years 4. Cruise by 1 year 5. Not run until 2 years Teaching Notes for Questi on 124 Theme: Milestones Importantly, a child wont roll at 2 months as spuriously suggested by perpetrators in cases of non accidental injuries. This milestone is at around 4 months. Cruising is moving around the round by holding on to furniture before in dependent walking begins and can be expected by 1 year. Running is usually achieved by 18 months. A child should kick as well as jump at 2 years. Riding a tricycle is expected at the 3 year .

125-The functional consequences of moderately severe mitral s tenosis include: 1. Reduced lung compliance. 2. Hypercapnia. 3. Reduced glomerular filtration rate. 4. Raised left atrial mean pressure. 5. Reduced pulmonary vascular resistance.

90

PEDIATRICS EXAMINATION REVIEW Teaching Notes for Question 125 Theme: Effects of mitral stenosis on heart physiology Chronic pulmonary venous congestion leads to reduced lung compliance thereby increasing the work of breathing. Reduced cardiac output reduces renal perfusion thereby reducing glomerular filtration rate. Pulmonary vascular resistance is increased due to raised left atrial pressures and chronic hypoxiemia 126-Evaluate the truth of the following descri ptions of cytokines and their functional activities:1. IL-3 : growth and differentiation of bone-marrow stem cells 2. IL-2 : activation induced cell death 3. IL-10: activation of B-cells 4. IL-6: induction of antibody secretion by plasma cells 5. IL-12: induction of IFN-gamma secretion by activated T-cells Teaching Notes for Question 126 Theme: Cytokines Cytokines often have many functions, depending on the range of target cells: IL-3 is produced by T-cells, NK cells and mast cells. It is a growth factor for progenitor haematopoietic cells (all types). IL-2 is produced by T-cells. It stimulates the growth and activation of T -cells, and may act as a co-stimulatory signal for naive CD8+ T- cells. It stimulates the growth and cytolytic activity of NK cells and high concentrations can produce lymphokine -activated killer cells (LAK cells). Paradoxically IL -2 is also important in activation induced cell death (AICD), a form of apoptosis. IL10 is produced by macrophages and some Th2 cells. It is an inhibitory cytokine - it inhibits macrophage production of IL -12, TNF and other cytokines. It also reduces expression of costimulatory molecules and MHC class II molecules on these cells. IL-6 is produced by macrophages, T-cells, fibroblasts, vascular endothelium and bone marrow stromal cells. It promotes differentiation of B -cells into plasma cells and increases antibody production. It also induces synthesis of acute phase proteins by hepatocytes and promotes myeloid stem cell differentiation. IL-12 is produced by macrophages and dendritic cells. It is a potent inducer of IFN -gamma production by T-cells and NK cells. It promotes the differentiation of Th0 cells into Th1 cells and is the most important cytokine implicated in this lineage decision. 127-Urine 1. normal pH is >5.5 2. normal Na content is <20mmol/L 3. normal osmolality is 300 -600 mosmol/kg 4. protein content of 500mg/day is within normal range 5. functional minimum urine output/day in 70kg male is 1400mls Teaching Notes for Question 127 Theme: Urine biochemistry 1 normal <5.3 2 true - one of the most basic renal functions is Na conservation 3 4 150mg / day is normal maximum 5 500mls is obligatory minimum and oliguria defined as <0.5mg/kg/hr 128-In children with renal failure

91

PEDIATRICS EXAMINATION REVIEW

1. In acute renal failure, dialysis is indicated with rapidly rising sodium 2. Anaemia of chronic renal failure is due to low plasma ferritin levels 3. Protein intake should be restricted in chronic renal failure. 4. In haemolytic uraemic syndrome, microangiopathic haemolytic anaemia is characteristic 5. Grade V vesico-ureteric reflux spontaneously resolves in only 10% Teaching Notes for Question 128 Theme: Renal failure Rising K+, hypertension, acidosis, drug OD, and rising Cr are all indications (if they are unresponsive to treatment) Erythropoitin is made in the kidneys, which stimulate s red cell production. This helps to minimise uraemia Usually seen post diarrhoea (especially E Coli 0157), involving anaemia, thrombocytaemia, and uraemia. 40% resolve spontaneously; grades I -III 85%. In children with CRF since growth failure may occu r and the metabolic demand is high, protein restriction is usually inappropriate 129-Lung surfactant 1. Decreases the surface tension within an alveolus 2. Causes an increase in chest wall compliance 3. Is a glycoprotein 4. Maintains the same surface tension for diffe rent sized alveoli 5. Appears only after the 1st week of life Teaching Notes for Question 129 Theme: Surfactant Surfactant is a dipalmitoyl-phosphatidyl choline and is a phospholipid, which prevents alveolar collapse by reducing alveoli surface tension. It is produced by type -II pneumocytes and is seen at about 24 weeks gestation. It causes an incre ase in lung compliance only (not chest wall compliance). 130-With regards to bronchiolitis, the following are true : 1. It is linked to maternal smoking. 2. RSV immune globulin is recommended in infants with cyanotic congenital heart disease to prevent severe RSV infection. 3. Ribavarin is curative. 4. Lymphopenia is common. 5. CXR commonly shows hyperinflation. Teaching Notes for Question 130 Theme: Bronchiolitis Infants whose mothers smoke are more likely to get bronchiolitis than those of non smoking mothers. RSV- IG is recommended for infants less than 2 yr with chronic lung disease or prematurity but shou ld not be given to those with CCHD because of increased complications. It is given IV/ IM just prior to and during RSV season. Ribavarin, an antiviral drug may be given to infants with cong heart disease or BPD. But there is no convincing evidence to suppo rt its use. The WBC count and differentials are usually within normal limits in bronchiolitis.CXR

92

PEDIATRICS EXAMINATION REVIEW commonly reveals hyperinflation. Scattered areas of atelectasis may be seen in approx 30%. 131-The following concerning gastrointestinal hormones are true 1. Somatostatin reduces gastric motility 2. Gastric distension increased motility 3. Glucagons increases GI motility 4. Secretin inhibits pancreatic bicarbonate secretion 5. Pancreatic polypeptide inhibits pancreatic secretion Teaching Notes for Question 131 Theme: GI hormones The roles of individual gut hormones are not completely defined, however the followi ng list details the important functions: Cholecystokinin (duodenum & jejunum) increases gall bladder contraction Increases colonic motility Gastrin (antrum & duodenum) increases gastric acid secretion Increases GI mucosal growth Secretin (Duodenum & Jejunum) increases pancreatic bicarbonate production VIP (enteric nerves) increased intestinal secretion Splanchnic vasodilation Pancreatic polypeptide (pancreas) reduces biliary and pancreatic secretion Peptide YY (ileum & colon) inhibits pancreatic ex ocrine function Neuropeptide Y (enteric nerves) regulated intestinal blood flow Motilin (whole gut) increases small bowel motility Bombesin (gut & Pancreas) stimulated pancreatic exocrine activity Somatostatin (stomach & pancreas) inhibits secretion an d action of gut hormones Glucagon (pancreas) gluconeogenesis and reduces GI motiltiy 132-Normal arterial pH is: 1. Equal to [H+] of 34 - 46 nmoles/L. 2. Lower than intracellular pH. 3. Not essential to maintain ion distribution. 4. Measured rather than calculated in a pH probe. 5. Decreased in pregnancy. Teaching Notes for Question 132 Theme: Acid/base balance 1. pH 7.35 - 7.45 = [H+] 36 45 nmol/L. 2. Intracellular pH = extracellular pH - they use this question a lot. 3. Normal pH absolutely essential for enzyme function/ion distribution/protein stability. 4. pH measured in pH probe. 5. In pregnancy, pH increases if anything because of excretion of bicarbonate. 133-The following organisms cause conjunctivitis: 1. Epstein Barr virus 2. Chlamydia trachomatis 3. Adenovirus 4. Haemophilus influenzae

93

PEDIATRICS EXAMINATION REVIEW

5. Neisseria gonorrhoeae Teaching Notes for Question 133 Theme: Organisms causing conjunctivitis Chlamydia trachomatis causes conjunctivitis in 30 -50% of neonates born to mothers with cervicitis. It is a purulent conjunctivitis, which develops 5 -14 days after birth and is indistinguishable from gonococcal infection. It is diagnosed on a swab scraped over the lower eyelid (to allow cells to be collected dont forget it is an intracellular organism) by direct fluorescent antibody, ELISA or PCR. Tetracycline ointment topically is combined with oral erythromycin the oral antibiotic is to prevent relapse after ointment is discontinued and to prevent progression to pneumonia. Gonococcal conjunctivitis presents earlier than chlamydial disease (usually within 2 days), is diagnosed on gram stain and culture and should be treated with IV penicillin and chloramphenicol eye drops. Dont forget sexual health screening for the mother and informing public health of ophthalmia neonatorum. Adenovirus causes conjunctivitis in summer outbreaks; enterovirus, coxsackie and herpes simplex are other viral causes. 134-The following statements are true: 1. The vagus nerve is responsible f or normal bronchial tone 2. There are 18 bronchopulmonary segments 3. Most of the alveoli in the adult lung are present at birth 4. The surface markings of the right middle lobe approximate to the axilla 5. The major component of surfactant is dipalmitoyl lecithin Teaching Notes for Question 134 Theme: Anatomy 2. There are 5 lobes divided into 18 segments. 3. At birth there are 25 million alveoli, which increase in the adult to 300 million per lung. 4. The right middle lobe extends from the 5th rib in the axilla to the 4th costochondral junction superiorly and to the 6th costochondral junction inferiorly 135-In beta thalassaemia:1. Beta thalassaemia major presents at birth with profound anaemia 2. The reticulocyte count is low 3. Diagnosis can be made by Haemoglobin electrophoresis 4. Anaemic patients should receive regular iron supplementation 5. Splenectomy is contraindicated due to extramedullary haemopoiesis Teaching Notes for Question 135 Theme: beta thalasseamia Beta thalassemia syndromes are a group of hereditary disorders characterized by a genetic deficiency in the synthesis of beta -globin chains. In the homozygous state, beta thalassemia (ie, thalassemia major) causes severe transfusion -dependent anemia with high reticulocyte count. The anaemia becomes manifest in late infancy. Diagnosis is by haemoglobin electrophoresis. Increased iron deposition resulting from multiple life -long transfusions and enhanced iron absorption results in secondary iron overload. This overload causes clinical problems similar to those observed with primary hemachromatosis (eg, endocrine dysfunction, liver dysfunction, cardiac dysfunction) o The physical findings are related to severe anemia, ineffective erythropoiesis, extramedullary hematopoiesis, and iron overload resulting from transfusion and increased iron absorption.

94

PEDIATRICS EXAMINATION REVIEW o Skin may show pallor from anemia and jaundice from hyperbilirubinemia. o The skull and other bones may be deformed secondary to erythroid hyperplasia with intramedullary expansion and cortical bone thinning. o Heart examination may reveal findings of cardiac failure and arrhythmia, related to either severe anemia or iron overload. o Abdominal examination may reveal changes in the liver, gall bladder, and spleen. Hepatomegaly related to significant extramedullary hematopoiesis typically is observed. Patients who have received blood transfusions may have hepatomegaly or chronic hepatitis due to iron overload; transfusion -associated viral hepatitis resulting in cirrhosis or portal hypertension also may be seen. The gall bladder may contain bilirubin stones formed as a result of the patient's life-long hemolytic state. Splenomegaly typically is observed as part of the extramedullary hematopoiesis or as a hypert rophic response related to the extravascular hemolysis. o Extremities may demonstrate skin ulceration. o Iron overload also may cause endocrine dysfunction, especially affecting the pancreas, testes, and thyroid.

136-The following statements concernin g CNS tumours are true: 1. Glioblastoma multiforme is a benign neoplasm. 2. Astrocytomas arise from the neuroglia. 3. Cerebral metastases comprise around 70% of all brain tumours. 4. Gastric carcinoma frequently metastasises to the brain. 5. Meningioma is usually benign. Teaching Notes for Question 136 Theme: Central nervous system Around 80% of brain tumours a re primary. The most frequent (around 40 - 50%) arise from the glial cells, the commonest being astrocytomas. These are of variable malignancy - glioblastoma multiforme is the most malignant subtype. Around 15 - 20% are meningiomas, usually benign and a rising from the meninges. Pituitary tumours are around 5% of the total. 15 - 25% are brain secondaries, usually multiple. The most frequent tumours, which metastasise to the brain, are bronchial carcinoma, breast carcinoma and malignant melanoma. 137-Regarding lead poisoning:1. Urinary lead concentration is the most useful indicator of exposure 2. In asymptomatic children with increased lead levels DMSA therapy is recomme nded 3. Haemodialyisis effectively eliminates lead 4. The half-life of the deep bone compartmen t is months 5. Is associated with basophilic stippling in white cells Teaching Notes for Questio n 137 Theme: Lead poisoning Unlike almost all other trace metals blood rather than urine concentrations are the most useful for assessment of lead exposure. Asymptomatic children with lead levels >46g/dL are treated with courses of DMSA (dimercaptosuccini c acid) for 2 weeks followed by a 2 week interval to assess post-treatment rebound which occurs due to equilibration from bone stores. Haemodialysis is useless at eliminating lead because of the large non -blood pools. 75% of lead in children is in the bone. There are 2 bone pools a shallow and a deep in addition to the blood, and soft tissue pool. Chelation removes lead from the soft tissue pool. The deep bone pool has a half life of decades. Lead poisoning is associated with basophilic stippling of red blo od cells.

95

PEDIATRICS EXAMINATION REVIEW

138-Benign rolandic epilepsy :1. Is a common form of partial epilepsy 2. Has a poor prognosis 3. Involves seizures that often occur at school 4. Can be successfully treated with carbamazepine 5. Has a typical EEG with a 3 per second spike and wave pattern Teaching Notes for Question 138 Theme: Epilepsy Benign rolandic epilepsy- the most common parti al epilepsy in childhood. 15-20% of childhood epilepsy. Seizures usually stop in mid teens, may not require treatment. Seizures usually occur in sleep: sensory aura followed by GTC seizures often involving face, arms. EEG shows typical cento -temporal spikes. Treatment with carbamazepine. Excellent prognosis. 139-Which of the following biological responses or consequences is mediated by the release of tumour necrosis factor (TNF) alpha? 1. Production of acute phase proteins by hepatocytes 2. Mobilisation of B-cells from the bone marrow 3. Disseminated intravascular coagulation in systemic Gram -negative bacterial infection 4. Resetting of the temperature threshold in the hypothalamus resulting in pyrexia 5. Induction of killing by cytotoxic T lymphocytes (CTL) Teaching Notes for Question139 Theme: Tumour necrosis factor (TNF) TNF-alpha is produced by mononuclear p hagocytes, activated T-cells, activated NK-cells and occassionally mast cells. It has a wide range of systemic effects. TNF increases the expression of adhesion molecules on vascular endothelium and results in increased microvascular endothelial permeability to facilitate recruitment of inflammatory cells It increases neutrophil adhesion molecule expression and mobilises neutrophils from the bone marrow. It activates phagocytes, enhancing cytokine production and microbicidal activity. Dendritic cell migrati on and maturation is enhanced. At higher concentrations, TNF acts as an endogenous pyrogen by increasing energy mobilisation in fat and muscles, and increases synthesis of acute phase proteins. It is a procoagulant, and chronic production suppresses bone marrow haematopoiesis. It also produces cachexia by inducing appetite suppression. At extremely high concentrations e.g. in gram-negative bacterial sepsis, TNF causes circulatory collapse associated with vasodilatation, decreased myocardial contractility a nd intravascular thrombosis. 140-The neck of the 1st rib is related to: 1. The thoracic duct on the right. 2. The superior intercostal artery. 3. Scalenus anterior. 4. Phrenic nerve. 5. Stellate ganglion. Teaching Notes for Question 140 Theme: Head-Neck

96

PEDIATRICS EXAMINATION REVIEW Stellate ganglion is formed by the c8 -t1 nerves. The thoracic duct is only present on the left side. The first rib is flattened from above downwards. It is the shortest flattest and most curved of the ribs. A prominent tubercle on the inner border of its upper surface marks the insertion of scalenus anterior. Structures crossing the neck of the first rib from me dial to lateral are: 1. THE SYMPATHETIC TRUNK 2. THE SUPERIOR INTERCOSTAL ARTERY 3. A BRANCH OF T1 THAT CONTRIBUTES TO THE BRACHIAL PLEXUS. 141-Which of the following would cause an increase in the carbon monoxide transfer factor? 1. Extra-pulmonary restriction 2. Churg-Strauss 3. Goodpastures syndrome 4. Exercise 5. Anaemia Teaching Notes for Question 14 1 Theme: Increase in CO transfer factor Any condition that increases the number of red cells that come into contact with the carbon monoxide will increase the TLCO and KCO. Alveolar haemorrhage, Alveolar inflammation, ( and ASTHMA). Exercise increases pulm onary blood flow. Extra-pulmonary restriction would increase the density of blood per unit lung volume with more red cells coming into contact with the CO. 142-A patient develops low back pain which is progressive. He is found to a spinal schwannoma. It is resected, but division of the posterior spinal root occurs. Which of the following is likely to result from this? 1. Loss of motor function 2. Loss of tendon reflex 3. Loss of pain, temperature and preservation of light touch 4. Spinothalamic tract degeneration abo ve the section level 5. Absence of sweating in the limb Teaching Notes for Question 142 Theme: spinal nerve root lesion Division of dorsal/posterior root usually causes loss of all sensory modalities in a dermatomal distribution, which makes it different from peripheral neuropathy which is not dermatomal, and usually not entirely sensory or motor. Posterior division causes spinothalamic tract degeneration. There is no effect on the sympathetic supply, as this is provided by the ganglia. 143-Autism :1. Can be treated successfully with psychotherapy 2. Is sometimes related to poor parenting 3. Is related to Fragile X syndrome 4. Is related to Sandifer Syndrome

97

PEDIATRICS EXAMINATION REVIEW

5. Is related to Rett syndrome Teaching Notes for Question 143 Theme: Autism and pervasive developmental disorders Features impairments in socialization, communication and imagination. More common in boys than girls. Causes: genetic (rate of autism in siblings increased, autism is a feature of f ragile X), infective aetiology, immunological theories. NOT related to parenting methods, child abuse or neglect. Usually diagnosed around 2-3 years of age as that is when speech delay becomes readily apparent. Social interaction: autistic aloneness. I nability to recognise or appreciate the feelings of others and respond to social cues. Unable to modify their behaviour on the basis of these social cues. Parents often report child not wanting to be cuddled as an infant. Late development of social smile. Gaze avoidance. Poor development of joint attention, protodeclarative pointing. Communication: verbal and non -verbal language affected. Poor speech development. Poor development of facial expression, use of gestures. abnormal use of speech - echolalia. Imagination: restricted range of interests. Stereotypies eg head banging, rocking, hand flapping. Inflexible routines. Inability to adapt to change. ritualistic behaviour. Poor symbolic or imaginative play. Aspergers syndrome: actually modern view suggests that Aspergers describes the milder autistic children ie an autistic spectrum of disorders exists. Aspergers describes those children who have impairments in the core features without learning difficulties. They tend to be diagnosed later because they a re able to cope and adapt better than severe autistics. Retts syndrome: occurs in girls. Initially normal development with regression and loss of skills before 2 years of age, especially language and manual dexterity skills. Autistic features. Treatment of developmental disorders: symptomatic treatment as causes poorly defined. Educational and behavioural methods are mainstay of treatment. 144-With regards the spleen: 1. The lymphoid function is carried out in the white pulp. 2. Following splenectomy, it is largely the kupffer cells in the liver that filter micro -organisms. 3. Following splenectomy, low dose penicillin & pneumovax are recommended. 4. The spleen is involved in the destruction of encapsulated organisms. 5. Basophilic stippling of neutrophils is seen with asplenia. Teaching Notes for Question 144 Theme: Spleen The spleen is specialized to trap and mount an immune response against blood borne antigens. It is comprised of the red pulp, which contains an abundance of erythrocytes. The white pulp forms a sleeve around penetrating arterioles and contains numerous T cells. An area called the marginal zone is closely associated with this periarteriolar lymphoid sheath (PALS) and is rich in B -cell follicles. The marginal zone and PALS are populated by interdigitating dendritic cells specialized in the presentation of antigen to T -cells. Macrophages located in the marginal zones of the spleen are very efficient at trapping and retaining polysaccharide antigens. These are then presented to B-cells and an appropriate immune response is initiated. Patients with asplenia or hyposplenia (sickle cell disease, some patients with coeliac disease) are at increased risk of overwhelming bacterial infection. The onset of symptoms may be extremely rapid, and death may occur in hours. The risk does not diminish with time post -splenectomy. It is highest in those who h ave had a splenectomy for lymphoproliferative disease (NHL, CLL) or who have concomitant liver disease. All patients regardless of underlying condition, should be on life -long antibiotic prophylaxis - either Penicillin V bd or Amoxycillin od, with a prefe rence for Pencillin V. Co-Trimoxazole may be an alternative for children, especially those with congenital asplenia, but is not recommended in adults. Adult doses: Penicillin V 250 mg bd, Amoxycillin 500 mg od. Children's dosage is 10mg/kg.

98

PEDIATRICS EXAMINATION REVIEW For Penicillin-allergic patients, erythromycin should be used (250 mg bd adult dose 10 mg/kg child). Patients undergoing elective splenectomy should receive pneumovax, haemophilus influenzae type B capsular polysaccharide - protein conjugate (either tetanus or diphtheria conjugates) and meningococcal A & C polysaccharide vaccines at least one week before surgery. All other patients should receive the 3 vaccines even after splenectomy, as there may be some benefit. Blood levels of antibodies to these polysaccharides s hould be measured before, 3 weeks after and then at yearly intervals thereafter. Booster immunization may be required if levels of antibody fall below protective levels. All adult patients should receive an annual influenza immunization. Patients travellin g to malarial areas and areas where penicillinresistant pneumococci have been found require specialist advice. Any asplenic/hyposplenic patient who develops a sudden febrile illness should be treated promptly with full dose antibiotics. Intravenous penic illin, after a blood culture (if possible), should be started immediately. Annual hospital follow-up is recommended. Basophillic stippling is seen in lead poisoning, Howell Joly bodies are seen in asplenia. 145-Chronic lymphocytic leukaemia :1. Is associated with smear or smudge cells in the peripheral blood 2. Is asymptomatic at diagnosis in less than 10% of patients 3. Should be treated aggressively in the early stages of the disease 4. Has a median survival of less than 5 years 5. Is associated with splenomegaly at diagnosis in about 50% of patients Teaching Notes for Question 145 Theme: Chronic lymphatic leukaemia Chronic lymphatic leukaemia (CLL) is due to an accumulation of mature lymphocytes in the peripheral blood, bone marrow, spleen, liver and lymph nodes. Smear or smudge cells are often seen in the peripheral blood which represent the fragile malig nant cells which have been squashed as the blood film is spread. It is asymptomatic at diagnosis in over 30% of patients, and splenomegaly is seen in 50% at diagnosis. It is often a slowly progressive disease and should not be treated in the early stages. Median survival is 3-5 years, but one third of patients die of causes other than their CLL. 146-In asthma:1. The eosinophils to the lung are recruited from the bone marrow 2. Steroids are contra-indicated if the patient is a diabetic 3. Blocking interleukin-5 maybe therapeutic 4. Leukotriene inhibitors are associated with the development of Churg -Strauss . 5. Bronchial hyper-responsiveness (BHR) does not have to be present. Teaching Notes for Question 146 Theme: Asthma, pathophysiology In asthma, by definition bronchial hyper -responsiveness must be present. IL-5 is produced by T helper cells (TH-2) and recruits eosinophils from the marow. Blocking IL -5 will therefore be therapeutic. Leukotriene inhibitors are associated with Churg -Strauss syndrome and causality can not be excluded. 147-With regards severe combined immunodeficiency: 1. Patients have defective T-cell dependent antibody responses and defective cell -mediated immunity 2. Patients are susceptible to infection with bacteria and viruses

99

PEDIATRICS EXAMINATION REVIEW

3. X-linked SCID is caused by a defect in the IL -2 receptor 4. ADA (adenosine deaminase) and PNP (purine nucleotide phosphoryl ase) deficiency cause accumulation of nucleotide metabolites that are toxic to developing T -cells resulting in T-cell and B-cell deficiencies and a SCID phenotype 5. SCID can be due to an autosomally inherited defect in DNA repair analogous to the genetic defect in the SCID mouse strain Teaching Notes for Question 147 Theme: Severe combined immunod eficiency Patients with SCID always have deficient T -cells but may or may not have absent B -cells. Several defects can lead to the phenotype of severe combined immunodeficiency. In X linked SCID there is a mutation in the gene encoding the common gamma ch ain a chain shared by receptors for a number of interleukin cytokines -(IL-2, 4, 7, 9 and 15). There is thus impaired T -cell differentiation as the interleukins are important T -cell growth factors. In this type of SCID, B-cells are present (T -B+ SCID). Humoral immunodeficiency occurs because of a lack of T-cell help. T-B+ SCIDs may also be due to an autosomal recessive mutation in Jak 3 an intracellular enzyme which is a component of the signalling pathway induced by various cytokines. T-B- SCIDs are du e to adenosine deaminase (ADA) deficiency, purine nucleoside phosphorylase (PNP) deficiency, or RAG -1 or 2 deficiency, all of which are autosomal recessive. ADA and PNP are enzymes which catalyze the metabolism of purines via the uric acid pathway. Deficiency of either of these enzymes leads to the accumulation of toxic metabolites which severely affect developing T-cells. B-cell development is also affected to a lesser extent. Functional Rag proteins are required for initiation of recombinatorial rearrangement of the genes encoding the antigen -specific receptors of both T- and B-cells. Patients with mutations resulting in non -functional Rag fail to develop both B - and Tlymphocytes. Other known causes of T -B- SCID include mutations of DNA-dependent protein kinase (DNA-PK - an enzyme involved in DNA repair) and lack of expression of MHC class II or MHC class I molecules (bare lymphocyte syndrome). Patients with SCID are susceptible to infection with bacteria and viruses as there is a complete defect in adaptive immunity. 148-Lower esophageal sphincter tone is decreased by: 1. Cisapride 2. Progesterone 3. Suxamethonium 4. Ganglion blockers 5. Ondansetron Teaching Notes for Question 148 Theme: Esophageal sphincters The effect of progesterone on oesophageal sphinter tone is one of the many reasons for the increase in gastro oesophageal reflux in pregnancy. 149-A normally developing 18 -month-old child: 1. Should recognise his/her name 2. Can easily distinguish primary colours by name 3. Plays in parallel with his/her peers 4. Should imitate observed actions 5. Should be able to copy a vertical line Teaching Notes for Question 149 Theme: Developmental milestones

100

PEDIATRICS EXAMINATION REVIEW 18-month-old child - normal developmental milestones Gross motor development: * walks well with feet only slightly apart - some children are not walking yet, but are bottom shuffling * pulls a toy as he walks * runs stiffly * climbs stairs holding on * throws a ball without falling Fine motor development: * tower of three cubes * scribbles * turns pages of a book two or three at a time * can use a spoon without rotation * can take off gloves and socks and unzip fasteners Personal and social development: - General understanding * points to own or dolls nose, hair, eyes on request - 2-3 parts of the body * imitative play * points to named pictures - Speech and language * 6-20 recognisable words * names 1-2 simple objects - Social behaviour * feeding - can use spoon without rotating it * drinks from a cup * may be dry by day (Forfar and Arneil's textbook of Paediatrics) 150-The following statements concerning congenital heart disease (CHD) are true: 1. VSDs account for 70% of all CHD. 2. ASDs account for 40% of all CHD 3. Approx. 13% of neonates with chromosomal abnormalities have associated CHD 4. Approx. 80% of children with Downs Syndrome have CHD 5. Approx. 40% of patients with Turners Syndrome have coarctation and -or aortic stenosis. Teaching Notes for Question 150 Theme: Congenital heart disease VSDs account for 25% of all congenital heart disease and are the most common anomaly. ASD (secundum) is the second most common defect and accounts for 6 -8% of all lesions. Approximately 3% of patients with congenital heart disease have an identifiable gene defect, such as Marfans or Noonans. Overall, 13% of patients with a chromosomal defect have associated CHD heart disease is found in over 90% of patients with trisomy 18, 50% of patients with trisomy 21 and 40% of those with Turners syndrome. 151-The following are clues to the cause of an upper gastrointestinal (GI) bleed: 1. Purpuric rash suggestive of Henoch-Schonlein purpura (HSP) 2. Hypopigmented spots on the lips suggestive of Peutz -Jeghers syndrome 3. Webbed neck and low hairline 4. Perianal skin tags 5. Haematochezia due to peptic ulceration Teaching Notes for Question 151 Theme: Gastrointestinal bleeding There are many causes of GI bleeds and some have clues to their origin. These include t he rash of HSP, a vasculitis that can cause GI bleeding.

101

PEDIATRICS EXAMINATION REVIEW Peutz-Jeghers syndrome - a syndrome manifesting with multiple intestinal polyps is associated with hyperpigmented spots on the lips. A webbed neck and low hairline would indicate Turner Syndrome wh ich is associated with GI vascular malformations. Perianal skin tags are found in inflammatory bowel disease. Haematochezia suggests a lower GI bleed. 152-The muscles of the soft palate are: 1. Tensor veli palatini 2. Palatoglossus 3. Levator veli palatini 4. Hyoglossus 5. Palatopharyngeus Teaching Notes for Question 152 Theme: Soft palate Hyoglossus is an extrinsic muscle of the tongue. 153-Horners syndrome invariably produces : 1. Ipsilateral ptosis 2. Contralateral mydriasis 3. Loss of sweating over the ipsilateral face 4. Heterochromia 5. Exophthalmos Teaching Notes for Question 153 Theme: Features of Horner's syndrome Horners syndrome consists of ptosis, miosis and anhydrosis and is caused by interrupti on of the sympathetic supply to the eye. The pathway starts in the ipsilateral hypothalamus and has three neurones with synapses in the lateral grey of the spinal cord at T1 and superior cervical sympathetic ganglion. Causes depend on the level of the lesion: a) Brainstem: tumour, demyelination, infarction b) Cervical cord: syringomyelia, tumour, demyelination c) T1: apical carcinoma of lung (Pancoast syndrome), cervical ribs, trauma d) Cervical sympathetic chain:thyroid carcinoma, carotid body tumour, neck surgery e) Internal carotid: dissection, jugular foramen syndromes All the features of Horners are ipsilateral. Loss of sweating may not occur if the lesion is distal to the superior sympathetic ganglion. Heterochromia refers to different pigmenta tion of the iris compared to the normal eye and is a feature of congenital Horners. The affected eyeball often seems sunken (enophthalmos) owing to slight elevation of the lower eyelid. 154-An abnormal response to oral glucose loading is seen in: 1. Malabsorption 2. Acromegaly 3. Partial gastrectomy 4. Wilson's disease 5. Gestational diabetes

102

PEDIATRICS EXAMINATION REVIEW Teaching Notes for Question 154 Theme: Abnormal glucose tolerance test No rise in glucose and so no rise in growth hormone or insulin levels are typical of malabsorption. In Acromegaly, there is a paradoxical rise in GH levels. Partial gastrectomy produces a lag response. In Wilsons disease there is an exaggerated rise in GH. An elevated glucose 2 hours after loading suggests diabetes. 155-Metabolic influences on pH 2: Which of the following statements are true? 1. Metabolic alkalosis can be induced by vomiting from the stomach. 2. Metabolic alkalosis is the reduction in H+ concentration by any means other than decreased PCO2. 3. Ventilation is reflexly depressed by metabolic alkalosis. 4. Increased central chemoreceptor firing rates decrease ventilation rates. 5. Hyperventilation induced by metabolic alkalosis returns arterial pH to the normal pre -set points. Teaching Notes for Question 154 Theme: Metabolic alkalosis Increased peripheral chemoreceptor firing rates (but not central) increases ventilation rates. Hypoventilation is induced by metabolic alkalosis to raise PCO2 and retain acid in order to bring pH back into the normal range. 155-With regard to the visual pathways: 1. A lesion of the optic chiasma will cause a homonymous hemianopia 2. A lesion of the left occipital lobe will resu lt in a right homonymous hemianopia 3. The pupillary light reflex is preserved in lesions of the optic nerve 4. Lesions of the optic nerve cause an enlargement of the blind spot 5. A pituitary adenoma may cause a bitemporal hemianopia Teaching Notes for Question 155 Theme: Visual pathway The visual pathway runs from the eye to the occipital cortex. The light reflex gives an indication of optic nerve function and will, therefore, be abnormal in lesions of the optic nerve; in addition there will be enlargement of the blind spot. Chiasmal lesions (e.g. due to compression by pituitary tumors) result in bitemporal hemianopia. A lesion of the optic tract/radiation or occipital cortex will result in a contralateral homonymous hemianopia. If incomplete, a lesion of the optic radiation with result in a contralateral upper quandrantinopia if in the temporal region, or lower quadrantinopia if in the parietal region. 156-The O2-Hb dissociation curve is displaced to the right in: 1. Fetal Hb 2. Methemoglobinemia 3. Increased lactic acid production 4. Chronic hypoxia due to cyanotic conditions 5. Increased 2,3 DPG

103

PEDIATRICS EXAMINATION REVIEW Teaching Notes for Question 156 Theme: Oxygen hemoglobin dissociation curve When shifted to the right, O2 is easily given up and this will be beneficial in conditions when the body is working hard. * Increase in CO2 production due to increased metabolism * Increase in H+ * Hyperthermia * Chronic hypoxic states/anemia, where there is an increase in 2,3 DPG 157-Intravenous immunoglobulin (IVIG) is used in the treatment of: 1. Guillain-Barre syndrome 2. IgA deficiency 3. Idiopathic thrombocytopenia purpura 4. Kawasaki disease 5. Metachromatic leukodystrophy Teaching Notes for Question 157 Theme: Intravenous immunoglobulin (IVIG). Human normal immunoglobulin is prepared from pools of at least a thousand donations of human plasma and contains antibody to common viruses. It can be given IM for protection of susceptible contacts against hepatitis A, measles and rubella. Intravenously, it is used as antibody replacement for patients with hypogammaglobulinaemia or agammaglobulinaemia but it is contraindicated in IgA deficiency because of the possibility of anti-IgA antibodies in patients with complete deficiency. For the same reason, blood should be given with caution: transfusion reactions are more likely. High dose IVIG (2g/kg) is used to treat Kawasaki disease. IVIG is now preferred to plasma exchange in the treatment of Guillain-Barre syndrome. IVIG and steroids can be used to treat ITP. Metachromatic leukodystrophy is an autosomal recessive lysosomal storage disorder. 158-In the kidney 1. The filtration fraction is the ratio of GFR to renal blood flow 2. Normal filtration fraction is 0.1 -0.2 3. Renal blood flow = plasma flow/(1 - HCT) 4. GFR can be measured using Doppler ultrasound 5. GFR can be measured using a thermodilution technique Teaching Notes for Question 158 Theme: Glomerular filtration The filtration fraction is the ratio of GFR to renal plasma flow 159-In enuresis :1. In a 3 year old investigations must be carried out 2. 5% of 5 year old children are affected 3. Urine dipstick is a useful investigation 4. First line treatment is imipramine 5. Conditioning is usually successful

104

PEDIATRICS EXAMINATION REVIEW Teaching Notes for Question 159 Theme: Enuresis Enuresis: is the involuntary voiding of urine by day and/or night. Occurs in 10-15% of 5 year olds and 3-5% of 10 year olds. Boys more common than girls. Night time wetting more common than daytime wetting. Must rule out organic causes first eg UTI, IDDM or neurological disorder. Genetic factors involved. Developmental delay in neurological maturation. Primary enuresis- when child has never been dry. Treatment: simple methods such as reducing fluid intake prior to going to bed. Mainstay is behavioural conditioning methods - bell and pad, star charts. Medication: tricyclic antidepressants, desmopressin -tablets or nasal spray. 160-Endocrine disorders that result in diarrhoea include: 1. Cushings syndrome 2. Hyperparathyroidism 3. Thyrotoxicosis 4. Primary hyperaldosteronism 5. Carcinoid syndrome Teaching Notes for Question 160 Theme: Endocrine causes of diarrhoea Addisons disease and thyrotoxicosis can both present as diarrhoea. Diarrhoea can complicate diabetes mellitus due to the aut onomic neuropathy. Carcinoid results in diarrhoea and facial flushing once metastasis to the liver has occurred. Right sided cardiac valve lesions can also occur. The diversion of typtophan from the niacin synthesis pathway can result in a pellagra diarrhoea, dermatitis and dementia. Carcinoids make up 10% of all small bowel tumours, and are commonly sited in the terminal ileum or appendix. The tumours secrete 5 HT, histamine and bradykinins. Diagnosis is by urinary 5HIAA. Treatment is by embolisation of the secondaries, resection of the primary or octreotide to reduce symptoms. 161-Regarding complex partial epilepsy:1. Absence seizures and generalised tonic -clonic seizures may occur 2. Consciousness is altered 3. Hyperventilation often provokes a complex partial seizure 4. EEG changes disappear in sleep 5. Ethosuximide is an affective treatment Teaching Notes for Question 161 Theme: Complex partial seizures Complex partial seizures, by definition, include altered consciousness. They are focal frontotemporal seizures. However complex partial epilepsy can involve generalised seizures such as absence and tonic-clonic seizures. Sleep often enhances EEG abnormalities. Hyperventilation provokes absence seizures, not complex partial seizures. Carbamazepine and sodium valproate are effective treatments for complex partial seizures. Ethosuximide is only effective in absence seizures. 162-In osteogenesis imperfecta 1. There is an increased incidence of osteosarcoma 2. Blue sclera is diagnostic 3. Pamidronate is a recognised treatment in children with this condition

105

PEDIATRICS EXAMINATION REVIEW

4. Hearing abnormalities are common 5. Diagnosis is usually made by bone biopsy Teaching Notes for Question 162 Theme: Osteogenesis imperfecta OI results from muta tion in Type I procollagen gene. Blue sclera is also found in Marfans, Ehlers -Danlos and normal infants. OI is a fairly easy clinical diagnosis to make, biopsy is rarely done. Although not a licenced treatment, bisphosphonates have been shown in series to improve BMD and reduce fractures. 163-Ventricular septal defects: 1. Can present with heart failure in the neonatal period 2. Results in wide splitting of the second heart sound 3. May regress by incorporation of the tricuspid leaflet tissue 4. Tricuspid atresia is a recognised association 5. Cardiac bypass is not necessary for surgical correction Teaching Notes for Question 163 Theme: Ventricular septal defects VSD is the most common cardiac malformation accounting for 25% of all congenital heart disease.The majority of defects are in the membranous septum, posteroinferiorly, and anterior to the septal leaflet of the tricuspid valve. In the tetralogy of Fallot, the defect lies between the crista supraventricularis and the papillary muscle of the conus, and is associated with pulmonary stenosis. The degree of left -to-right shunt is determined by the size of the defect and the pulmonary vascular resistance as compared to the SVR. A restrictive VSD (<0.5 cm2) is usually associated with normal RV pressures. In non restrictive VSDs (>1 cm2), RV and LV pressures equalise. Patients with a large VSD often have a limited left-to -right shunt immediately after birth as the pulmonary resistance may remain higher than normal. As the PVR continues to fall in the first few weeks after birth, the size of the left-to-right shunt increases and clinical symptoms become ap parent dyspnoea, feeding difficulties, poor growth, profuse sweating, recurrent chest infections and cardiac failure in infancy. Small VSDs are usually asymptomatic with a harsh pansystolic murmur at the LLSE in neonates, the murmur may be inaudible du ring the first few days of life as the pulmonary pressures are high. 164-Neostigmine: 1. Has been used in the treatment of SVT 2. Prolongs the duration of action of suxamethonium 3. Has an oral bioavailability of 50% 4. Is metabolized by plasma esterases 5. Crosses the placenta Teaching Notes for Question 164 Theme: Anticholinesterases * Neostigmine also inhib its pseudocholinesterase and so can prolong the duration of action of suxamethonium. * Its oral bioavailability is only 1 - 2%, it does not cross the BBB or the placenta owing to its highly ionized state at physiological pH and also its quaternary amine s tructure. * It is metabolized by plasma and hepatic esterases and is excreted predominantly in the urine, but also approx 30% in the bile.

106

PEDIATRICS EXAMINATION REVIEW

165-Recognized associations of systemic conditions and cataracts include: 1. Muscular dystrophy 2. Galactosemia 3. Hyperthyroidism 4. Relapsing polychondritis 5. Septicemia Teaching Notes for Question 165 Theme: Cataracts, systemic diseases A cataract is simply any loss of the transparency of the lens of the eye. The list of systemic associations with cataracts is too long to discuss in full, but important associations include: * Myotonic dystrophy (not muscular dystrophy! ); * Atopy; * Downs syndrome; * Diabetes mellitus; * Galactosemia; * Fabrys disease; * Wilsons disease; * Congenital rubella; * Toxoplasmosis and * CMV. 166-The following statements are true: 1. A moro reflex with the fists remaining clenched is normal. 2. The plantar grasp persists longer than the palmar grasp 3. Deep tendon reflex es are difficult to elicit at birth 4. An extensor Babinski response is normal in the first year 5. A newborn baby from 32 weeks gestation will turn towards a light Teaching Notes for Question 166 Theme: neuro -developmental assessment Most primitive reflexes disappear by 6 months of age. Exceptions are the plantar reflex (9 months), asymmetric tonic neck reflex (9 months), stepping reflex (11 months). The moro reflex consists of arm extension followed by flexion with extension of the fingers, and usually flexion of the t highs at the hips. A moro reflex with the fists remaining clenched is abnormal. Deep tendon reflexes are present and usually brisk at birth. The Babinski response is usually extensor until the infant begins to walk (usually from 1 year). A newborn baby from 32 weeks gestation with a normally functioning visual cortex will turn the head towards a light. 167-Glucocorticoids: 1. Are secreted from the zona fasciculata within the adrenal cortex. 2. Molecules consist of 21 carbon atoms. 3. Cause retention of sodium. 4. Inhibit gluconeogenesis. 5. Reduce lymphocyte counts.

107

PEDIATRICS EXAMINATION REVIEW Teaching Notes for Question 167 Theme: Glucocorticoids The glucocorticoids consist of cortisol and corticosterone. The main effects are in protein, glucose and fat metabolism. There is a slight mineralocorticoid effect, and also an anti inflammatory effect. 168-In the treatment of attention defici t hyperactivity disorder :1. Behavioural therapy is never useful 2. Pharmacological intervention includes the use of stimulant drugs such as amphetamines 3. Methylphenidate (Ritalin) should not be given later than 4pm 4. Parenting techniques can be taught 5. Poor appetite is a common side affect of methylphenidate (Ritalin) Teaching Notes for Question 168 Theme: Attention deficit hyperactivity disorder Treatment of attention deficit hyperactivity disorder: stimulant medication methylphenidate (Ritalin), dexamphetamine. Not to be taken after 4pm. Side effects include difficulty sleeping, loss of appetite, ab dominal pain, headache, thrombocytopenia. Behavioural modification, parenting techniques. Outcome: symptoms tend to lessen as child gets older. 169-Concerning the synthesis of thyroid hormones: 1. TRH is released by the anterior pituitary. 2. Iodine is taken up passively by the thyroid gland. 3. Thyroid peroxidase catalyses the formation of thyr oxine. 4. T4 is released bound to thryroglobulin. 5. T4 is released in greater quantities than T3.

Teaching Notes for Question 169 Theme: Thyroid physiology TRH is secreted by the hypothalamus. It stimulates release of TSH from the anterior pituitary, which in turn u p-regulates synthesis of hormones by the thyroid gland. Iodine is taken up by secondary active transport, coupled with sodium. Subsequently it is oxidised and coupled to thyroglobulin. Iodination of the tyrosine residues of thyroglobulin leads to the prod uction of T3 and T4, which are cleaved from the parent molecule prior to release. 170-The following diseases are inherited in an autosomal recessive manner: 1. Insulin dependant diabetes 2. Primary ciliary dyskinesia 3. Batten's disease 4. Congenital adrenal hyperplasia 5. Pyloric stenosis Teaching Notes for Question 170 Theme: Autosomal recessive diseases Diseases that are inherited in an autosomal recessive manner are usually those in which

108

PEDIATRICS EXAMINATION REVIEW there is a biochemical defect. Primary ciliary dyskinesia (PCD) is an exception since at least one of the genes encodes a structural protein of the cilia, dynein. The inh eritance of pyloric stenosis and insulin dependant diabetes is multifactorial although it has been proposed that pyloric stenosis is inherited as autosomal dominant with reduced penetrance on a multifactorial background.Batten Disease is an AR disorder als o known as SpielmeyerVogt-Sjogren -Batten Disease, it is the most common form of a group of disorders called Neuronal Ceroid Lipofuscinoses (or NCLs). Affected children suffer progressive mental impairment, worsening seizures, and progressive loss of sight and motor skills. 171-Noradrenaline 1. Is a neurotransmitter at the pre-ganglionic sympathetic nervous system 2. Is produced by the adrenal cortex 3. May cause a reflex bradycardia when given by infusion 4. Causes a rise in metabolic rate 5. It acts predominantly via the beta adrenoceptor Teaching Notes for Question 171 Theme: Catecholamines Noradrenaline is a catecholamine produced by the adrenal medulla. It also acts as a neurotransmitter in the post-ganglionic sympathetic neurons and parts of the brain. Its actions are mediated mostly through the alpha -adrenoceptor, though it does have some action at the beta-adrenoceptor. It causes vasoconstriction, leading to a rise in blood pressure. It activates the baroreceptor reflex, and hence may cause a reflex bradycardia. It causes an increase in metabolic rate, hence, in the heart, although coronary perfusion may be increased, myocardial oxygen consumption will also be increased. 172-In a healthy baby the transition from fetal to neonatal circulation involves: 1. Functional closure of the foramen ovale in the first 24 hours 2. Blood flow in the ductus arteriosus continues from right to left until its closure 3. Decrease in pulmonary artery resistance following closure of the ductus arteriosus 4. The ductus arteriosus closes in response to decreased oxygen concentrations 5. The umbilical artery is a branch of the common iliac artery Teaching Notes for Question 172 Theme: Cardio -pulmonary changes at birth Functional closure of the ductus arteriosus occurs soon after birth but anatomical closure can take upto one week. As pulmonary pressures fall after birth, blood flow in the ductus is reversed ie from left to right. The umbilical artery is a branch of the internal iliac artery. Factors influencing closure of the ductus include: 1. Increased oxygen concentrations 2. Decreased prostaglandin levels 3. Drop in pulmonary artery pressures N.B. Prostaglandin E2 keeps the ductus open. 173-The pancreas: 1. Is entirely retroperitoneal 2. Is supplied by branches of the coeliac artery alone 3. Is crossed by the attac hment of the transverse mesocolon

109

PEDIATRICS EXAMINATION REVIEW

4. Is crossed by the left renal vein 5. Is related to the common bile duct posteriorly Teaching Notes for Question 173 Theme: Abdomen Tail of pancreas lies intraperitoneally and can be damaged easily during splenectomy when the splenic vessels are clamped. 174-IgA deficiency:1. Is seen in Kartagener's syndrome 2. Is usually asymptomatic 3. May delay diagnosis of coeliac disease 4. Spontaneous recovery is a recognised feature 5. Affects both serum and secretory IgA levels Teaching Notes for Question 174 Theme: IgA deficiency IgA deficiency may be seen in: i) CVID (all Ig groups) ii) Bruton's agammaglobulinemia (all Ig groups) iii)Transient hypogammaglobulinemia of infancy (IgG,M+A) iv) Ataxia telangiectasia (IgA+E) v) Bloom's syndrome (IgA+M) vi) Kartagener's syndrome (IgA) vii)Selective IgA deficiency (IgA)Selective IgA deficie ncy is common (1:350) but the incidence is not universal worldwide (rarer in Japan 1:1,800), affecting the levels of IgA (A1, A2 & secretory forms). Most people are completely asymptomatic, but occasionally there is an increased incidence of respiratory, GI and GU infections. Increasingly the diagnosis is been made following investigation of FTT where the diagnosis of coeliac disease is made based on serology (antigliadin & endomyesial IgA) and therefore it is worth bearing in mind in a negative coeliac screen where malabsorption seems likely and other conditions are similarly ruled out. Alternatively, pre - and post- jejunal biopsy should be performed. In selective IgA deficiency there is an increased incidence of cows' milk allergy, anaphylaxis, autoimmune disease and malignancy. In transient hypogammaglobulinemia of infancy following the erosion of maternal passive immunity there may be a delay of up to 36 months before full humoral immunity is restored. The levels of IgG & A are predominantly affected, bu t also IgM. 175-Abnormalities associated with foetal alcohol syndrome include: 1. Short palpebral fissures 2. Maxillary hyperplasia 3. Hyperactivity in childhood 4. Intra uterine growth retardation 5. Hypotonia Teaching Notes for Question175 Theme: Fetal alcohol syndrome The incidence of fetal alcohol syndrome is thought to be as high as one in 1000 babies born in UK. The greater the intake of alcohol the more severe the signs in the neonate. Up to 32% of infants born to heavy drinkers demonstrate congenital malformations. Characteristic features include IUGR, microcephaly, moderate to severe mental retardation, irritability in neonatal period,, hyperactivity in childhood, hypotonia, facial

110

PEDIATRICS EXAMINATION REVIEW abnormalities such as short palpabral fissures, epicanthic folds, maxillary hypoplasia, micrognathia, and thin upper lip, cardiac defects (mainly septal defects). There is some evidence to suggest that alcohol may interfere with transfer of essential amino acids and zinc, which are necessary for protein synthesis. 176-The seventh cranial nerve: 1. Has bilateral cortical representation. 2. Receives taste fibres from the palate. 3. When lesioned below the level of the stylomastoid foramen will cause hyperacusis. 4. Receives taste fibres from the anterior 2/3rd of the tongue via the greater petrosal nerve. 5. Has a sensory supply in the ear canal. Teaching Notes for Question 176 Theme: The seventh cranial nerve The difference between the pattern of facial weakness seen in lower an d upper motor neurone lesions is explained by the presence of bilateral cortical representation. The seventh nerve receives taste fibres from the palate (via the greater petrosal nerve) and the anterior 2/3rd of the tongue (via the chorda tympani and the lingual branch of the fifth). Herpetic vesicles may be seen in the ear canal in patients with geniculate zoster (Ramsay Hunt syndrome): this is why the sensory supply to this area is important. 177-Strokes in childhood are associated with : 1. Nephrotic syndrome 2. Activated protein C resistence 3. Systemic lupus erythematosis 4. Haemoglobin SS 5. Moya moya disease Teaching Notes for Question 177 Theme: Cerebrovascular Disease Strokes are rare in childhood. Due to arterial occlusion, venous thrombosis. Causes: -sickle cell disease -arteriovenous malformations and vascular dysplasia eg moya moya disease -vasculitides eg SLE, PAN -coagulopathies eg AT III, protein C and S deficiency, activated protein C resistence. -dehydration -trauma -metabolic disorders eg MELAS, homocysteinuria 178-The following statement(s) concerning inherited immunodeficiency s yndromes are true: 1. The commonest inherited form of severe combined immune deficiency is x -linked 2. The incidence of lymphoid malignancy is increased in ataxia telangiec tasia 3. The defect in Chediak-Higashi syndrome is in T-cell activation 4. Deficiency in glucose-6-phosphate dehydrogenase results in impaired motility of leukocytes 5. The molecular defect in X -linked hyper IgM syndrome is in the expression of CD40 on B -cells

111

PEDIATRICS EXAMINATION REVIEW Teaching Notes for Question 178 Theme: Inherited immunodeficiency syndromes Several defects can lead to the phenotype of severe combined immunodeficiency, the commonest inherited form is X-linked. There is a mutation in the gene encoding the common gamma chain a chain shared by receptors for a number of interleukin cytokines. There is thus impaired T -cell differentiation as the interleukins are important T -cell growth and differentiati on factors. Humoral immunodeficiency occurs because of a lack of T-cell help. The commonest autosomally inherited forms are adenosine deaminase and purine nucleotide phosphorylase deficiency. ADA and PNP are enzymes which catalyze the metabolism of purines via the uric acid pathway. Deficiency of either of these enzymes leads to the accumulation of toxic metabolites which severely affect lymphocyte production and function. Other known genetic defects that lead to severe immunodeficiency include mutations of DNA-dependent protein kinase (DNA-PK) and recombination activating genes (RAG). Lack of expression of MHC class II or MHC class I molecules also cause severe immunodeficiency. Ataxia telangiectasia is due to defects in ATM, a protein that is important in DNA repair. Patients have impaired cell mediated immunity, low IgE, IgA and IgG2 and are prone to respiratory infections and skin infections. Lymphoid malignancies as well as carcinomas occur with with increased frequency in these patients. Chediak -Higashi syndrome is an autosomal recessive disorder, characterised by giant granules in phagocytes due to a genetic defect in intracellular vesicle formation and hence impaired intracellular killing following phagocytosis. Other clinical features include partial oculocutaneous albinism and abnormal platelet function. Cytolytic T -cell-mediated killing is normal, but NK killing is impaired. G6PDH is an important enzyme in the hexose monophosphate shunt. This pathway maintains glutathione in a reduced state protectin g the red cell membrane from oxidant stress . The enzyme also functions in the respiratory burst - a process important for intracellular killing by phagocytes. Complete G6PDH deficiency results in susceptibility to chronic bacterial and fungal infection - the clinical picture resembles a mild form of chronic granulomatous disease. X -linked hyper IgM syndrome is due to the lack of CD40 ligand (CD40L) expression on activated T -cells. This leads to impairment in immunoglobulin isotype switching and germinal ce ntre formation as CD40/CD40L interaction is needed for these processes. Serum IgM levels are high with deficiency of IgG and IgA. Clinically, these patients are particularly susceptible to opportunistic lung pathogens such as Pneumocystis carinii. 179-The following diseases may be transmitted by needlestick injury: 1. Hepatitis C. 2. Hepatitis B. 3. Malaria. 4. Tuberculosis. 5. HIV. Teaching Notes for Question 179 Theme: Needlestick injury Over 20 infections have been shown to be transmissible by needlestick injury. Prevention is key, by sharps awareness training, use of safer needles, sharps bins and so on. Management of an injury should include encouraging bleeding, washing and disinfecting the wound and reporting the incident, with attendance at Occupational Health. The patient may be approached for consent for testing for HIV, HBV and HCV. 180-Leptospirosis:1. Most recover within two weeks without treatment 2. May be diagnosed on urine culture 3. Is treated with tetracycline

112

PEDIATRICS EXAMINATION REVIEW

4. May be complicated by thrombotic thrombocytopenic purpurea 5. Is more common around the New Forest Teaching Notes for Question 180 Theme: Leptospirosis Diagnosis relys on demonstration of leptospira in blood and CSF, or serologi cally. The disease affects predominantly men, and is commonly acquired through occupational contact (L hardjo from cattle), via contact sports (L hardjo), or exposure to rat urine (L.icterohaemorrhagiae); the organism gains entry via mucous membranes or cu ts. A minority of patients develop meningitis, cholestatic jaundice and renal failure (Weil's disease). Treatment: penicillin, and if treatment is prompt, the renal failure of Weils disease can recover. 181-The lymphatic system: 1. Is a valveless system 2. Maintains a uni-directional flow of lymph towards the thoracic ducts 3. The thoracic ducts drain into the venous circulation at the junction of the sub -clavian veins and internal jugular veins 4. Has a total lymph drainage of 6-8 litres of fluid/24 hours from inter stitial fluid 5. Contains lymph draining from the liver and intestine with a protein concentration of approximately 40 -60 g/l Teaching Notes for Question 181 Theme: Lymphatic system * Lymph flows in single layered lymphatic capillaries into lymphatic vessels that contain smooth muscle. * Distension by the lymphatic fluid causes contraction of this smooth muscle. * Valves then direct flow towards the thoracic ducts in a uni -directional fashion. * The total lymph drainage for an adult, from the interstitial fluid, is about 2 -4 liters/24 hours. 182-The following have a much lower concentration in th e cerebrospinal fluid (CSF) than in the cerebral capillary blood. 1. K+ 2. Na+ 3. Osmolarity 4. Protein 5. Sugar Teaching Notes for Question 182 Theme: Cerebrospinal fluid CSF has a very low plasma concentration because large protein molecules cannot cross the blood-brain barrier. The Na+ concentration is roughly the same or slightly more in CSF and the K+ concentration is slightly less in CSF. The CSF sugar content is about two third than that of blood. 183-Co-administration of the following pairs of drugs results in reduced absorption of the latter:1. Antacids and prednisolone

113

PEDIATRICS EXAMINATION REVIEW

2. Cholestyramine and warfarin 3. Sucralfate and phenytoin 4. Amitriptyline and digoxin 5. Metoclopramide and paracetamol Teaching Notes for Question 183 Theme: Absorption Aluminium and magnesium (in antacids) form insoluble complexes with tetracyclines, iron and prednisolone. The antimuscarinic effect of tricyclics reduces gastric emptying and delays absorption. Metoclopromide increases gastric emptying and is used in combination with analgesics in migraine to enhance their absorption. 184-Toe-walking in an 18 month old child : 1. Is always abnormal 2. Spinal cord imaging may be indicated 3. Serum creatine kinase should be measured 4. Does not need to be investigated in a girl 5. May be a sign of cerebral palsy

Teaching Notes for Question 184 Theme: Abnormal Gait Pathological causes of toe walking are i) neuromuscular disorders e.g. Duchenne muscular (where initially due to proximal weakness and pelvic instability shifting the centre of gravity forward and later to tightness of the tendo a chilles) dystrophy. Creatine kinase is a muscle enzyme and is increased in conditions with increased muscle turnover e.g. muscular dystrophies or myopathies. ii) upper motor neurone lesions (i.e. in brain and spinal cord) due to extensor hypertonus and tightness of tendo achilles. Toe walking may be a feature of diplegic cerebral palsy. Toe walking may be normal but this is a diagnosis of exclusion.
185-Conn's syndrome: 1. Is more common in females. 2. Is due to atrophy of zona glomerulosa. 3. Causes muscle weakness. 4. Leads to sodium retention. 5. Leads to increased levels of plasma angiotensinogen 2.

Teaching Notes for Question 185 Theme: Conn's syndrome Conn's syndrome is common in males and is associated with hypertrophy of zona glomerulosa, leading to excess aldosterone production. Its other name is primary hyperaldosteronism. The best method of diagnos is is to measure the blood levels of two hormones; aldosterone and renin (which play a role in stimulating aldosterone production). In Conn's syndrome (primary hyperaldosteronism), the aldosterone level is elevated whilst the renin level is low or undetec table.

114

PEDIATRICS EXAMINATION REVIEW

186-Haemochromatosis:1. Is associated with vitamin C deficiency 2. Is associated with Jaccoud's arthropathy 3. Is more common in men 4. Is associated with HLA B8 5. Is associated with a gene on chromosome 13 Teaching Notes for Question 186 Theme: Haemochromatosis Hemochromatosis is autosomal recessive and is carried on the short arm of chromosome 6. W ilson's disease is associated with a locus is on chromosome 13. Hemachromatosis is associated with HLA A3,B14 rather than B8,DR3. Menstruation protects homozygous women from the effects of iron overload. Erosive arthritis affecting the hip/knee/wrist and 2 nd and 3rd MCP joints occurs, as does chondrocalcinosis in 50 percent of cases. Jaccoud's arthropathy is non erosive and associated with SLE, acute rheumatic fever and hypocomplementaemic vasculitis. There is an associated Vitamin C deficiency. Treatment is with venesection and desferrioxamine (one molecule of desferrioxime binds one molecule of iron).
187-Phagocytic Cells :1. Have receptors that recognise bacterial cell wall constituents 2. Require clonal expansion to mount an effective response 3. Release cytokines wich affect the adaptive immune system 4. Are important for the resolution of inflammation 5. Initiate firm attachment by binding to ICAM on the endothelium

Teaching Notes for Question 187 Theme: Phagocytic Cells The major phagocytic cells of the immune response are macrophages and neutrophils. Phagocytes of the innate immune system recognise com mon motifs on invading pathogens using cell surface pattern recognition receptors. They mount a rapid response that does not require clonal expansion. These cells phagocytose foreign particles and microbes. They are also important in clearing apoptotic cells and debris in the resolution phase. Phagocytic cells have surface receptors that recognise the Fc portion of immunoglobulin as well as complement components. Binding of these receptors to their ligands enhances phagocytosis of antigen - a process termed opsonisation. Internalised microbes are killed by phagocytes which secrete reactive oxygen intermediates and possess potent microbicidal properties. These cells initiate and amplify the adaptive immune response by producing pro -inflammatory cytokines such as IL1, IL6 and TNF. Phagocytic cells enter tissues by rolling on selectins and binding via integrins to ICAM (and similar molecules). They transmigrate by binding to PECAM and similar adhesion molecules.
188-With regards to aniridia:1. Two thirds of the cases are sporadic. 2. It is usually bilateral.

115

PEDIATRICS EXAMINATION REVIEW

3. Vision is commonly decreased. 4. It is associated with partial deletion of long arm of chromosome 11. 5. Wilm's tumour is associated with familial aniridia.

Teaching Notes for Question 188 Theme: Aniridia In aniridia, the iris tissue is hypoplastic, not usually absent. Two thirds of the cases are dominantly inherited and 1/3rd are sporadic. It is bilateral in 98% of cases. Macular and optic nerve hypoplasia are common and leads to visual impairment. Corneal and lens abnormalities may occur and glaucoma occurs in 75% of patients with aniridia. Wilm's tumour ma y occur in 1/5th of patients with sporadic aniridia. There is an association of anirida, genitourinary anomalies, mental retardation and a partial deletion of the short arm of chromosome 11.
189-Immune Complex deposition in the kidney occurs in: 1. Goodpasture's disease 2. Infective endocarditis 3. Systemic lupus 4. Polyarteritis nodosa 5. Plasmodium malariae infection

Teaching Notes for Question 189 Theme: Immune complex deposition In Goodpasture's disease, circulating antibodies bind to the antigen in the glomerular basement membrane and the alveolar membrane. There are no circulating immune complexes. Immune complexes are also found in SLE, serum sickness, Takayasus arteritis, hepatitis B antigenemia, etc
190-Regarding oxygen measurement: 1. Red light absorbs more oxyhemoglobin than deoxyhemoglobin 2. The isobestic point will vary with hemoglobin concentration 3. The readings from pulse oximeters are affected by anemia 4. The clark electrode uses a silver/silver chloride cathode 5. Water vapor will cause inaccurate readings from a paramagne tic analyzer

Teaching Notes for Question 190 Theme: Oxygen measurement * Red light absorbs, less oxyhemoglobin than deoxyhemoglobin, hence oxygenated blood looks red. * The isobestic point (where absorption of the two forms is the same) depends on hemoglobin concentration. * Pulse oximeters will not give accurate readings in states of vasoconstriction or venous congestion, high carbon monoxide levels, methemoglobinemia or lo w oxygen tensions. * Nail varnish/ stains on the skin or nail will also distort the readings.

116

PEDIATRICS EXAMINATION REVIEW

* Dark skin, jaundice or anemia will not affect the results. * The clark or polarographic electrode features a platinum cathode and Ag/AgCl anode in an electrolyte of KCl. * Water vapor tends to affect the readings of most gas analyzers.
191-The following syndromes/ associations are associated with congenital heart disease. 1. holt-Oram syndrome. 2. CHARG association. 3. Vacterl association. 4. Septo-optic dysplasia. 5. Goldenhar

Teaching Notes for Question 191 Theme: Congenital heart disease. Holt-Oram syndrom e is an autosomal dominant syndrome consisting of limb defects, thumb/radius/ulna hypoplasia associated with atrial & ventricular septal defect. CHARGE =Coloboma, heart defects (Tetralogy of fallot, coarctation of the aorta,and ventricular septal defect, c hoanal atresia), growth/mental retardation, genitourinary abnormalities, ear abnormalities including deafness. Cardiac defects include VSD & ASDs, conotruncal malformations, PDA&tetralogy of fallot VACTERL=Vertebral, anal atresia, cardiac (VSD), tracheo -esophageal fistula,oesophageal atresia, renal,limb defects. The most common associated heart defects are VSD,ASD &Tetralogy of fallot. Less common defects are truncus arteriosus and transposition of the great arteries. Septo-optic dysplasia is a midline defect consisting of hypopituiterism and optic nerve hypoplasia with absence of the septum pellucidum, there may be midline defect of the heart (VSD, ASD). Goldenhar Syndrome is associated with VSD, PDA, and Coarctation of the aorta,and transposition of the great arteries, in addition they may be renal anomalies, neurological & musculoskeletal anomalies. .The cardinal features are anomalies of the face, ears &vertebrae.

192- Regarding myasthenia gravis in childhood: 1. The neonatal form is always due to maternal antibodies. 2. The diagnostic test is subcutaneous endophonium. 3. EMG shows decremental response to repetitive motor nerve stimulation. 4. Thyroid dysfunction is a recognised association. 5. The disease can be confined to ocular muscles.

Teaching Notes for Question 192 Theme: Myasthenia Gravis. Neonatal form is commonly due to maternal antibodies, but may also be an inherited channelopathy. In 40% of childhood myasthenia, the disease remains confined to the occular muscles. Generalised weakness is present at onset in 40% or in 20% develops within the year. 10% of juvenile cases have associated hyperthyroidism.

117

PEDIATRICS EXAMINATION REVIEW

193- The following statements regrd the treatment of asthma: 1. All patients should be on inhaled steroids. 2. High dose regular short acting bronchodilators should be tried before inhaled steroids. 3. Either long acting beta-2 agonists (LABAs) and low dose steroid or high dose inhaled steroid can be used to the same clinical effect. 4. Patient education is emphasized. 5. Leukotrene inhibitors are recommended in step 4.

Teaching Notes for Question 193 Theme: Treatment of asthma. Some patients with mild or exertional asthma can be controlled on short acting beta agonists alone. Opinion about what is exce ss short acting beta agonist use between cinicians however. 9 studies with serevent and fluticasone and one with eformoterol and budesonide confirm that steroid LABA combination is as effective inhaled steroid dose. In the new BTS/SIGN asthma guidelines LA BA therapy is recommended at step 3 before increasing inhaled steroid dose., with other therapies such as leukotriene inhibitors coming later in the treatment cascade.
194- Regarding child development: 1. Sense of taste is present from birth. 2. 25% of children are left handed. 3. Left handedness is more common in exprem babies without cerebral p alsy. 4. Left handed children are more likely to have un intentional injuries. 5. Prevalence rate of thumb-sucking in children are universal across races.

Teaching Notes for Question 194 Theme: Child development. Sense of taste is prersent from birth. Infants show prefe rence for sweet over salty or plain water. 7 -10% of children are left handed. This is a surprise as animals e.g. cats & dogs are equally left or right paired. Up to 25% of exprem babies without cerebral palsy are left handed but the reason is unclear. It is thought to be due to antecedent brain injury. Left handed children are more likely to have un intentional injuries as well as a higher likelihood of developmental disorders e.g.dyslexia and autism. Prevalence rates of thum -sucking in children vary from 4 5% of US children to 1% in eskimos. It tends to be less where children are carried by parents.
195-Persistant, conjugated hyperbilirubinaemia may be caused by: 1. Alpha-1-antitrypsin deficiency. 2. Hypothyroidism. 3. Haemolytic disease. 4. Cytomegalovirus infection. 5. Cystic fibrosis.

118

PEDIATRICS EXAMINATION REVIEW

Teaching Notes for Question 195 Theme: Neonatal jaundice. all cases need further investigation. Causes of conjugated hyperbilirubinaemia in a neonte are: 1)-Intrahepatic cholestasis -Infections. -Congenital infections (STORCH-syphlis, toxoplasma, rubella, cytomegalovirus, hepatitis, herpes virus infection) -Acquired infections- septicaemia, UTI. -Metabolic disorders cystic fibrosis, alpha-1- antitrypsin deficiency, galactosaemia, fructosaemia, lysosomal storage disorders, peroxisomal disorders. -Endocrine disorders hypothyroidism, hypopituitarism, hypoadrenalism. -Anatomical disorders intrahepatic:bile duct hypoplasia. -Miscellaneous idiopathic neonatal hepatitis, chromosomal abnormalities, trisomy 21, 18 and 13. 2)-Extrahepatic cholestasis, biliary atresia, choledochal cyst, spontaneous bile duct perforation, inspissat ed bile syndrome. Unconjugated neonatal jaundice is normally physiological and resolves spontaneously.
196-Complete transaction of the oculomotor nerve results in : 1. Ptosis. 2. Convergent squint. 3. Constriction of the pupil. 4. Loss of accommodation and light reflex. 5. Diplopia.

Teaching Notes for Question 196 Theme: Oculomotor nerve. The nucleus of the third nerve lies ventral to the aqueduct in the midbrain. Efferent fibres to four external ocular muscles (superior, inferior and medial recti, and inferior oblique), levator palpebrae superioris and sphincter pupillae (parasympathetic) enter the orbit thro ugh the superior orbital fissure \9from kumar and clark). A complete oculomotor palsy results in the affected eye being displaced down and out with a dilated pupil.
197-The following involve deficiencies in T -cell immunity: 1. Di George syndrome. 2. Wiskott-Aldrich syndrome. 3. Chronic granulomatous disease. 4. Brutons agammaglobulinaemia. 5. Chronic mucocutaneous candidiasis.

Teaching Notes for Question 197 Theme: Deficiencies in T -cell immunity. Di George syndrome and chronic mucocotaneous candidiasis are predominantly T cell defects. The underlying defect in Di George syndrome is failure of normal development of the 3 rd and 4th pharyngeal pouches leading to aplasia or hypoplasia of the thymus and associated disorders. T -cell numbers are reduced.

119

PEDIATRICS EXAMINATION REVIEW

Chronic mucocutaneous candidiasis presents with persistant candidal infection of the nails, skin and mucu s membranes. Wiskott-Aldrich syndrome is associated with humoral and T-cell immunodeficiency. The defect is a deficiency in the wiskott -aldrich protein which play a role in cytoskeletal organization and possibly intracellular signaling. Microvilli on T -cell appear abnormal. Crdinal features are thrombocytopenia, eczema and immunodeficiency, Brutons agammaglobulinaemia is an X -linked recessive B-cell (humoral) defect due to mutation in the btk gene. This gene encodes a protein tyrosine kinase B-cell tyrosine kinase (Btk). Btk is necessary for transducing signals initiated by ligation of the pre-Breceptor. All Ig levels are low and mature B -cells and plasma cells are present. T -cell numbers and function are normal. Chronic granulomatous disease is due to defective neutrophil killing.
198-In young children. 1. Night terrors occur in the stages of deep sleep. 2. Sleep walking occurs during REM sleep. 3. Sleep disorders are never treated with drugs. 4. Breath holding attacks cause cyanosis. 5. Breath holding attacks should be investigated with CT scan.

Teaching Notes for Question 198 Theme: Sleep disorders. Night terrors: child wakes up, frightened and inaccessible, not responding when spoken to, may be difficult to comfort. Period of disturbed behaviour may last up to 15 minutes. Child eventually goes back to sleep. Occur in up to 3% of toddlers. Occur on waking from deep sleep , unlike nightmares that occur in REM or light sleep. Sleepwalking: Occur in a state of deep. May last for a few minutes to half an hour. Behavioural treatments best. Parental reassurance. Alternative of the parents responses to the childrens night time behaviour so as to prevent reinforcement of the problem. Drug treatments not thought to be particularly effective. Breath Holding spells: Common in preschool children. Usually before the age of 2. Precipitated by minor upsets. Follow crying which increases in intensity until breathing stops, usually in expiration, the child goes blue in the face especially around the face. Occasionally, there is a minor epileptic seizure. The child then starts to breath in half a minute or s o. Can be used by children to express anger towards parents. Reinforced when parents show concern.
199-The following maternal conditions are known to cause adverse effects on the neonate: 1. Diabetes insipidus. 2. Toxoplasmosis. 3. Chronic myeloid leukaemia. 4. SLE. 5. Hyperthyroidism.

Teaching Notes for Question 199 Theme: Maternal conditions causing adverse effects on the neonate.

120

PEDIATRICS EXAMINATION REVIEW

Maternal SLE is a risk factor for neonatal lupus syndrome. The neonate presents with clinical features of SLE due to transplacental passage of maternal Ab. The skin is frequently involved with malar rashes and there can be haematological and cardiac abnormalities. The most frequent heart abnormality is congenital heart block 90% of mothers whose infants have congenital heart block are anti Ro(SSA)Ab positive. Toxoplasmosis leads to congenital infection by transplacental t ransmission. The rate of transmission is +/ - 60% of third trimester infections and 20% -30% during the first two trimesters. The ewborn presents with the classic triad of hydrocephalus, chorioretinitis, and cerebral calcification. Maternal chronic myeloid l eukaemia can have adverse effect on pregnancy fetal/neonatal mortality is 16 -38%. Pathology can be secondary to placental leukaemic infiltrates, anaemia and infectious complications. Splenomegaly can restrict intrauterine growth and lead to premature de livery. If hyperthyroidism is due to Graves disease or Hashimoto thyroiditis, the neonate may present with thyrotoxicosis due to the transplacental passage of TSI. These symptoms are frequentely short -lived i.e as long as the circulating antibodies persist in the babys circulation. 200-en uptake by haemoglobin is characterized by: 1. Alkalosis. 2. Raised temperature. 3. Increased 2-3 DPG. 4. Increased red cell volume and MCH. 5. High altitude. Teaching Notes for Question 200 Theme: Oxygen uptake. 99% of the oxygen which dissolves in blood is transported by the oxygen carrying compound haemoglobin. Hae moglbin is composed of four globin subunits (in normal adults two alpha and ywo beta globin chains) each of which contains a haem group. Haem is a complex of prophyrin and ferrous iron. The oxygen haemoglobin dissociation curve relates the saturation of haem with oxygen as a % (the y axis) to the PO2 in mmHg(on the x axis). This curve is affected by a number of factors: 1- PH a fall in PH shifts the curve to the right i.e a higher PO2 is required for haemoglobin to bind oxygen. This is known as the Bohr ef fect. An increase in CO2 (as is found in peripheral tissues) will cause a fall in PH, and the associated decrease in affinity of haemoglobin for oxygen contributes to the transfer of oxygen from the blood in to the tissues. Alkalosis will have the opposite effect. 2- Temperature A rise in temperature shifts the curve to the right i.e oxygen uptake by haemoglobin is decreased. 3- 2,3-DPG levels- this substance is formed by glycolosis via the Embden Meyerhof pathway. It binds to the beta -globin chains of deoxyhaemoglobin and causes oxygen to dissociate from Hb. A rise in 2,3 -DPG shifts the oxygen dissociation curve to the right releasing more O2. 4- High altitude results in a rise in 2,3 -DPG concentrations in rbcs, with an increase in the availability of oxygen to th e tissues. The greater affinity of foetal haemoglobin for oxygen as compared to its adult counterpart is a result of the decreased binding of 2,3 -DPG by the foetal gamma globin chain. 5- With regards rbc indices, it is the MCHC which affects the oxygen dissociation curve, rather than the MCH or MCV. In anaemia, the MCHC is reduced, and the curve is right shifted, increasing the release of oxygen to the tissues.

121

PEDIATRICS EXAMINATION REVIEW

Questions
(PART 2)

SELECT ONLY ONE ANSWER

122

PEDIATRICS EXAMINATION REVIEW

1- A 14-year-old boy is rescued from a burning building and is brought to the hospital. He has nausea, vomiting, diarrhoea and abdominal pain. He is confused. The carboxyhaemoglobin
(COHb) level is 25%.

Which of the following is the best treatment for this patient? Options: A. Hyperbaric oxygen B. High flow oxygen via a facial mask C. CT brain and lumbar puncture D. Intravenous fluid and oxygen via nasal prongs E. Urgent senior surgical opinion to exclude bowel obstruction

2-Low protein C levels are not seen with: Options: A. Warfarin therapy B. Neonatal purpura fulminans C. Factor V Leiden D. Chicken pox E. Meningococcal septicaemia

3-Choose the correct matching pair of disease and antibody out of the following options:Options: A. Anti-liver/kidney microsomal antibodies (anti LKM-1) in type I autoimmune hepatitis B. Anti-smooth muscle antibodies in Primary biliary cirrhosis C. Anti-nuclear antibodies in Primary biliary cirrhosis D. Anti-mitochondrial antibodies in type II autoimmune hepatitis E. Anti liver cytosol antibodies in primary sclerosing cholangitis

4-hich one of the following cells in the lung parenchyma produces surfactant? Options: A. Alveolar macrophage B. Endothelial cell C. Goblet cell D. Type I pneumocyte E. Type II pneumocyte

5-Which of the following conditions with renal involvement is the most likely to have normal complement levels: Options:

123

PEDIATRICS EXAMINATION REVIEW

A. Grade IV lupus nephritis B. Post-streptococcal glomerulonephritis C. Subacute bacterial endocarditis D. Type II cryoglobulinaemia E. IgA nephropathy (Buergers disease)

6-Which of the following statements about the epidemiology of tropical diseases is correct? Options: A. Yellow fever is endemic in India and South -East Asia. B. Malaria is only endemic in the tropics. C. Chaga's disease is confined to South America. D. Visceral leishmaniasis is not found in the southern European countries. E. The distribution of infection wi th Schistosoma mansoni and Schistosoma haematobium are similar.

7-The best treatment option for a patient with Usual Interstitial Pneumonia (UIP or CFA (Cryptogenic fibrosisng alveolitis) is : Options:

A. Lung transplantation B. Interferon-1-Beta C. Azathioprine D. Cyclophophamide E. High -dose steroids

124

PEDIATRICS EXAMINATION REVIEW

8-What is the diagnosis that can you infer from the X-ray abdomen? Options: A. LBO (Large bowel obstruction) B. SBO (Small bowel obstruction) C. Toxic Megacolon D. Chagas Disease E. Ankylosing Spondylitis

9-A 6 month old boy with severe eczema presents with easy bru ising and two nose bleeds. He is below the 2nd centile for weight. He has had 3 previous admissions to hospital with pneumonia and has required two courses of antibiotics for otitis media. What is the most likely underlying diagnosis :Options: A. Primary antibody deficiency B. Ataxia telangiectasia C. Adenosine deaminase deficiency D. C4 deficiency E. Wiskott Aldrich syndrome

10-All of the following are type II hypersensitivities except: Options: A. Production of autoantibodies to acetylcholine receptors in myasthenia gravis B. Haemolysis following transfusion of blood bearing mismatched blood group antigens C. The Arthus reaction D. Hyperacute graft rejection E. Quinidine-induced thrombocytopenia

125

PEDIATRICS EXAMINATION REVIEW

11-Polycystic kidney disease is: Options A. Usually fatal at term B. Characterised histologically by tubule dysplasia and cystic tissue throughout the renal parenchyma C. Always clinically apparent at birth D. Characterised histologically by micro - and macroscopic cysts distributed throughout the renal parenchyma E. Kown to exist in 4 genetic forms

12-Which of the following statements regarding toxoplasmosis in pregnancy is false? Options A. Foetal abnormality may occur in cases of maternal infection occurring 6 months pre-conception. B. Treatment with pyrimethamine and sulphadiazine is recommended during the first trimester to reduce risk of foetal infection. C. Documented seroconversion confirms infection. D. Maternal toxoplasma-specific IgM may persist for over a year. E. Risk of fetal infection is increased with increasing gestational age.

13-A 9-year-old has a cranial CT scan. The radiologist reports the presence of intracranial calcification. Which of the following diagnoses is l east likely to be associated with the radiologists report? Options: A. Subdural haematoma B. Sturge-Weber syndrome C. Toxoplasmosis D. Ataxia telangiectasia E. Hypoparathyroidism

14-Choose the one correct statement regarding a Congenital diaphragmatic hernia:Options A. It is most commonly seen on the right side B. It is associated with pulmonary hypoplasia C. It is associated with oligohydramnios D. It is always symptomatic at birth E. It usually presents with vomiting

15-Regarding Histiocytosis X :Options: A. It is a cystic disease of lung, bone and anterior pituitary

126

PEDIATRICS EXAMINATION REVIEW

B. CXR shows small lungs with prominent markings and honeycombing C. 80% of patients are smokers; most are men 20 -40 years old D. Most important treatment strategy is steroids E. It is a rare cause of pneumothorax

16-A girl with an unusual facial appearance is found to have coarctation. Possible diagnoses are: Options: A. Noonan's syndrome B. William's syndrome C. Down's Syndrome D. Turner's syndrome E. Edward's syndrome

17-A study measures the heights of 13 -yr-old children. Which is the best conclusion to draw from the data? Options: A. The data are positively skewed B. The data are normally distributed C. The mean is the best measure of central tendency D. The median is greater than the mode E. The median and interquartile range is a good way of summarising the data

18-Which of the following hypotheses on the mechanisms of ageing is least accurate? Options: A. Ageing is caused by nitric oxide free radical-induced mitochondrial damage. B. Ageing is associated with altered insulin signalling. C. Ageing can be slowed in animal models by free radical scavengers. D. Ageing can be reduced in mammals by caloric restriction. E. Ageing is thought to be due to shortening of the telomeres.

127

PEDIATRICS EXAMINATION REVIEW

19-What is the diagnosis? Options: A. Left lower lobe consolidation B. Left lower lobe collapse C. Left lingular consolidation D. Left lingular collapse E. Right lower lobe consolidation

20-Which one of the following is true regarding cancer genetics?: Options: A. Any gene whose encoded protein directly or indirectly inhibits progression through the cell cycle and in which a loss-of-function mutation is oncogenic is an oncogene. B. RB, APC, and BRCA1 are oncogenes C. A gene whose product is involved either in transforming cells in culture or in inducing cancer in animals is called a proto-oncogene. D. A proto -oncogene is an abnormal gene that encodes a protein involved in regulation of cell growth or proliferation and that can be mutated into a cancer -promoting gene. E. A proto-oncogene is transformed onto an oncogene by changing the protein -coding segment or by altering its expression.

21-Which of the following renal disorders is associated with normal serum complement levels? Options: A. Membranoproliferative glomerulonephritis B. All forms of lupus nephritis C. Shunt nephritis D. Post-infectious glomerulonephritis E. Goodpastures disease

128

PEDIATRICS EXAMINATION REVIEW

22-What is the most likely explanation for these biochemical changes in a 15 -year-old girl with erosive changes in her mouth? Na 138 mmol/l K 3.2 mmol/l HCO3 36 mmol/l Ca 2.4 mmol/l P04 1.0 mmol/l Urine chloride 10 mmol/l Options: A. Primary hyperparathyroidism B. Primary hyperaldosteronism C. Persistent vomiting D. Anorexia nervosa E. Ehlers Danlos syndrome

23-A patient presents with acute infective arthritis. The resulting inflammation: Options: A. Causes the release of C8a, a potent chemokine B. Causes the release of CRP, the half -life of which is several days C. May be defined as increased vascular permeability accompanied by an infiltration of inflammatory cells D. Is always harmful E. Once initiated, it follows a stereotypical cascade, which is not regulated

24-What does this ECG of an 8-year-old boy with Ebsteins anomaly show? Options: A. Atrial flutter B. Supraventricular tachycardia C. Sinus tachycardia D. AV reentrant tachycardia due to accessory pathway E. Ectopic atrial tachycardia

129

PEDIATRICS EXAMINATION REVIEW

25-Which of the following conditions is not associated with hyperuricaemia : Options: A. Lesch-Nyhan syndrome B. Thiazide diuretic therapy C. Polycythaemia rubra vera D. Severe exfoliative psoriasis E. High-dose aspirin therapy

26-In a patient with hypomagnesaemia, the following statement is not true: Options: A. Recent cisplatin treatment could have been the cause B. Hypocalcaemia may occur C. Neuromuscular symptoms are similar to those in hypocalcaemia D. Prolonged vomiting is an indication for high -dose oral magnesium supplementation E. Hungry bone syndrome is associated with reduced serum magnesium

27-All these are major clinical features of neurofibromatosis -1 (NF-1) except: Options A. Axillary freckling B. Tibial pseudoarthrosis C. Cortical tubers D. Lisch nodules E. Caf au lait patches

28-Choose the one correct statement regarding necrotising enterocolitis: Options: A. It usually involves the lower colon B. It is commonly associated with gas in the biliary tree C. It is not seen in babies who haven't been started on feeds yet D. It may lead to intestinal obstruction a few weeks later E. It can be prevented by the use of oral antibiotics

29- Choose the correct statement regarding alpha 1 -antitrypsin (AAT): Options: A. It is a membrane bound tyrosine kinase

130

PEDIATRICS EXAMINATION REVIEW

B. Over 75% of cases of emphysema in the UK are associated with the deficiency of this enzyme C. Deficiency presents as cirrhosis in children D. The low levels of the enzyme in deficiency syndromes are due to lack of synthesis of the variant enzyme E. Emphysema in deficiency syndromes is characterised histologically by inclusion bodies

30-Which of the following is not a feature of the Holmes Adie syndrome? Options A. The affected pupil is always larger than the unaffected B. Both pupils may be affected C. There is denervation supersensitivity of the pupil D. Global areflexia is a recognised association E. Ptosis is rare

31-A 17 year old girl presents with primary amenorrhoea. She has recently started developing breasts. She is 145 cm tall and weighs 48 kg. Both her sisters (age 14 and 19) are greater then 170cm tall and are well. Hb 10.5 , ALP 320 MCV 108 Bili 12, WCC 7.6 , AST 34 Ca2+ 1.9 U&E normal, PO4 0.7 , TFTs normal What is the underlying diagnosis ? Options: A. Crohns disease B. Coeliac disease C. Cystic fibrosis D. Addisons disease E. Ulcerative colitis

32-Which of the following is not true of narcolepsy? Options: A. It is associated with cataplexy B. It exhibits X-linked inheritance pattern C. Night time insomnia can be a feature D. It is associated with early -onset REM sleep E. It may be associated with hypnopompic hallucinations

131

PEDIATRICS EXAMINATION REVIEW

33-The following are all true about migraine in children except: Options: A. More than 50% of headaches are unilateral B. EEG will help in the diagnosis C. Other members of the family may be affected D. An aura and postdrome stage is present as in adul t patients E. Sumatriptan is not licensed for use in children

34-The following statements about techniques using FISH (fluorescent in situ hybridisation) are true except: Options: A. The technique can be performed on uncultured cells B. The technique requires specialised microscopes C. The technique can detect small deletions in DNA D. The technique is used instead of light microscopy for routine chromosome analysis E. The technique can be used to map the position of genes on a chr omosome

35-Cardiac tamponade may occur with all of the following except: Options: A. Tuberculosis B. Rheumatic fever C. Pneumococcal pneumonia D. Rheumatoid arthritis E. Uraemia

36-The secondary immunoglobulin response is not c haracterized by:Options: A. Isotype switching to IgG B. Production of immunoglobulin with higher affinity to the antigen C. Production of memory plasma cells D. A quantitatively increased antibody production as compared to the primary immune r esponse E. Requirement of help from T-cells

132

PEDIATRICS EXAMINATION REVIEW

37-What is the abnormality seen in this girl with bone pain? Options: A. Subperiosteal resorption of HPTH (Hyperparathyroidism) B. HPOA (Hypertrophic Osteoarthropathy) C. AVN (Avascular necrosis) talus D. Myelomatous deposit E. Osteopetrosis

38-In the investigation of a patient who is admitted following 3 dizzy spells each markedly improved by eating:Options: A. Urine sulphonylurea levels are unhelpful B. A 72hr fast should be organised, with the doctor called when the blood sugar reaches 4mmol/l. C. Serum insulin, glucose and C-peptide should be taken every morning for 7 days D. Serum C -peptide will not be raised in exogenous insulin use E. A family history of scars in the neck is irrelevant

39-A 16-year-old girl presents with fever, joint aches and a red rash on her face, neck and hands. She attributed the rash to sev ere sunburn and says that she has sensitive skin. What are the most appropriate tests to request? Options: A. Anti nuclear antibody and c-reactive protein B. c-reactive protein and erythrocyte sedimentation rate C. Full blood count and chemistry D. Anti-nuclear antibody and erythrocyte sedimentation rate E. Complement levels

133

PEDIATRICS EXAMINATION REVIEW

40-Which of the following is true about prolactin? Options A. Elevated levels produce gynaecomastia without galactorrhoea B. Damage to the pituitary stalk reduces prolactin levels C. Metoclopramide increases prolactin production D. Levels are elevated in >90% of cases of acromegaly E. Levels are increased by cimetidine

41-Choose the incorrect statement. By the age of 9 months a child: Options A. Can sit with support B. Can crawl C. Can stand alone D. Can play peek -a-boo E. Has lost the Moro reflex

42-An 18-year-old man is referred to the outpatient clinic as his general practitioner has found him to have microscopic haematuria. H e also has bilateral sensorineural deafness with a sister and younger brother who are similarly afflicted. What is the most useful investigation? Options: A. Urine microscopy B. Serum creatinine C. Anti-GBM antibodies D. Audiometry E. Renal biopsy

43-The following statement is false regarding treatment of Juvenile Idiopathic Arthritis (JIA):Options A. Steroids are the agents of first choice. B. Hydroxychloroquine is associated with ocular toxicity. C. Iridocyclitis usually res ponds to topical steroids. D. Gold salts may lead to nephrotic syndrome. E. Low dose methotrexate may be beneficial

134

PEDIATRICS EXAMINATION REVIEW

44-What is the abnormality shown on Chest X -ray? Options: A. Consolidation right lower lobe B. Collapse right lower lobe C. Collapse lingular lobe D. Perforation E. Bilateral pneumothoraces

135

PEDIATRICS EXAMINATION REVIEW

45-What is the diagnosis? Options: A. Proliferative DM retinopathy B. Grade 3 hypertensive retinopathy C. Endocarditis D. CMV retinitis E. Background retinopathy

46-All of the following statements about Glu cose-6-phosphate dehydrogenase deficiency are false except: Options: A. Is more common in women than in men. B. Is caused by mutations of the G6PD gene on chromosome 4. C. Often leads to chronic haemolysis. D. Requires treatment with desferrioxam ine to prevent iron overload. E. Is associated with haemolytic crises induced by sulfonamides.

47-Which of the following statements correctly describes the clinical phenotypes which are associated with a deficiency of the various complement compon ents? Options: A. C5-C9 deficiency and mycoplasma infection B. C1q deficiency and angio -oedema C. C2 deficiency and rheumatoid arthritis D. C3 def is associated with partial lipodystrophy E. A raised C4 and low C3 in SLE is associated with acti ve disease

136

PEDIATRICS EXAMINATION REVIEW

48-The following autoantibodies are commonly found in the named disease: Options A. Anti-liver/kidney microsomal antibodies (anti LKM-1) in type I autoimmune hepatitis B. Anti-phospholipid antibodies in Primary biliary cirrhosis C. Anti-mitochondrial antibodies in Primary biliary cirrhosis D. ANCA in type II autoimmune hepatitis E. Anti-liver cytosol antibodies in primary sclerosing cholangitis

49-All of the following are clinical features of severe aortic stenosis exce pt: Options: A. Fourth heart sound B. Third heart sound C. Soft aortic component of second heart sound D. Systolic thrill E. Length of murmur in systole

50-A two-year-old boy is brought to the resuscitation room. He has been unwell for 1 2 hours with fever and has vomited once. On examination, his airway is patent and he is breathing oxygen via a mask with non -rebreathing bag with oxygen saturations of 100%. His heart rate is 160/min, capillary refill time is 5 seconds and he has a purpuri c rash over his lower limbs. He is not responsive to pain. A colleague has been trying for 5 minutes to secure intravenous access when you arrive. The next step should be: Options: A. Continue to try for IV access and then give 20mL/kg fluid if success ful B. Check blood sugar C. Secure intraosseus access and give 20mL/kg fluid D. Give immediate IM ceftriaxone before securing IV access E. Check a capillary blood gas

51-A patient is noted to have a small left pupil. There is also a ptosis p resent in this eye. His mother has noted that the left side of his face is dryer than normal. Where is the lesion likely to be? Options A. Oculomotor nerve B. Trigeminal nerve C. Facial nerve D. Vagus nerve E. Cervical sympathetic trunk

137

PEDIATRICS EXAMINATION REVIEW

52-A patient is diagnosed as having a Horner's syndrome following carotid surgery. Which of the following is NOT a symptom of Horners syndrome? Options: A. Pupillary constriction B. Ptosis C. Sinking in of one eye D. Absence of sweating on face and neck E. Lack of lacrimation

53-Which of the following sub-group of populations is the influenza vaccine not recommended for?:Options A. Immunosupressed people due to disease or treatment B. Those with chronic heart disease C. Those with chronic renal failure D. Those with diabetes mellitus E. Those with coeliac disease

54-The lipid-lowering effect of a new statin is compared to atorvastatin. Critical level is set at 0.01 and difference is found (p = 0.001). What is the best conclusion to draw? Options A. A larger sample should have been used B. The alternative hypothesis should be accepted C. The probability of a type I error occuring is 1% D. The p-value associated with this statin means that it is a clinic ally significant new drug E. The trial would have been better run against placebo

55-Which of the following is correct regarding atrial flutter? Options: A. Is usually left atrial in origin B. Usually causes a narrow complex tachycardia at 150 bpm C. Is often associated with an irregular pulse D. Can be terminated with intravenous adenosine E. Implies the presence an accessory pathway

138

PEDIATRICS EXAMINATION REVIEW

56-The following statements are true about Tetralogy of Fallot (TOF) except: Options : A. Children with TOF are susceptible to cerebral abscess B. The chest X-ray shows oligaemic lung fields C. The second heart sound is soft and single D. TOF is associated with dextroposition of the aorta E. This disorder results in a left to rig ht shunt

57-In terms of the NICE guideline of screening for diabetic nephropathy in Type 2 patients: Options: A. The target HbA1c is about 8%. B. A renal consultation is advised if the serum creatinine rises above 250 mol/l. C. All patients with confirmed microalbuminuria, if no contraindications, should be on an ACE inhibitor. D. Blood pressure is less important than reducing the degree of proteinuria. E. Calculating ratios of albumin and creatinine in the urine is not as useful as the total urine protein level.

58-What can you infer from the Chest X-ray? Options: A. Left pneumothorax B. Left lower lobe collapse C. Left lingula collapse D. Left tension pneumothorax E. Left hilar adenopathy

139

PEDIATRICS EXAMINATION REVIEW

59-Which of the organs below is considered an immunologically privileged site : Options: A. Skin B. Thymus C. Bone marrow D. Testicle E. Pancreas

60-Following a dislocated shoulder in a rugby match that gets out of hand, a patients axillary nerve is damaged. What is the likely result? Options: A. The teres major atrophies. B. The rounded contour of the shoulder disappears. C. A loss of sensation may occur in the lateral forearm. D. The patient may lose the ability to adduct the arm. E. The patient may have a wrist drop.

61-Steve, a 10-year-old boy presents with a chronic cough, recurrent lung infections and clinical evidence of obstructive pulmo nary disease. His mother reveals that when he was 2 years old he was diagnosed as having malabsorption, and she has noticed that his sweat tastes excessively salty. All the above are features of this condition EXCEPT: Options: A. 510% of cases manifest with meconium ileus at birth B. In 70% of patients, the genetic abnormality is a four -base pair deletion at the locus on chromosome 7 C. Pancreatic abnormalities are present in 80% of patients D. The primary defect is in the transport of chlor ide ions across epithelial tissue E. Staphylococcus aureus and pseudomonas aeruginosa are two pathogens most commonly isolated from the sputum of patients with this condition

140

PEDIATRICS EXAMINATION REVIEW

62-A 26-year-old airhostess is brought to the A&E depar tment following a witnessed grand mal seizure. She has been previously fit and well except for a history of mild intermittent arthralgia affecting both hands. Her only medication is the OCP. She smokes 20 cigarettes per day and drinks 15 units of alcohol per week. * On examination she has no focal neurology. * There is slight puffiness of her fingers and she has low -grade pyrexia of 37.6C. * BP 160/98 mmHg. * Fundoscopy reveals silver wiring and AV nipping bilaterally. * Urinalysis tests 3+, blood 2+ proteins. Her blood test results reveal: Hb MCV WCC Platelets CRP ESR Sodium Potassium Urea Creatinine Albumin Total protein LFTs BM Clotting screen: PTT APTT VDRL Drug screen of urine Plasma alcohol 9.9 g/dl 83 fl 3.1 x 109/l 117 x 109/l 2 g/l 71 mm in the first hour 134 mmol/l 5.1 mmol/l 10.1 mmol/l 178 mmol/l 30 g/l 80 g/l Normal 7.8 mmol

11 seconds 39 seconds Positive 1:320 Negative Not detected

What is the likely diagnosis? Options: A. SLE B. Epilepsy C. Pulmonary embolism D. Syphilis E. Systemic sclerosis

63-An elderly man presents to casualty with a wrist drop and pain radiating down his right arm. He recently visited his GP for a deep IM steroid injection. The probable site of the injection was the: Options: A. Posterior cord of the brachial plexus. B. The medial cord of the brachial plexus. C. The long thoracic nerve. D. The C5 spinal root. E. The ulnar nerve.

141

PEDIATRICS EXAMINATION REVIEW

64-A boy presents with fever, arthralgia and headache and a red, annular rash in his axilla, which his mother says has been present for one week and has increased in size. Which is the most likely diagnosis? Options: A. Kawasaki disease B. Infectious mononucleosis C. Scarlet fever D. Lyme disease E. Rheumatic fever

65-Milkmaid grasp is characteristically seen in which of the following conditions? Options A. Hemiballismus B. Sydenhams chorea C. Huntingtons disease D. Torticollis E. Gilles de la Tourette syndrome

66-A 2-year old boy is admitted wit h fever. The presence of jaundice would make which one of the following diagnoses more likely? Options: A. Typhoid B. Malaria C. Toxocariasis D. Lyme disease E. Cat scratch fever

67-A two-month-old child had his first DTP injection 3 days previously and his mother has returned because his thigh is swollen. On examination, most of the anterolateral surface of the thigh is red and indurated. What will you recommend for this child in the future? Options A. He should not have any further immunisations B. He should have all immunisations except pertussis C. He should receive the acellular pertussis vaccine when next immunised D. He should receive the oral pertussis vaccine when next immunised E. He should receive the whole cell pertussis vaccine (as DTP) as usual at his next immunisation

142

PEDIATRICS EXAMINATION REVIEW

68-You see a 15-year-old girl in clinic with the following results: * Hepatitis B surface antigen: positive * Antibodies to hepatitis B surface antigen: negative * Hepatitis B 'e' antigen: negative * Antibodies to hepatitis B core antigen: IgG positive * Antibodies to hepatitis B 'e' antigen: positive Which of the following statements best describes her clinical condition? Options: A. Acute hepatitis B infection B. Past hepatitis B infection C. Chronic hepatitis B infection (low infectivity) D. Chronic hepatitis B infection (high infectivity) E. Previously vaccinated against hepatitis B

69-In the syndrome of inappropriate ADH secretion (SIADH): Options: A. Plasma sodium is usually normal B. Plasma urate levels are elevated C. Oedema may occur due to fluid overload D. Urine osmolality is >500 mosm/kg E. Serum globulins are high

70-A 14 year-old boy is seen in the Ambulance and Emergency Department. He is unconscious and he has a rash around his mouth. The following blood test results are found. pH 7.29 , Pco2 4.5, HCO3 15, Cl 110, K 5, Na 135. What has he taken? Options: A. Methanol B. Ethylene glycol C. Opiates D. Glue E. MDMA

71-The most important cause of death in infectious mononucleosis is: Options: A. Liver failure. B. Kidney failure. C. Heart failure. D. Respiratory failure. E. Splenic rupture.

143

PEDIATRICS EXAMINATION REVIEW

72-Acute attacks of porphyria: Options: A. Can be precipitated by paracetamol B. Can produce hyponatraemia due to Diabetes Insipidus C. Are a feature of porphyria cutanea tarda D. Are precipitated by the contraceptive pill E. Are associated with malabsorption

73-A 17-yr-old male medical stu dent presents with a sudden onset of tunnel vision. On examination, he reports having only central vision over an area of approximately 1 m by 1 m. He reports this at a distance of 3 m and also at 6 m. Which of the following diagnoses best fits his sympt oms? Options: A. Optic neuritis B. Anterior uveitis C. Methanol poisoning D. Non -organic symptoms E. Retinitis pigmentosa

74-A middle aged lady presents to Casualty with a wrist drop and pain radiating down her right arm after sustaining an injury to her neck, right arm and shoulder during a game of tennis. The probable site of the injury was the: Options: A. Posterior cord of the brachial plexus B. Medial cord of the brachial plexus C. T1 spinal root D. C5 spinal root E. Ulnar nerve

144

PEDIATRICS EXAMINATION REVIEW

75-A 26-year-old air hostess is brought to A&E following a witnessed grand mal seizure. She has been previously fit and well except for a history of mild intermittent arthralgia affecting both hands. Her only medication is the OCP, she smokes 20/day and drinks 15 units of alcohol per week. On examination she has no focal neurology. There is slight puffiness of the fingers and low -grade pyrexia of 37.6C. BP 160/98 mmHg. Fundoscopy reveals silver wiring and AV nipping b ilaterally. Urine analysis tests show 3+ blood, 2+ protein. Results: Hb MCV WCC platelets CRP ESR Sodium potassium urea creatinine Albumin total protein LFTs BM Clotting screen: PTT APTT VDRL Drug screen of urine Plasma alcohol 11 seconds 39 seconds positive 1:320 negative not detected 9.9 g/dl 83 fl 3.1 X 109/l 117 X 109/l 2 g/l 71 mm in the first hour 134 mmol/l 5.1 mmol/l 10.1 mmol/l 178 mmol/l 30 g/l 80 g/l normal 7.8 mmol

The patient then develops acute dyspnoea and examination reveals decreased air entry at the right base, tachycardia, BP 105/55. O2 sats 94% on room air. She responds well to prednisolone and azathioprine, but at an OPD appointment 3 months later is complaining of worsening right hip pain poorly res ponsive to NSAIDs. X-ray is normal but bone scan shows increased uptake in the hip. What is the most likely cause of her hip pain? Options: A. Osteoporosis B. Avascular necrosis C. Septic arthritis D. Stress fracture E. Trochanteric bursitis

76-A 35-year-old woman who complained of easy bruising, weight gain and striae is being worked up for Cushings syndrome. Her 24 -hour urinary free cortisol is 450 nmol. Which is the next investigation to confirm the diagnosis? Options: A. Low-dose dexamethasone suppression test B. High-dose dexamethasone suppression test C. Measuring cortisol levels D. Measuring ACTH levels E. CRH stimulation test

145

PEDIATRICS EXAMINATION REVIEW

77-A 26-year-old woman is admitted to hospital for cadaveric renal transplantation. Th e operation is uneventful however she is admitted to the Intensive Therapy unit for 48 hours postoperatively. She is given treatment with prednisolone and cyclosporin, however 10 days after admission she develops a fever, diarrhoea, headaches and a non -productive cough. Examination reveals no focal signs of infection. Blood cultures are taken and a CXR is unremarkable. She is treated with cefotaxime, however after 48 hours the fevers continue and a repeat CXR shows left lower lobe consolidation. Options: A. Streptococcus pneumoniae B. Klebsiella pneumoniae C. Pneumocystis carinii D. Cytomegalovirus E. Cryptococcus neoformans

78-A 6 week old male baby presents with a 2 day history of multiple seizures. He has been vomiting non-billious breast milk for the last 3 weeks. On examination the child is afebrile and dehydrated with a depressed anterior fontanelle. Ultrasound of the abdomen shows hypertrophic pyloric stenosis. The intravenous (iv) fluid of choice for administration would be: Options: A. iv 50% calcium gluconate B. iv isolyte P C. iv 5% dextrose D. iv sodium bicarbonate E. iv Hypertonic saline

79-Regarding Legionnaire's disease:Options: A. Strict barrier nursing is essential to contain the infectious agent B. Preexisting immunocompromise is a risk factor C. Rifampicin alone is as effective as erythromycin for the treatment D. Pontiac fever is a rarer and milder form of the disease E. Diagnosis can be made from stool culture

80-A 15 year old boy is admitted with pneumonia. Investigations show hyponatraemia and mildly deranged LFTs. CXR shows hazy shadowing in the right mid and lower zones. Therapy is started and 5 days later he becomes acutely jaundiced with red discolouration of the urine. Which of the following drugs is the likely cause :Options: A. Cefotaxime B. Penicillin C. Rifampicin D. Ciprofloxacin E. Tetracycline

146

PEDIATRICS EXAMINATION REVIEW

81-Seven weeks after returning from rural Ghana, a 16 -year-old boy develops fever and rigors. The least usefu l test is: Options: A. Thick and thin blood film B. IgM for viral haemorrhagic fever C. Chest radiograph D. Blood cultures E. HIV antibody test

82-A 15-year-old is undergoing investigation for hirsuitism. Which of the following would not be implicated in the aetiology? Options: A. Cyclosporin A. B. Phenytoin. C. Moxonidine. D. Congenital adrenal hyperplasia. E. Cushing's syndrome.

83-A family attends genetic counselling. They have one child affected with Downs syndrom e and are considering another pregnancy. Which of the following statements is most accurate? Options: A. Both parents should have karyotype analysis, but there is no need to karyotye the affected child B. Most children with Down's syndrome are born to women over the age of 35 years C. Amniocentesis should only be offered to confirm the diagnosis of Downs syndrome if the family will consider termination of the pregnancy D. A karyotype of 46XX excludes a diagnosis of Downs syndrome E. A nuchal scan together with bHCG and PAP (pregnancy associated protein) between 11 and 13 weeks of pregnancy gives an accurate risk of Downs syndrome in >80% of cases

84-A 17 year old student develops a deterioration in vision in her left eye over three day s. She complains of discomfort in the eye and thinks that difficulty with perception of colour was the first problem that she noticed, during a trip to an art gallery. On examination, visual acuity on the left is down to light perception. The pupil appears dilated and does not constrict to light, although does when a torch is shone in the right eye. What is the most likely diagnosis : Options: A. CMV retinitis B. Optic neuritis C. Anterior iscaemic optic neuropathy D. Amaurosis Fugax E. Central retinal artery occlusion

147

PEDIATRICS EXAMINATION REVIEW

85-A 29 year old Cypriot man gives a history of recurrent pain and swelling of the left knee. He also describes recurrent episodes of fever, pleuritic pain, and rash. He is then hospitalised with severe headache, photophob ia and neck stiffness. He is lucid with a GCS of 15. His blood test results show: Hb 13.8g/dl plats 210 X 109/l WCC 4.9 X 109/l plasma glucose 6.5 mmol/l CRP 39g/l ESR 56 mm in the first hour RF, ANA, ENA and ANCA negative CSF analysis: opening pressure 14cmH2O protein 0.6 g/l RBC 2 /ml WBC 21/ml (100% lymphocytes) CSF glucose 4.1 mmol/l no organisms seen on microscopy 48 hour culture of CSF negative What is the diagnosis? Options: A. Familial Mediterranean fever B. Adult onset Still's disease C. Behcets syndrome D. Familial Hibernian fever E. Non Hodgkins lymphoma

86-Which of the following do not tend to make the symptoms of myasthenia gravis worse? Options: A. Prednisolone B. Aminoglycosides C. Pyridostigmine D. Lithium E. Quinidine

148

PEDIATRICS EXAMINATION REVIEW

87-This 15-year-old was cyanosed on exertion. What is the diagnosis? Options: A. Pulmonary thromboembolic disease B. Fallots tetralogy C. Transposition of great arteries D. Eisenmengers syndrome E. Mitral stenosis

88-A patient has hypothyroidism. She develops a painful hand at night. In view of the probable aetiology, which muscle would you look for w asting? Options: A. Lateral two interossei B. Abductor pollicis brevis C. Medial two lumbricals D. Flexor digiti minimi E. Extensor pollicis

89-In a patient that has been previously diagnosed with Wilsons disease, which of the following statements is true : Options: A. Serum cerruloplasmin levels are raised B. When penicillamine is administrated then urinary copper level rise C. Is inherited as an autosomial dominant trait D. May result in retinal abnormalities E. Most common presentation is with a manic depressive illness

149

PEDIATRICS EXAMINATION REVIEW

90-Which of the following statement(s) regarding childhood minimal -change nephrotic syndrome is/are true?: Options; A. On immunofluorescent studies, deposition is seen along the glomerular baseme nt membrane B. Hypervolaemia is a common problem C. Albumin is routinely given to children with proteinuria D. Long-term cyclophosphamide therapy is beneficial in the steroid resistant group E. Minimal change disease is a histological diagnosis

91-A child has had his chromosomes analysed in the course of various investigations and the result is: 46 XY, t (2;5)(q35;p21.3}. Which of the following statements is correct?: Options: A. He has more than 46 chromosomes B. The result shows all his genetic defects C. There is a translocation between the short arm of chromosome 2 and the short arm of chromosome 5 D. He is likely to be infertile E. There is an increased risk of him having a child with difficulties

92-In leptospirosis which one of the following is true? Options: A. It is usually transmitted by rat bite B. It is a notifiable disease C. The patient may be jaundiced during the immune phase D. The organism may be isolated from urine during the septic phase E. Ciprofloxacin is the treatment of choice for severe forms

93-A 9 months old baby girl is brought to you with failure to thrive and rickets. Her parents tell you that she has always been a very thirsty child, drinking 3 -4 bottles of water through the night, and having soaking wet napping, urine dipstick shows 1+ glucose, and 1+protein. Which of the following features are you likkely to find on examination and investigation. Options: A. Cataract. B. Corneal opacities. C. Hyperkalaemia. D. Metabolic acidosis with a wide anion gap. E. Nephrocalcinosis.

150

PEDIATRICS EXAMINATION REVIEW

94-A 5 years old presents to casuality with a 2 months history of inguinal lymphadenopathy, his full blood count is normal , but there are small blasts in the blood film, a bone marrow aspirate confirm that he has acute lymphoblastic leukaemia. Which is the most appropriate initial course of action ? Options: A. Commonce chemotherapy straight away. B. Do a lymph node biopsy to exclude concomitant lymphoma. C. Referre him for a bone marrow transplantation. D. Start IV fluids & allopurinol. E. Discharge him and review in 1 week time as an out patient.

95- Which of the following is not a recognised complication of anticoagulation therapy? Options: A. Deep venous thrombosis. B. Thrombocytopaenia. C. Skin necrosis. D. Osteoporosis. E. Peripheral neuropathy.

96-The mother of a baby in the postnatal ward has an antenatal screening test for sickle haemoglobin. This is positive deposite a norm al Hb & MCV. The Hb-electrophoresis shows HbA 55% and Hbs 45%. Her partner has a microcytic anaemia, and Hb -electrophoresis shows an HbA band only with HbA2 quantitation of 4.8%. Which diagnosis is impossible in the baby? Options: A. Sickle cell trait. B. Haematologically normal. C. B-thalassaemia trait. D. Sickle-B-thalassaemia. E. Sickle cell diseas(HbSS).

151

PEDIATRICS EXAMINATION REVIEW

97-A 9 years old male has always wet the bed, he has no daytime enuresis, which one of the following statement(s) regarding enuresis is not true? Options: A. Runs in families. B. There is a 30% chance that his father was a bed-wetter. C. Drinking tea causes bed-wetting D. At 15 years age, there is a 1% chance that he will still bed -wet. E. If a child has tried many treatments over a number of years the prognosis is poor.

98-A 18 years old student becomes un well with a sore throat, low grade fever and has some cervical lymphadenopathy, on consulting his GP 3 days after the onset of this illness, exa mination is otherwise unremarkable, investigations reveals: FBC normal ,Na=137 K=4.4 urea=10.1 ,creatinin =141 ,C3= 1.31g/l (0.55 1.20)C4= 0.42g/l (0.20-0.50),CH50= 87% (50-110) Urinalysis: blood 3+, protein +, few red cell casts. What is the most likely diagnosis? Options: A. post-streptococcal glomerulonephritis. B. Wegeners granulomatosis. C. Ig A nephropathy. D. Interstitial nephrosis. E. Henoch-Schonlein purpura.

99-An adolescent male is admitted with progressive limb weakness, on examination he has flacid weakness predominantly in the legs and absent reflexes. CSF examination shows markedly raised protein & a mild lymphocytosis, Which of the following does not imply a worse prognosis? Options: A. History of diarrhea in the preceding few weeks. B. presence of postural hypotension. C. Markedly reduced FVC. D. Evidence demyelination on nerve conduction studies. E. Sever weakness.

100-Only one of the following is correct regarding physiology of the colon: Options: A. It is the site of most vitamin C absorption. B. Na and Cl are actively secreted. C. Mucous, K and bicarbonate are absorbed. D.Stomach distension increases colonic motility. E. Increased parasympathetic activity leads to decreased colonic motility.

152

PEDIATRICS EXAMINATION REVIEW

Answer &Teaching Notes for Questions


(PART 2)

1- A 14-year-old boy is rescued from a burning building and is brought to the hospital. He

153

PEDIATRICS EXAMINATION REVIEW

has nausea, vomiting, diarrhoea and abdominal pain. He is confused. The carboxyhaemoglobin
(COHb) level is 25%.

Which of the following is the best treatment for this patient? Options: A. Hyperbaric oxygen B. High flow oxygen via a facial mask C. CT brain and lumbar puncture D. Intravenous fluid and oxygen via nasal prongs E. Urgent senior surgical opinion to exclude bowel obstruction

A
Teaching Notes for Question 1 Theme: Carbon monoxide poisoning Carboxyhaemoglobin (COHb) levels do not correlate well with the clinical picture. The indications for hyperbaric oxygen are neurological or psychiatric symptoms, cardiac complications, COHb levels of >40% and pregnant women 2-Low protein C levels are not seen with: Options: A. Warfarin therapy B. Neonatal purpura fulminans C. Factor V Leiden D. Chicken pox E. Meningococcal septicaemia

C
Teaching Notes for Question 2 Theme: Low protein C levels
Protein C is a vitamin K -dependent protein synthesised in the liver. It circulates in an inactive form, which becomes activated by thrombin/thrombomodulin complexes on the surface of endothelial cells. Activated protein C inhibits coagulation by degrading coagulation factors Va and VIIIa and resulting in decreased thrombin formation. It requires protein S as a cofactor.

Causes of protein C deficiency are: 1) Congenital


* homozygous deficiency - those with severe deficiency - (protein C levels <1% of normal) present with severe neonatal purpura fulminans, cerebral thrombosis and DIC. Those with milder defects have protein C le vels of 10 -24 % of normal. They can present with massive venous thrombosis as older children. * heterozygous deficiency (protein C levels 30 -40% of normal) most frequently presents after puberty with deep venous thrombosis of the lower limb. It can also present with recurrent superficial thrombophlebitis and DVT. Most thrombotic episodes occur spontaneously but known associated risk factors include surgery with immobilisation, pregnancy and the oral contraceptive. Many people remain asymptomatic. A posit ive family history of thrombosis is associated with an increased risk of symptoms - + - 50% of individuals with heterozygous protein C deficiency and a family history of thrombosis, will experience a thrombotic event.

2) Causes of acquired protein C defic iency: * infections - meningococcal disease and varicella infection
* DIC * hepatic disease * drugs - warfarin, chemotherapeutic agents e.g. cyclophosphamide, methotrexate, 5 -fluorouracil), lasparaginase * malignancy - acute myeloid leukaemia * chronic inflammation - inflammatory bowel disease * haemolytic uraemic syndrome * thrombotic thrombocytopaenic purpura Patients with protein C deficiency are at high risk for warfarin -induced skin necrosis during initiation of therapy with warfarin. Factor V Leiden refers to a point mutation in the gene for factor V results in resistance of factor V to the actions of activated protein C, and consequently a thrombophillic state. Protein C levels are not decreased

154

PEDIATRICS EXAMINATION REVIEW

3-Choose the correct matching pair of dis ease and antibody out of the following options: Options: A. Anti-liver/kidney microsomal antibodies (anti LKM-1) in type I autoimmune hepatitis B. Anti-smooth muscle antibodies in Primary biliary cirrhosis C. Anti-nuclear antibodies in Primary biliary cirrhosis D. Anti-mitochondrial antibodies in type II autoimmune hepatitis E. Anti liver cytosol antibodies in primary sclerosing cholangitis

C
Teaching Notes for Question 3 Theme: Antibodies in GI diseases Autoimmune hepatitis type I - Anti nuclear antibodies, Anti smooth muscle antibodies, Anti mitochondrial antibodies type II - Anti liver/kidney microsomal antibodies (LKM -1) Anti liver cytosol Primary Biliary Cirrhosis - Anti nuclear antibodies, Anti mitochondrial (M2) antibodies Primary Sclerosing cholangitis - ANCA positive . 4-hich one of the following cells in the lung parenchyma produces surfactant? Options: A. Alveolar macrophage B. Endothelial cell C. Goblet cell D. Type I pneumocyte E. Type II pneumocyte

E
Teaching Notes for Question 4 Theme: Surfactant production Type II pneumocytes produces surfactant in the lungs. Type I pneumocytes, which are as many as type II pneumocytes, cover 95% of the epithelial surface area. Its key function is gaseous exchange.

5-Which of the following conditions with renal involvement is the most likely to have normal complement levels: Options: A. Grade IV lupus nephritis B. Post-streptococcal glomerulonephritis C. Subacute bacterial endocarditis D. Type II cryoglobulinaemia E. IgA nephropathy (Buergers disease)

E
Teaching Notes for Question 5 Theme: Renal disease and complement levels Primary membranous nephropathy and Buergers disease can have normal complement levels. The other conditions are all associated with activation of the classical complement pathway and immune complex formation .

155

PEDIATRICS EXAMINATION REVIEW

6-Which of the following statements about the epidemiology of tropical diseases is correct? Options: A. Yellow fever is endemic in India and South -East Asia. B. Malaria is only endemic in the tropics. C. Chaga's disease is confined to South America. D. Visceral leishmaniasis is not found in the southern European countries. E. The distribution of infection with Schistosoma mansoni and Schistosoma haemato bium are similar.

Teaching Notes for Question 6 Theme: Epidemiology of tropical diseases Yellow fever occurs in Africa and South America only . Malaria is endemic in large a reas of the south shore of the Mediterranean, Arabia, Northern India and South East Asia , which lie north of the Tropic of Cancer. Chagas' disease is found mainly in South America . Visceral leishmaniasis is endemic in parts of Spain , Italy and Greece. Schistosoma haematobium and S .mansoni are both endemic in Africa , but the latter is also found in South America .
7-The best treatment option for a patient with Usual Interstitial Pneumonia (UIP or CFA (Cryptogenic fibrosisng alveolitis) is :Options:

A. Lung transplantation B. Interferon-1-Beta C. Azathioprine D. Cyclophophamide E. High -dose steroids

Teaching Notes for Question 7 Theme: Usual Interstitial Pneumonia (UIP or CFA ) At present the treatments for UIP have limited success and most patients will die from their disease. Single lung transplantation for UIP results in an actuarial survival of 80% at one year and 5 5% at three years.

156

PEDIATRICS EXAMINATION REVIEW

8-What is the diagnosis that can you infer from the X -ray abdomen? Options: A. LBO (Large bowel obstruction) B. SBO (Small bowel obstruction) C. Toxic Megacolon D. Chagas Disease E. Ankylosing Spondylitis

Teaching Notes for Question 8 Theme: Toxic megacolon Large bowel thick walled and di lated, at risk of perforation Causes: * IBD * Ischaemia * Infection (clostridium)
9-A 6 month old boy with severe eczema presents with easy bruising and two nose bleeds. He is below the 2nd centile for weight. He has had 3 previous admissions to hosp ital with pneumonia and has required two courses of antibiotics for otitis media. What is the most likely underlying diagnosis :Options: A. Primary antibody deficiency B. Ataxia telangiectasia C. Adenosine deaminase deficiency D. C4 deficiency E. Wiskott Aldrich syndrome

Teaching Notes for Question 9 Theme: Severe eczema and bruising Wiskott Aldrich syndrome is an X-linked combined immunodeficiency caused by a deficiency of WASP (Wiskott Aldrich syndrome associated protein). This is a cytoskeletal associated protein which is required for B - and T- cell interaction and platelet function. In vitro studies with T-cells, platelets, and phagocytes of WAS patients show defects in the

157

PEDIATRICS EXAMINATION REVIEW formation of microvilli and phagocytic vacuoles - structures which are affected by actin reorganisation. There is also reduced expression of CD43 (sialophorin) on the surface of B and T-lymphocytes. Patients present with eczema, thrombocyt opaenia and recurrent bacterial infections. They also have an increased risk of lymphoid malignancy. Initially lymphocyte numbers are normal, and the major defect is impaired production of antibody to thymus independent polysaccharide antigen. Later lympho cyte numbers decrease, and immunodeficiency becomes more severe. Patients have recurrent respiratory infections and are susceptible to organisms such as Pneumocystis carinii and herpes simplex virus. There is impaired cell mediated immunity (with lymphopen ia) and a characteristic pattern of immunoglobulin levels:- low IgM, normal levels of IgG and high levels of IgA and IgE. Platelet numbers are decreased and platelets are smaller than usual. These children can present with life threatening bleeding. Bone marrow transplant is curative. Ataxia telangiectasia is caused by defective DNA repair mechanisms. Patients present with progressive neurological deterioration, ataxia, malignant disease, impaired cell mediated immunity and defective antibody production. Adenosine deaminase defiency causes about 20% of cases of SCID. 10-All of the following are type II hypersensitivities except: Options: A. Production of autoantibodies to acetylcholine receptors in myasthenia gravis B. Haemolysis following transfusion of blood bearing mismatched blood group antigens C. The Arthus reaction D. Hyperacute graft rejection E. Quinidine-induced thrombocytopenia

C
Teaching Notes for Question 10 Theme: Type II hypersensitivities. Myasthenia gravis, autoimmune haemolysis, drug-induced thrombocytopenia and hyperacute graft rejection are all examples of type II hypersensitivity reactions. Type II hypersensitivity reactions involve antibody-mediated destruction of cells. Antibody - usually IgG or IgM can destroy ce lls by activating complement, or promoting phagocytosis of antibody-coated cells. They can also bind to cell surface receptors interfering with normal function. An example of antibody interfering with normal receptor function occurs in myasthenia gravis. In this disease, autoantibodies bind to acetylcholine receptors at the neuromuscular junction. This interferes with normal motor end plate function and also results in a reduction in receptor numbers. Blood transfusion reactions exemplify type II hypersen sitivity. Host antibodies (usually IgM) react with foreign antigens on the incompatible transfused blood cells and lead to complement-mediated haemolysis of these cells. The Arthus reaction is a type III hypersensitivity reaction mediated by the local dep osition of immune complexes with consequent complement activation and neutrophil infiltration. Hyperacute rejection is usually due to preformed anti -donor antibody. These antibodies are directed against foreign MHC molecules of other alloantigens 11-Polycystic kidney disease is: Options A. Usually fatal at term B. Characterised histologically by tubule dysplasia and cystic tissue throughout the renal parenchyma C. Always clinically apparent at birth D. Characterised histologically by micro - and macroscopic cysts distributed throughout the renal parenchyma E. Kown to exist in 4 genetic forms

158

PEDIATRICS EXAMINATION REVIEW Teaching Notes for Question 11 Theme: Polycystic kidney disease * This c ondition is generally classified as autosomal dominant or recessive, the recessive form not being apparent often until adulthood. * Histologically it is characterised by the presence of cysts, but not renal dysplasia. * Neither type is always obvious at birth, and nor is either form universally fatal, today in the absence of severe pulmonary hypoplasia affected infants can be expected to survive beyond the neonatal period. 12-Which of the following statements regarding toxoplasmosis in pregnancy is false ? Options A. Foetal abnormality may occur in cases of maternal infection occurring 6 months pre -conception. B. Treatment with pyrimethamine and sulphadiazine is recommended during the first trimester to reduce risk of foetal infection. C. Documented seroconversion confirms infection. D. Maternal toxoplasma-specific IgM may persist for over a year. E. Risk of fetal infection is increased with increasing gestational age.

Teaching Notes for Question 12 Theme: Toxoplasmosis in pregnancy. Infection in the mother is usually asymptomatic. Although foetal infection risk is greater during late gestation, the consequences of infection are greater during early pregnancy. Infected women may be serofast; chronically IgM positive with low levels lasting up to 2 years post -infection. Pyrimethamine is contraindicated during the first trimester. Features of congenital infection include choroidoretinopathy, microcephaly and intracerebral calcification with psychomotor retardation. The neonate may present acutely with petechiae, maculopapular rash and hepatomegaly. 13-A 9-year-old has a cranial CT scan. The radiologist reports the presence of intracranial calcification. Which of the following diagnoses is least likely to be associated with the radiologists report? Options: A. Subdural haematoma B. Sturge-Weber syndrome C. Toxoplasmosis D. Ataxia telangiectasia E. Hypoparathyroidism

Teaching Notes for Question 13 Theme: Intracranial calcification Causes of intracranial calcification: (i) Choroid plexus calcification (ii) Secondary to intracranial bleed (iii) Infectious causes (toxop lasma, CMV, TB, abscesses) (iv) Tuberose sclerosis (v) Sturge-Weber syndrome (vi) Hypo- and hyperparathyroidism (vii) Tumours (craniopharyngioma, astrocytoma) (viii) Vascular lesions Ataxia telangiectasia is not generally associated with intracrania l calcification

159

PEDIATRICS EXAMINATION REVIEW

14-Choose the one correct statement regarding a Congenital diaphragmatic hernia: Options A. It is most commonly seen on the right side B. It is associated with pulmonary hypoplasia C. It is associated with oligohydramnios D. It is always symptomatic at birth E. It usually presents with vomiting

Teaching Notes for Question 14 Theme: Congenital Diaphragmatic Hernia Congenital diaphragmatic hernia refers to the herniation of abdominal contents into the thoracic cavity. Congenital diaphragmatic hernias comprise three types: - Bochdalek hernia - the most common defect is through the posterolateral foramen of Bochdalek (left sided -90%) - Morgagni hernia - hernia through the anterior foramen of Morgagni - hiatus hernia. There is herniation of the small and large bowel, abdominal organs and in right sided hernias- liver into the thoracic cavity. Pulmonary hypoplasia is associated and is believed to be a causative factor. Malrotation is also seen as part of this condition. The usual presentation is with severe respiratory distress soon after birth. The abdomen is scaphoid, and in left sided hernias, there is decreased left sided air entry with hea rt sounds heard on the right. Treatment is with active resuscitation followed by surgery. Other associated anomalies include omphalocele, cardiovascular and CNS anomalies and Trisomy 21. A few infants may have a delayed presentation with vomiting as a re sult of intestinal obstruction or mild respiratory distress. 15-Regarding Histiocytosis X :Options: A. It is a cystic disease of lung, bone and anterior pituitary B. CXR shows small lungs with prominent markings and honeycombing C. 80% of patien ts are smokers; most are men 20-40 years old D. Most important treatment strategy is steroids E. It is a rare cause of pneumothorax

Teaching Notes for Question 15 Theme: Histiocytosis X This is a rare disease (prevalence 1/50000) affecting lung, bone and posterior pituitary function. Clinical presentation is varied from unifocal bone lesions to widespread multiple small lung cysts and nodular shadowing. It is indeed a ra re cause of pneumothorax. Rapidly worsening progressive disease is treated with etopiside. 16-A girl with an unusual facial appearance is found to have coarctation. Possible diagnoses are: Options: A. Noonan's syndrome B. William's syndrome C. Down's Syndrome D. Turner's syndrome E. Edward's syndrome

160

PEDIATRICS EXAMINATION REVIEW

Teaching Notes for Question 16 Theme: Unusual facial appearance and aortic coarctation Turners syndrome is assoc iated with aortic coarctation. Noonans syndrome has a phenotype similar to Turners, but is AD, thus meaning equal sex distribution. In addition, Noonans syndrome is associated with pulmonary stenosis and HOCM, not coarctation. Williams syndrome has a ssociated supravalvular aortic stenosis. Facial features of Turners syndrome include: webbed neck, high -arched palate and low posterior hairline.

17-A study measures the heights of 13 -yr-old children. Which is the best conclusion to draw from the data? Options: A. The data are positively skewed B. The data are normally distributed C. The mean is the best measure of central tendency D. The median is greater than the mode E. The median and interquartile range is a good way of summarising the data

Teaching Notes for Question 17 Theme: Skewed distribution This is a negatively skewed distribution which is shown above with tail towards the negative side. In general, it is always good to remember that: 1. Mode is towards the hump 2. Mean is towards the tail 3. Median is in the middle Best way of summarising skewed data is to give the median and interquartile range. A Normal distribution is best summarised by quoting the mean (for its middle point) and standard deviation (for its spread). A skewed distribution is best summarised by quoting the median (for its midd le point) and interquartile range (for its spread). 18-Which of the following hypotheses on the mechanisms of ageing is least accurate? Options: A. Ageing is caused by nitric oxide free radical-induced mitochondrial damage. B. Ageing is associated with altered insulin signalling. C. Ageing can be slowed in animal models by free radical scavengers. D. Ageing can be reduced in mammals by caloric restriction. E. Ageing is thought to be due to shortening of the telomeres.

Teaching Notes for Question 18 Theme: Mechanisms of ageing Ageing is complex and the mechanism unclear. It is thought to occur due to superoxide induced damage to structures such as mitochondria, pa rticularly by reactive oxygen species. This has been disputed by the finding that mitochondria from the elderly brain are functional. In animal models, a slowing of ageing can be induced by caloric restriction even in mammals and by free radical scavengin g. Studies in invertebrates have identified genes

161

PEDIATRICS EXAMINATION REVIEW that alter lifespan and which also encode components of insulin signalling pathways. Examples include DAF-2 (caenorhabditis elegans) and InR (Drosophila). A recent study using knockout mice implicates the mammalian insulin-like growth factor receptor as an important regulator of lifespan. Another prominent hypothesis for ageing is that associated with telomere shortening.

19-What is the diagnosis? Options: A. Left lower lobe consolidation B. Left lower lobe collapse C. Left lingular consolidation D. Left lingular collapse E. Right lower lobe consolidation

Teaching Notes for Question 19 Theme: Lower lobe consolidation * No loss of volume and there is airspace shadowing, therefore, it is consolidation. * Left paraspinal line (and left hemi -diaphragm) obscured, therefore, left lower lobe.

162

PEDIATRICS EXAMINATION REVIEW

20-Which one of the following is true regarding cancer genetics?: Options: A. Any gene whose encoded protein directly or ind irectly inhibits progression through the cell cycle and in which a loss-of-function mutation is oncogenic is an oncogene. B. RB, APC, and BRCA1 are oncogenes C. A gene whose product is involved either in transforming cells in culture or in inducing can cer in animals is called a proto-oncogene. D. A proto -oncogene is an abnormal gene that encodes a protein involved in regulation of cell growth or proliferation and that can be mutated into a cancer -promoting gene. E. A proto-oncogene is transformed on to an oncogene by changing the protein -coding segment or by altering its expression.

Teaching Notes for Question 20 Theme: Cancer genetics No teaching note available. 21-Which of the following renal disorders is associated with normal serum complement levels? Options: A. Membranoproliferative glomerulonephritis B. All forms of lupus nephritis C. Shunt nephritis D. Post-infectious glomerulonephritis E. Goodpastures disease

Teaching Notes for Question 21 Theme: Complement levels and glomerular disease. Causes of low C3 levels include: 1. Post-infectious glomerulonephritis 2. Lupus nephritis 3. MPGN 4. Shunt nephritis 5. Infective endocarditis with renal involvement 6. Type II cryoglobulinemia Both infective endocarditis and shunt nephritis are immune complex mediated disorders with activation of the complement pathway and resul ting hypocomplementemia. Note that Focal segmental glomerulosclerosis is not associated with low C3 levels. Similarly, rapidly progressive GN is a clinical diagnosis and is not necessarily hypocomplementemic. 22-What is the most likely explanation for th ese biochemical changes in a 15 -year-old girl with erosive changes in her mouth? Na 138 mmol/l K 3.2 mmol/l HCO3 36 mmol/l Ca 2.4 mmol/l P04 1.0 mmol/l Urine chloride 10 mmol/l Options: A. Primary hyperparathyroidism B. Primary hyperaldosteronism C. Persistent vomiting D. Anorexia nervosa E. Ehlers Danlos syndrome

163

PEDIATRICS EXAMINATION REVIEW Teaching Notes for Question 95 Theme: Acid-base disorders She has a metabolic alkalo sis with hypokalaemia and a low urine chloride, indicating volume depletion. 23-A patient presents with acute infective arthritis. The resulting inflammation: Options: A. Causes the release of C8a, a potent chemokine B. Causes the release of CRP, th e half-life of which is several days C. May be defined as increased vascular permeability accompanied by an infiltration of inflammatory cells D. Is always harmful E. Once initiated, it follows a stereotypical cascade, which is not regulated

Teaching Notes for Question 23 Theme: Acute inflammation. The combination of increased vascular permeability and cellular influx gives rise to the calor, rubor and dolor of the inflammatory process. CRP is an acute phase protein which is synthesised by hepatocytes as a result of the acute inflammatory process. It is secreted in increased amounts within 6 hours of an acute inflammatory stimulus. CRP has a half -life of about 12 hours. C-reactive protein binds to phosphocholine moieties found on microbial polysaccharides. It acts as an opsonin enhancing phagocytosis, and also activates the classical complement pathway. Acute infection results in activation of complement. C5a is a compl ement intermediate which acts as a potent chemokine. The resolution of inflammation is tightly regulated.

24-What does this ECG of an 8-year-old boy with Ebsteins anomaly show? Options: A. Atrial flutter B. Supraventricular tachycardia C. Sinus tachycardia D. AV reentrant tachycardia due to accessory pathway E. Ectopic atrial tachycardia

164

PEDIATRICS EXAMINATION REVIEW Teaching Notes for Question 24 Theme: Atrial flutter * This ECG shows classical saw -toothed atrial waves typical of atrial flutter. These are negative in the inferior leads and have a rate of 300/min. The ventricular rate is typica lly 150/min because of 2:1 AV conduction. * The next most likely diagnosis with this ECG is ectopic atrial tachycardia. However the typical P wave morphology and rate is pathognomonic of atrial flutter. * In other SVTs (including those utilising accessor y pathways), dissociated atrial and ventricular activity is never seen as both atria and ventricles are part of the same circuit. 25-Which of the following conditions is not associated with hyperuricaemia : Options: A. Lesch-Nyhan syndrome B. Thiazide diuretic therapy C. Polycythaemia rubra vera D. Severe exfoliative psoriasis E. High-dose aspirin therapy

Teaching Notes for Question 25 Theme: Hyperuricaemia Uric acid is the final product of purine metabolism. Endogenous uric acid comprises roughly two thirds of total body urate. The remaining one third is derived from the diet. Approximately 70% of the urate produced daily is excreted by the kidney, while the res t is eliminated by the intestines. Hyperuricaemia may be due to: -Decreased excretion (under-excretors) -Increased production (over-producers) -A combination of these 2 mechanisms Decreased uric acid excretion: This can result from a decreased glomerular filtration, decreased tubular secretion, or increased renal tubular reabsorption. Causes are: -Renal insufficiency -Acidosis - diabetic ketoacidosis, ethanol, starvation ketosis. Accumulation of organic acids competes with uric acid for renal tubula r secretion. -Diuretic therapy - enhanced renal tubular reabsorption of uric acid. -Low-dose aspirin - inhibition of uric acid secretion. Low-dose aspirin is associated with hyperuricaemia, whereas high-dose aspirin can have a therapeutic effect in gout. -Diabetes insipidus - enhanced renal tubular reabsorption of uric acid. Overproduction of uric acid: This accounts for only a minority of patients presenting with hyperuricaemia. Causes are: -Increased intake in the diet. -Increased purine nucleotide breakdown - due to rapid cell proliferation e.g. blast crisis of leukaemias, increased cell death rhabdomyolysis, severe psoriasis, cytotoxic therapy -Enzymatic defects - Lesch-Nyhan syndrome - an inherited X-linked disorder due to a deficiency of HGPRT - an enzyme that catalyses the conversion of hypoxanthine to inosinic acid. The deficiency of HGPRT accelerates purine biosynthesis with a resultant increase in uric acid production. In addition to arthropathy and obstructive nephropathy, these patients develop a neurological disorder that is characterised by choreoathetosis, spasticity and occasionally self-mutilation. -Glycogenoses types III, IV, and VII can result in hyperuricaemia due to excessive degradation of skeletal muscle ATP. Combined mechanisms: -Alcohol consumption - results in accelerated hepatic breakdown of ATP and generation of organic acids that compete with urate for tubular secretion.

165

PEDIATRICS EXAMINATION REVIEW

26-In a patient with hypomagnesaemia, the following statement is not true: Options: A. Recent cisplatin treatment could have been the cause B. Hypocalcaemia may occur C. Neuromuscular symptoms are similar to those in hypocalcaemia D. Prolonged vomiting is an indication for high -dose oral magnesium supplementation E. Hungry bone syndrome is associated with reduced serum magnesium

Teaching Notes for Question 26 Theme: Hypomagnesaemia. Hypomagnesaemia results in defective PTH secretion and secondary hypercal caemia. 27-All these are major clinical features of neurofibromatosis -1 (NF-1) except: Options A. Axillary freckling B. Tibial pseudoarthrosis C. Cortical tubers D. Lisch nodules E. Caf au lait patches

Teaching Notes for Question 27 Theme: Neurofibromatosis -1 * NF-1 is an autosomal dominant neurocutaneous disorder affecting approximately 1 in 5,000 individuals. Diagnosis of this condition is mainly done on a cl inical basis with gene testing offered to a small minority. The main features of this condition include: - caf au lait patches, - axillary freckling, - neurofibromas, - Lisch nodules in the eye, - skeletal manifestations and - a family history of a first-degree relative diagnosed with NF -1. * Cortical tubers are seen on neuroimaging in another AD neurocutaneous condition called tuberous sclerosis 28-Choose the one correct statement regarding necrotising enterocolitis: Options: A. It usually involves the lower colon B. It is commonly associated with gas in the biliary tree C. It is not seen in babies who haven't been started on feeds yet D. It may lead to intestinal obstruction a few weeks later E. It can be prevented by the use of oral antibiotics

Teaching Notes for Question 28 Theme: Necrotising enterocolitis Necrotising enterocolitis is the most common serious surgical condition affecting neonates. It is predominantly seen in preterms. It usually involves the terminal ileum and ascending colon. Gas in the biliary tree although a pathognomonic sign of NEC, is not frequently seen. A number of factors have been implicated in the aetiology of NEC, includi ng luminal substrate, bacterial overgrowth, rapid introduction of feeds and the use of formula feeds.

166

PEDIATRICS EXAMINATION REVIEW However, it may also occur in babies who have never been fed. The use of oral antibiotics does not reduce the risk of developing NEC. 29- Choose the correct statement regarding alpha 1 -antitrypsin (AAT): Options: A. It is a membrane bound tyrosine kinase B. Over 75% of cases of emphysema in the UK are associated with the deficiency of this enzyme C. Deficiency presents as cirrhosis in children D. The low levels of the enzyme in deficiency syndromes are due to lack of synthesis of the variant enzyme E. Emphysema in deficiency syndromes is characterised histologically by inclusion bodies

Teaching Notes for Question 29 Theme: Alpha1-antitrypsin (AAT) Alpha 1-antitrypsin deficiency is an inherited condition caused by a defective gene on chromosome 14. Alpha 1-antitrypsin (AAT) is a serine protease inhibitor (serpin) that protects the connective tissue of the lungs from elastase - an enzyme released by leucocytes during phagocytosis. Other members of the serpin family include C1 -esterase inhibitor, alpha 2 -antiplasmin, and brain-specific neuroserpin. In alpha 1-antitrypsin deficiency, emphysema results from the uncontrolled action of proteases on pulmonary tissue. Hepatic pathology is also associated with tissue damage secondary to the accumulation of polymerised AAT. Alpha 1-antitrypsin is encoded by the protease inh ibitor (Pi) locus. Over 100 different variants of alpha 1-antitrypsin have been identified, and the protease alleles are co dominant. The normal allele (M) is seen in over 95% of Europeans. Another allele (Z) results in an increased risk of emphysema, and patients with the ZZ phenotype have levels of AAT less than 20% of normal and develop severe disease. Heterozygous states (PiMZ, PiMS, PiSZ) have a less severe phenotype. People with both PiS genes (PiSS) are less severely affected, with alpha 1 -antitrypsin levels 6070 per cent of normal. AAT deficiency accounts for less than 1% of cases of COPD in the UK. Although plasma levels of the enzyme are low, this is not due to a lack of synthesis rather a blockage of its processing and secretion from the liver. The majority of patients with clinical liver disease are homozygotes with a PiZZ phenotype. Initially, hepatic pathology is often limited to minor portal fibrosis but frequently progresses to overt liver damage in late adulthood. Alpha 1-antitrypsin deficiency can produce jaundice and liver problems in babies within days of their birth. Some neonates with the PiZZ genotype may develop rapidly progressive liver damage - a clinical indication for liver transplantation. A few children with alpha 1 -antitrypsin deficiency develop significant liver disease before the age of 20, and approximately 1015% of adult patients will develop cirrhosis usually over the age of 50 years. Approximately 5% of patients die of their liver disease. The variability in the severity and age of onset of liver disease among patients with AAT deficiency can be explained in part by individual variations in episodes of inflammation and hence to increased synthesis of AAT. The consistent occurrence of overt liver disease in newborn infants, in contrast to its only occasional occurrence in young adults, can be explained by the fact that the liver cells in the infants are less capable of degrading the polymerised protein. Inclusion bodies in hepatocytes are a feature of AAT deficiency. Smoking is the greatest risk factor for the development of emphysema in adults with alpha 1-antitrypsin deficiency. In non -smokers, this may never develop or occur later in life. Cigarette smoking increases the number of neutrophils in the alveolus and ina ctivates the remaining small amounts of antiprotease.

167

PEDIATRICS EXAMINATION REVIEW

30-Which of the following is not a feature of the Holmes Adie syndrome? Options A. The affected pupil is always larger than the unaffected B. Both pupils may be affected C. There is d enervation supersensitivity of the pupil D. Global areflexia is a recognised association E. Ptosis is rare

Teaching Notes for Question 30 Theme: Holmes -Adie syndrome Holmes-Adie syndrome consists of impaired pupillary constriction to light associated with loss of deep tendon reflexes. It is caused by idiopathic degeneration of parasympathetic neurones in the ciliary ganglion and therefore exhibits denervation supersensit ivity: the affected pupil will constrict on exposure to low concentration of pilocarpine (0.125%) whereas the unaffected will not. With time the affected pupil may become small; the other pupil may also become affected 31-A 17 year old girl presents with primary amenorrhoea. She has recently started developing breasts. She is 145 cm tall and weighs 48 kg. Both her sisters (age 14 and 19) are greater then 170cm tall and are well. Hb 10.5 , ALP 320 MCV 108 Bili 12, WCC 7.6 , AST 34 Ca2+ 1.9 U&E normal, PO4 0.7 , TFTs normal What is the underlying diagnosis ? Options: A. Crohns disease B. Coeliac disease C. Cystic fibrosis D. Addisons disease E. Ulcerative colitis

Teaching Notes for Question 31 Theme: Primary amenorrhoea This young woman has a classical complication of celiac: Vitamin D deficiency resulting in hypocalcaemia, she also has macrocytic anaemia, again suggestive of the diagnosis. There will be a compensatory rise in her parathyroid hormone levels and there is a risk of secondary hyperparathyroidism. 32-Which of the following is not true of narcolepsy? Options: A. It is associated with cataplexy B. It exhibits X-linked inheritance pattern C. Night time insomnia can be a feature D. It is associated with early -onset REM sleep E. It may be associated with hypnopompic hallucinations

168

PEDIATRICS EXAMINATION REVIEW Teaching Notes for Question 32 Theme: Narcolepsy This idiopathic condition is characterised by excessive daytime sleepiness associated with cataplexy (loss of muscular tone triggered by emotional events), vivid hallucinations on falling asleep (hypnogogic) or waking (hypnopompic), and sleep paralysis. Sleep studies show abnormal periods of early -onset REM sleep. Sleepiness responds to stimulants and cataplexy to clomipramine. There is a breed of Doberman pinschers in whom the condition is dominantly inherited (AD), the affected gene encodes a protein produced in the lateral hypothalamus called hypocretin. 33-The following are all true about migraine in children except: Options: A. More than 50% of headaches are unilateral B. EEG will help in the diagnosis C. Other members of the family may be affected D. An aura and postdrome stage is present as in adult patients E. Sumatriptan is not licensed for use in children

Teaching Notes for Question 33 Theme: Migraine Headache is one of the commonest reasons for referral to the neurology clinic. Chronic, non-progressive headache needs non -urgent investigation and initial treatment with simple analgesia can be commenced. Sometimes a multi -disciplinary team approach is required. The acute headache, which presents over a short period of time with neurological deficit, requires urgent investigation. Migraine is an acute headache, which occurs once per week or per month and is usually associated with a strong family history. More than 50% of cases are not unilateral, and usually the classical picture of migraine attack does not accompany the attack. A history of stomach pain, blurred vision and even pain in the throat are warning signs. In many cases headache is associated with photophobia, blurred vision and vomiting. The headache may last from one hour to 2 3 days. Sleep in a dark quiet room with analgesia is often useful. Triptans are not licensed in children but various studies have proven their efficacy in adults. 34-The following statements about techniques using FISH (fluorescent in situ hybridisation) are true except: Options: A. The technique can be performed on uncultured cells B. The technique requires specialised microscopes C. The technique can detect small deletions in DNA D. The technique is used instead of light microscopy for routine chromosome analysis E. The technique can be used to map the position of genes on a chromosome

Teaching Notes for Question 34 Theme: Diagnostic techniques Artificially generated DNA probes carrying a fluorescent label hybridise (attach) to known specific chromosomal regions. Special microscopes using UV filters are used to detect the fluorescent signals. Very small rearrangements can be identified by this technique. Interphase FISH on uncultured cells is currently used in rapid prenatal diag nosis. FISH analysis is used as a second line of investigation after conventional light microscopy. One of the main research uses of FISH is to localise genes on chromosomes.

169

PEDIATRICS EXAMINATION REVIEW

35-Cardiac tamponade may occur with all of the following except: Options: A. Tuberculosis B. Rheumatic fever C. Pneumococcal pneumonia D. Rheumatoid arthritis E. Uraemia

Teaching Notes for Question 35 Theme: Cardiac tamponade All the above cause pericarditis, but rheumatic fever does not lead to tamponade that needs decompression. 36-The secondary immunoglobulin response is not characterized by: Options: A. Isotype switching to IgG B. Production of immunoglobulin with higher affin ity to the antigen C. Production of memory plasma cells D. A quantitatively increased antibody production as compared to the primary immune response E. Requirement of help from T-cells

Teaching Notes for Question 36 Theme: Secondary immunoglobulin response Memory B-cells are responsible for the secondary Ig response. Memory B -cells differ qualitatively as well as quantitatively from nave B cells. After priming during t he primary immune response, the frequency of antigen -specific B-cells may increase 10-100 fold. Therefore the secondary immune response to a specific antigen occurs more rapidly than the primary response, and larger amounts of antibody are produced. Memor y B-cells also have higher affinity for antigen as they have undergone affinity maturation in the germinal centres of lymphoid tissue. Memory B -lymphocytes have undergone class switch and produce mainly IgG but also other immunoglobulin isotypes. Both prim ary and secondary immunoglobulin responses require T-cell help. (NB. Certain bacterial antigens can activate B cells directly, e.g. LPS). Plasma cells are the effector cells of both primary and secondary immune responses. They secrete large amounts of anti body, but are short lived cells in order to limit antibody responses.

170

PEDIATRICS EXAMINATION REVIEW

37-What is the abnormality seen in this girl with bone pain? Options: A. Subperiosteal resorption of HPTH (Hyperparathyroidism) B. HPOA (Hypertrophic Osteoarthropathy) C. AVN (Avascular necrosis) talus D. Myelomatous deposit E. Osteopetrosis

Teaching Notes for Question 37 Theme: Hypertrophic Osteoarthropathy HPOA - periosteal reaction along distal long bones. Causes: * Chest: CA bronchus, lymphoma, abscess, bronchiectasis, metastasis * Pleural: fibroma or mesothelioma * CVS: Cyanotic congen heart disease * GIT: IBD, dysentery, lymphoma, Whipples, coeliac, cirrhosis esp PBC Subperiosteal resorption HPTH. 38-In the investigation of a patient who is admitted following 3 dizzy spells each markedly improved by eating:Options: A. Urine sulphonylurea levels are unhelpful B. A 72hr fast should be organised, with the doctor called when the blood sugar reaches 4mmol/l. C. Serum insulin, glucose and C-peptide should be taken every morning for 7 days D. Serum C -peptide will not be raised in exogenous insulin use E. A family history of scars in the neck is irrelevant

Teaching Notes for Question 38 Theme: Hypoglycaemia Investigation of hypoglycaemia requires a 72hr fast, ideally, levels of glucose, C-peptide and insulin are taken when BM is LOW <4. Serum insulin and urine sulphonylureas levels help check for use of these drugs. In multiple endocrine neoplasia type 1, pituitary, pancreas, and parathyroid tumours occur. Causes of hypoglycaemia include insulinoma,

171

PEDIATRICS EXAMINATION REVIEW exogenous insulin administration, and sulphonylurea intake. In insulinoma, plasma insulin and C-peptide are high, and urine sulphonylurea levels are undetectable. In cases of exogenous insulin administration, plasma insulin is very high, C-peptide is not elevated, and urine sulphonylurea levels are undetectable. In cases of sulphonylurea -induced hypoglycaemia, plasma insulin and C -peptide are high, and sulphonylurea can be detected in urine. 39-A 16-year-old girl presents with fever, joint aches and a red rash on her face, neck and hands. She attributed the rash to severe sunburn and says that she has sensitive skin. What are the most appropriate tests to request? Options: A. Anti nuclear antibody and c-reactive protein B. c-reactive protein and erythrocyte sedimentation rate C. Full blood count and chemistry D. Anti-nuclear antibody and erythrocyte sedimentation rate E. Complement levels

Teaching Notes for Question 39 Theme: Systemic lupus erythematosus This patient has Systemic Lupus Erythematosus. The cutaneous lesions of SLE can be sub -divided: i) Mauve erythema over cheeks and nose: Butterfly rash ii) Maculopapular erythematous rash on sun -exposed skin iii) Chilblain lupus iv) Vasculitis v) Diffuse alopecia vi) Raynauds phenomenon vii) Livedo Reticularis (light purple net -like rash on limbs) viii) Urticarial eruptions ix) Bullous lesions from photosensitivity x) Discoid Lupus (Chronic Cutaneous Lupus) xi) Annular erythema in photosensitive distribution (Sub -acute Lupus Erythematosus) xii) Generally, any rash in a photosensitive distribution should have SLE excluded. Check ANA, antiDNA antibodies. Also check ESR and complement levels. 40-Which of the following is true about prolactin? Options A. Elevated levels produce gynaecomastia without galactorrhoea B. Damage to the pituitary stalk reduces prolactin levels C. Metoclopramide increases prolactin production D. Levels are elevated in >90% of cases of acromegaly E. Levels are increased by cimetidine

Teaching Notes for Question 40 Theme: Prolactin Elevated prolactin levels produce gynaecomastia with galactorrhoea. Prolactin secretion is inhibited by dopaminergic stimulation carried in the pituitary stalk. Damage to this increases prolactin levels. Metoclopramide is a dopamine antagonist, which increases pro lactin levels. In acromegaly, injury to the stalk, increased TRH (as TSH decreased) and co -secretion of prolactin mean that 30% have prolactin elevation. Cimetidine has a weak anti-androgen effect, which produces gynaecomastia, but not galactorrhoea or e levated prolactin.

172

PEDIATRICS EXAMINATION REVIEW

41-Choose the incorrect statement. By the age of 9 months a child: Options A. Can sit with support B. Can crawl C. Can stand alone D. Can play peek -a-boo E. Has lost the Moro reflex

Teaching Notes for Question 41 Theme: Developmental milestones Developmental milestones reached by 9 months: Gross motor development: * Can sit unsupported for 10 -15 minutes * Can reach out for objects and still maintain balance * Crawling * Pulls to stand Fine motor development: * Starts to grasp object between thumb and forefinger * Bangs two bricks together * No hand preference Social development: * Feed self with biscuit (5 to 8 months) * Responds to name Speech and language: * Babbles with syllables Moro reflex appears at birth and disappears at 4 -5/12 of age. Most children begin to stand alone at 12 months. 42-An 18-year-old man is referred to the outpatient clinic as his general practitioner has found him to have microscopic haematuria. He also has bilateral sensorineural deafness with a sister and younger brother who are similarly afflicted. What is the most useful investigation? Options: A. Urine microscopy B. Serum creatinine C. Anti-GBM antibodies D. Audiometry E. Renal biopsy

Teaching Notes for Question 42 Theme: Alport's syndrome Only renal biopsy can confirm a diagnosis of Alport's. Important for discussion about inheritance. Alports is usually (approx 80%) inherited in an X -linked fashion with a defect in the COL4A5 gene (alpha 5 chain of type IV collagen), which is present in the glomerular basement membrane, ear and eye leading to clinical disease of haematuria, then proteinuria and progressive renal impairment, deafness, anterior lenticonus, corneal lesions and rarely leiomyomata. Diagnosis is by renal biopsy or occasionally skin biopsy. 43-The following statement is false regarding treatment of Juvenile Idiopathic Arthritis (JIA):Options A. Steroids are the agents of first choice.

173

PEDIATRICS EXAMINATION REVIEW

B. Hydroxychloroquine is associated with ocular toxicity. C. Iridocyclitis usually responds to topical steroids. D. Gold salts may lead to nephrotic syndrome. E. Low dose methotrexate may be beneficial

Teaching Notes for Question 43 Theme: treatment of Juvenile Idiopathic Arthritis Methotrexate is the treatment of cho ice in polyarticular and extended oligoarticular diseases. Side Effects include marrow suppression, pulmonary fibrosis and GI symptoms especially nausea and vomiting. Popular viva question in Part2: MTX is given weekly, and can be administered via the ora l, subcutaneous or intramuscular route. In cases of oligoarticular disease, intra -articular steroids are usually tried first before systemic treatments.

44-What is the abnormality shown on Chest X -ray? Options: A. Consolidation right lower lobe B. Collapse right lower lobe C. Collapse lingular lobe D. Perforation E. Bilateral pneumothoraces

Teaching Notes for Question 44 Theme: Chest X-ray - Perforation Perforation * Free air under the hemi-diaphragm, outlines liver due to perforation of an intraperitoneal viscus (not a retroperitoneal viscus).

174

PEDIATRICS EXAMINATION REVIEW

45-What is the diagnosis? Options: A. Proliferative DM retinopathy B. Grade 3 hypertensive retinopathy C. Endocarditis D. CMV retinitis E. Background retinopathy

Teaching Notes for Question 45 Theme: Retinopathy. This slide demonstrates retinal blot haemorrhages and exudates. This appearance would do for hypertensive or diabetic retinopathy. The absence of new blood vessels is against a proliferative retinopathy. Endocarditis would tend to produce more discreet white balls on fundoscopy. The haemorrhages and exudates look too well defined for this to be CMV retinitis. The absence of microaneurysms is against a diagnosis of background retinopathy. 46-All of the following statements about Glucose -6-phosphate dehydrogenase deficiency are false except: Options: A. Is more common in women than in men. B. Is caused by mutations of the G6PD gene on chromosome 4. C. Often leads to chronic haemolysis. D. Requires treatment with desferrioxamine to prevent iron overload. E. Is associated with haemolytic crises induced by sulfonamides.

Teaching Notes for Question 46 Theme: Glucose-6-phosphate dehydrogenase deficiency G6PDH catalyses the synthesis of NADPH from the hexose monophosphate pathway. The NADPH is then used to reduce glutathione and hence in c onjunction with glutathione reductase reduces cellular oxidative damage.

175

PEDIATRICS EXAMINATION REVIEW

G6PDH enzyme deficiency, caused by mutations in the X -linked (Xq28) G6PD gene, affects over half a billion people worldwide. It is a balanced polymorphism associated with resistance to falciparum malaria in heterozygous females. This evolutionary advantage outweighs the small negative effect of affected hemizygous males. At least 400 G6PD variants have been identified. The various mutations lead to altered expression of G6PD or an altered half-life of the enzyme. There are a number of disease manifestations: - patients may be asymptomatic - patients may present with neonatal jaundice - usually the deficiency manifests as acute haemolytic crises provoked by oxidising stress e.g. certain foods - fava bean, chemicals - naphthalene, certain drugs - dapsone, primaquine, chloroquine, aspirin (high doses), sulphonamides, nitrofurantoin, vitamin K and nalidixic acid. Infections can also provoke haemolysis. Haemolytic crises vary in severi ty, rarely chronic haemolysis may occur. If severe, G6PD deficiency can cause immunodeficiency by reducing the NADPH required for activation of the NADPH oxidase enzyme (cf. CGD). 47-Which of the following statements correctly describes the clinical pheno types which are associated with a deficiency of the various complement components? Options: A. C5-C9 deficiency and mycoplasma infection B. C1q deficiency and angio -oedema C. C2 deficiency and rheumatoid arthritis D. C3 def is associated with par tial lipodystrophy E. A raised C4 and low C3 in SLE is associated with active disease

Teaching Notes for Question 47 Theme: Complement Deficiencies in the components of th e classical pathway (C1,C2, C4) predispose to immune-complex disease and systemic lupus erythematosis (due to inefficient removal of apoptotic cells). Deficiencies in the alternative pathway (Factor D, P) leads to infection with pyogenic bacteria and Neisseria but no immune complex disease. C3 deficiency is associated with a wide range of pyogenic infections and Neisseria infections and sometimes with immune complex disease. Deficiencies in the components of the membrane attack complex (C5 -C9) result in susceptibility to Neisseria infection only (estimated 10,000 fold!). Deficiencies of MBL (MBL, MASP1, MASP2, C2, C4) leads to bacterial infections, mainly in childhood. Hereditary angioneurotic oedema is due to C1 inhibitor deficiency. CD59 or DAF deficiencies result in paroxysmal nocturnal haemoglobinuria. In active SLE, both C3 and C4 are reduced. Reduced levels of C3 can also be seen in cholesterol emboli, post streptococcal glomerulonephritis and mesangiocapillary glomerulonephritis type II. 48-The following autoantibodies are commonly found in the named disease: Options A. Anti-liver/kidney microsomal antibodies (anti LKM-1) in type I autoimmune hepatitis B. Anti-phospholipid antibodies in Primary biliary cirrhosis C. Anti-mitochondrial antibod ies in Primary biliary cirrhosis D. ANCA in type II autoimmune hepatitis E. Anti-liver cytosol antibodies in primary sclerosing cholangitis

Teaching Notes for Question 4 8 Theme: Autoimmune hepatitis Type I autoimmune hepatitis is characterised by the presence of anti -nuclear antibodies, anti-smooth muscle antibodies and anti-mitochondrial antibodies. Type 2 is characterised by anti -liver/kidney microsomal antibodies (LKM-1), anti-liver

176

PEDIATRICS EXAMINATION REVIEW cytosol antibodies. Primary biliary cirrhosis is characterised by anti -nuclear antibodies and anti-mitochondrial (M2) antibodies. Primary sclerosing cholangitis is characterised by ANCA positivity. 49-All of the following are clinical fe atures of severe aortic stenosis except: Options: A. Fourth heart sound B. Third heart sound C. Soft aortic component of second heart sound D. Systolic thrill E. Length of murmur in systole

Teaching Notes for Question 49 Theme: Features of aortic stenosis 5 clinical signs of severe AS are: * Slow rising pulse character/Narrow pulse pressure * 4th HS * Soft aortic 2nd HS * Systolic thrill * Length of murmur in systole 3rd HS is a feature of the volume -loaded LV e.g. decompensated heart failure, severe MR/AR. Loudness of a murmur is usually NOT a clinical feature of its severity. 50-A two-year-old boy is brought to the resuscitation room. He has been unwell f or 12 hours with fever and has vomited once. On examination, his airway is patent and he is breathing oxygen via a mask with non -rebreathing bag with oxygen saturations of 100%. His heart rate is 160/min, capillary refill time is 5 seconds and he has a pur puric rash over his lower limbs. He is not responsive to pain. A colleague has been trying for 5 minutes to secure intravenous access when you arrive. The next step should be: Options: A. Continue to try for IV access and then give 20mL/kg fluid if suc cessful B. Check blood sugar C. Secure intraosseus access and give 20mL/kg fluid D. Give immediate IM ceftriaxone before securing IV access E. Check a capillary blood gas

Teaching Notes for Question 50 Theme: Resuscitation in sepsis The priorities, as always, are ABC. This child has a patent airway and adequate oxygenation. The current priority therefore is treatment of his shock (tachycardia and prolonged CRT). It is very difficult to secure IV access when a child is so peripherally shut down, and in a child like this who is unresponsive to pain, intraosseus access should be secured and fluid resuscitation with colloid administered. Ceftriaxone can be given through the IO needle. After fluid resuscitation, 'disability' can be fully assessed (coma score, pupils, posture, and blood sugar). 51-A patient is noted to have a small left pupil. There is also a ptosis present in this eye. His mother has noted that the left si de of his face is dryer than normal. Where is the lesion likely to be? Options A. Oculomotor nerve B. Trigeminal nerve

177

PEDIATRICS EXAMINATION REVIEW

C. Facial nerve D. Vagus nerve E. Cervical sympathetic trunk

Teaching Notes for Question 51 Theme: Horner's syndrome Interruption of the cervical sympathetic trunk results in Horners syndrome. A Horner's consists of ptosis, pupil constriction and absent sweating over the affected side of the face . Possible causes include vascular lesions (carotid pathology), lung lesions (especially apical squamous cell tumours), trauma and syringomyelia. 52-A patient is diagnosed as having a Horner's syndrome following carotid surgery. Which of the following is NOT a symptom of Horners syndrome? Options: A. Pupillary constriction B. Ptosis C. Sinking in of one eye D. Absence of sweating on face and neck E. Lack of lacrimation

Teaching Notes for Question 52 Theme: Horner's Symptoms of Horners syndrome include miosis, ptosis, anhydrosis and enophthalmos. It is due to an interruption in the symapthetic nerve supply to the eye. Causes include carotid pathology (tumours, a neurysm and dissection), neck injuries, a syrinx and lung lesions. Especially apical squamous cell carcinoma. 53-Which of the following sub-group of populations is the influenza vaccine not recommended for?:Options A. Immunosupressed people due to disease or treatment B. Those with chronic heart disease C. Those with chronic renal failure D. Those with diabetes mellitus E. Those with coeliac disease

Teaching Notes for Question 53 Theme: Influenza vaccine Influenza viruses constantly alter there H (haemagglutinin) and N(neuraminidases)antigens present on the surface. Every year the World Health Organisation (WHO) recommends which strains should be included. Since these vaccines will not control an epidemic they are only for those people at high risk from infection.It is recommended for people who are at risk of serious influenza or its complications, and for people who can spread influenza to those at high ris k (including all household members): People at high risk for complications from influenza: All children 6-23 months of age. People 65 years of age and older. Residents of long-term care facilities housing persons with chronic medical conditions . People who have long-term health problems with: - heart disease - kidney disease

178

PEDIATRICS EXAMINATION REVIEW - lung disease - metabolic disease, such as diabetes - asthma - anemia, and other blood disorders People with certain conditions (such as neuromuscular disorders) that can cause breathing problems. People with a weakened immune system due to: - HIV/AIDS or other diseases affecting the immune system - long-term treatment with drugs such as steroids - cancer treatment with x-rays or drugs People 6 months to 18 years of age on long -term aspirin treatment (these people could develop Reye Syndrome if they got influenza). Women who will be pregnant during influenza season. The vaccine is an inactivated virus. Other medical conditions, including coeliac disea se are not high risk for developing influenza. 54-The lipid-lowering effect of a new statin is compared to atorvastatin. Critical level is set at 0.01 and difference is found (p = 0.001). What is the best conclusion to draw? Options A. A larger sample should have been used B. The alternative hypothesis should be accepted C. The probability of a type I error occuring is 1% D. The p-value associated with this statin means that it is a clinically significant new drug E. The trial would have been better run against placebo

Teaching Notes for Question 54 Theme: Data analysis Where the p-value is less than the critical value, there is good evidence to accept the alternative hypothesis. The p -value is the probability of committing a Type I error; C would be correct if it said 0.1%. 55-Which of the following is correct regarding atrial flutter? Options: A. Is usually left atrial in origin B. Usually causes a narrow complex tachycardia at 150 bpm C. Is often associated with an irregular pulse D. Can be terminated with intravenous adenosine E. Implies the presence an accessory pathway

Teaching Notes for Question 55 Theme: Atrial flutter * Atrial flutter occurs because of an abnormal circuit of current in the atria, which usually cycles at a rate of 300 bpm (280320). The ventricular rate is determined by the degree of AV nodal block. Commonly, every second flutter wave is conducted (2:1 AV block), giving a ventricular rate of 150 bpm. Although usually regular, the degree of AV block sometimes varies from beat to beat, giving an irregular heart rate. * Adenosine does not terminate atrial flutter, but does increase the degree of AV block this slows the ventricular rate transiently and may reveal flutter waves on ECG, which can be diagnostic. * The circuit of current usually occurs via an anatomically distinct pathway, which can be interrupted by radiofrequency ablation. * Most atrial flutter originates in the RA, with a counterclockwise circuit around the tricuspid valve annulus.

179

PEDIATRICS EXAMINATION REVIEW

56-The following statements are true about Tetralogy of Fallot (TOF) except: Options: A. Children with TOF are susceptible to cerebral abscess B. The chest X-ray shows oligaemic lung fields C. The second heart sound is soft and single D. TOF is associated with dextroposition of the aorta E. This disorder results in a left to right shunt

Teaching Notes for Question 56 Theme: Tetralogy of Fallot TOF- 6% of congenital heart disease It comprises: -ventricular septal defect -pulmonary stenosis -overiding of the ventricular septum by the aorta -right ventricular hypertrophy * The VSD is usually large and non-restrictive. * Pulmonary stenosis affects the pulmonary artery, the pulmonary valve and the infundibulum. * Severity is related to the degree of right ou tflow tract obstruction. Patients present with: -Cyanosis - usually not present at birth but develops over the first year. -'Hypercyanotic attacks' - due to spasm of the infundibular muscle. During these attacks the infant becomes cyanosed, restless and tachypnoeic. Loss of consciousness may occur. On auscultation, the murmur may decrease in intensity -Squatting - this increases systemic blood pressure, and increases pulmonary blood flow -Clubbing, pan or ejection systolic murmur due to right ventricul ar obstruction, single second heart sound, soft P2 -Failure to thrive -Polycythaemia -CXR findings: * boot-shaped heart (enlarged right ventricle) * right aortic arch -25% * small or concave pulmonary artery * oligaemic lung fields -ECG shows right axis deviation, right atrial and right ventricular hypertrophy -Echocardiogram is diagnostic -cardiac catheterisation -angiography Treatment of hypercyanotic spells: -placing the child in the knee-chest position -morphine -beta -blockers to reduce infundibular spasm -bicarbonate to correct the acidosis Antimicrobial prophylaxis is initiated for prevention of endocarditis. Surgery is usually in two stages: -Blalock shunt - the subclavian artery is joined to the pulmonary artery -Waterston operati on - the ascending aorta is joined to the right pulmonary artery Total repair is undertaken at school age. 57-In terms of the NICE guideline of screening for diabetic nephropathy in Type 2 patients: Options: A. The target HbA1c is about 8%. B. A renal consultation is advised if the serum creatinine rises above 250 mol/l. C. All patients with confirmed microalbuminuria, if no contraindications, should be on an ACE inhibitor. D. Blood pressure is less important than reducing the degree of protein uria. E. Calculating ratios of albumin and creatinine in the urine is not as useful as the total urine protein level.

180

PEDIATRICS EXAMINATION REVIEW

Teaching Notes for Question 57 Theme: Diabetic nephropathy For this nephropathy and the retinopathy guideline, one can check; www.nice.org.uk In fact: * target HbA1c is 6.5-7.5% * BP target 135/75 * albumin:creatinine ratio >2.5 (men) and >3.5 (women) is of higher risk * renal consultation advised if creatinine >150 * ACE inhibition is the drug of choice

58-What can you infer from the Chest X-ray? Options: A. Left pneumothorax B. Left lower lobe collapse C. Left lingula collapse D. Left tension pneumothorax E. Left hilar adenopathy

Teaching Notes for Question 58 Theme: Tension pneumothorax * Left hemithorax greate r volume than right * Air outlines right mediastinal border (i.e. has crept posteriorly) 59-Which of the organs below is considered an immunologically privileged site : Options: A. Skin B. Thymus C. Bone marrow D. Testicle

181

PEDIATRICS EXAMINATION REVIEW

E. Pancreas

Teaching Notes for Question 59 Theme: Immune privilege Some tissues fail to elicit an immune response and are termed immunologically privileged sites- brain, cornea, testis, uterus and arguably liver. Several factors may contribute to immune privilege. The blood-brain barrier may block entry of lymphocytes into this organ and the avascularity of the cornea may also limit antigen/lymphocyte interaction. The eye also has high conc entrations of TGF-beta which is a cytokine with immunosuppressive properties. Both the eye and the testis constitutively express Fas ligand. This membrane protein can bind to Fas on infiltrating leukocytes, and can induce apoptotic cell death in the Fas ex pressing cell. Liver transplantation may induce antigen -specific tolerogenic effects. The precise mechanisms are not clear, but have been attributed to the establishment of micro chimerism, the existence of tolerogenic donor immature dendritic cells or th eir precursors within the liver graft. In contrast, skin is highly immunogenic. Until modern HLA typing techniques developed, skin grafting was used as a clinical test before solid organ transplantation to determine the tissue type compatibility between t he donor and recipient.

60-Following a dislocated shoulder in a rugby match that gets out of hand, a patients axillary nerve is damaged. What is the likely result? Options: A. The teres major atrophies. B. The rounded contour of the shoulder disappears. C. A loss of sensation may occur in the lateral forearm. D. The patient may lose the ability to adduct the arm. E. The patient may have a wrist drop.

Teaching Notes for Question 60 Theme: Axillary nerve The deltoid atrophies when the axillary nerve (C5, 6) is damaged. Therefore, the rounded contour of the shoulder often disappears. A loss of sensation may occur over the epaulette area of the shoulder. 61-Steve, a 10-year-old boy presents with a chronic cough, recurrent lung infections and clinical evidence of obstructive pulmonary disease. His mother reveals that when he was 2 years old he was diagnosed as having malabsorption, and she has noticed that his s weat tastes excessively salty. All the above are features of this condition EXCEPT: Options: A. 510% of cases manifest with meconium ileus at birth B. In 70% of patients, the genetic abnormality is a four -base pair deletion at the locus on chromosome 7 C. Pancreatic abnormalities are present in 80% of patients D. The primary defect is in the transport of chloride ions across epithelial tissue E. Staphylococcus aureus and pseudomonas aeruginosa are two pathogens most commonly isolated from the sputum of patients with this condition

Teaching Notes for Question 61 Theme: Cystic fibrosis

182

PEDIATRICS EXAMINATION REVIEW Cystic fibrosis is an autosomal recessive disorder, caused by a mutation in th e gene encoding cystic fibrosis transmembrane conductance regulator (CFTR). The gene product is a cAMP regulated chloride transporter. The CFTR gene is found in region q31 -q32 on the long (q) arm of human chromosome 7. About 70% of mutations observed in CF patients result from deletion of three base pairs in CFTRs nucleotide sequence. This deletion causes loss of the amino acid phenylalanine located at position 508 in the protein; therefore, this mutation is referred to as deltaF508 CFTR. The disease incidence is about 1/2,300 and the average life expectancy is about 31 years. Abnormalities in epithelial cell chloride and water transport result in viscid secretions in the respiratory tract, pancreas, gastrointestinal tract, sweat glands, and other exocrin e tissues. Patients present with recurrent respiratory infections. Recurrent inflammation and chronic bacterial colonisation with organisms such as staphylococcus aureus, mucoid form of pseudomonas aeroginosa and rare microbes such as burkholderia cepacia result in extensive pulmonary structural damage, bronchiectasis, endstage lung disease and early death. Pancreatic insufficiency occurs in 75% 80% of patients. There is evidence of lipid malabsorption, and failure to thrive. Viscid gastrointestinal secretions may result in meconium ileus at birth and in distal intestinal obstruction syndrome later in life. Increased viscosity of bile with biliary ductule obstruction may ultimately result in cirrhosis. 62-A 26-year-old airhostess is brought to the A&E d epartment following a witnessed grand mal seizure. She has been previously fit and well except for a history of mild intermittent arthralgia affecting both hands. Her only medication is the OCP. She smokes 20 cigarettes per day and drinks 15 units of alc ohol per week. * * * * * On examination she has no focal neurology. There is slight puffiness of her fingers and she has low -grade pyrexia of 37.6C. BP 160/98 mmHg. Fundoscopy reveals silver wiring and AV nipping bilaterally. Urinalysis tests 3+, blood 2+ proteins.

Her blood test results reveal: Hb MCV WCC Platelets CRP ESR Sodium Potassium Urea Creatinine Albumin Tota l protein LFTs BM Clotting screen: PTT APTT VDRL Drug screen of urine Plasma alcohol

9.9 g/dl 83 fl 3.1 x 109/l 117 x 109/l 2 g/l 71 mm in the first hour 134 mmol/l 5.1 mmol/l 10.1 mmol/l 178 mmol/l 30 g/l 80 g/l Normal 7.8 mmol

11 seconds 39 seconds Positive 1:320 Negative Not detected

What is the likely diagnosis? Options:

183

PEDIATRICS EXAMINATION REVIEW

A. SLE B. Epilepsy C. Pulmonary embolism D. Syphilis E. Systemic sclerosis

Teaching Notes for Question 62 Theme: SLE The presentation with a fit in the absence of structural or metabolic derangement accompanied with pancytopaenia, arthralgia, and an elevated ESR with normal CRP is indicative of systemic lupus erythematosus (SLE). This patient has central nervous system (CNS) and renal lupus. The positive VDRL, prolonged APTT and PE are compatible with associated anti-phospholipid syndrome. AVN is a well-recognised complication of steroid treatment. 63-An elderly man presents to casualty with a wrist drop and pain radiating down his right arm. He recently visited his GP for a deep IM steroid injectio n. The probable site of the injection was the: Options: A. Posterior cord of the brachial plexus. B. The medial cord of the brachial plexus. C. The long thoracic nerve. D. The C5 spinal root. E. The ulnar nerve.

Teaching Notes for Question 63 Theme: Brachial plexus injury The brachial plexus has three cords, lateral, posterior and medial. Infraclavicular injuries may damage the cords. Lesions of the lateral cord affect the musculocutaneous nerve and the lateral root of the median nerve. This manifest as weakness of forearm flexion and pronation. The medial cord supplies the medial branch of the median nerve and the ulnar nerve. A lesion of this cord will cause to tal paralysis of the small muscles of the hand and have the same effect as a combined ulnar and radial nerve palsy. The posterior cord supplies the deltoid muscle and all extensors of the arm and fingers. Sensory loss would occur at the upper outer arm.

64-A boy presents with fever, arthralgia and headache and a red, annular rash in his axilla, which his mother says has been present for one week and has increased in size. Which is the most likely diagnosis? Options: A. Kawasaki disease B. Infectious mononucleosis C. Scarlet fever D. Lyme disease E. Rheumatic fever

Teaching Notes for Question 64 Theme: Most likely diagnosis This is a case of Lyme disease - caused by the spirochaete Borrelia burgdorferi. The vector

184

PEDIATRICS EXAMINATION REVIEW is the Ixodid species of tick. Lyme disease is the most common tick -borne illness in the United States and occurs widely throughout Europe and the former Soviet Union. The Ixodid tick is harboured by de er in the New Forest. Transmission is by a tick bite or by deposition of B. burgdorferi on the skin in faecal material. Incubation period is 3 - 32 days. The spirochaete may disseminate via the lymphatics or the bloodstream to the skin, central nervous sys tem, eye, muscle, heart, joints etc. The annular rash of Lyme disease (erythema chronicum migrans) is usually the first feature, and it tends to enlarge over 1-2 weeks, in association with non-specific symptoms such as fever, headache, regional lymphadenopathy, myalgia, arthralgia and malaise. Late disease presents weeks to months after inoculation. Features include VII palsy, heart block, uveitis, aseptic meningitis and chronic arthritis. Arthritis is caused by persistence of organisms in the synovium. Joint involvement is commonly asymmetrical and usually affects the large joints - knee, shoulder and elbow. Cardiac disease is relatively uncommon in children, but when present includes AV block, complete heart block and myopericarditis. 15% of patients present with neurological symptoms and signs. These include meningitis, encephalitis, cranioneuropathies, especially peripheral seventh nerve (Bell palsy)(3%), and rarely encephalitis. Diagnosis is confirmed by serology. Treatment is with doxycycline or amoxycillin or IM ceftriaxone if there is neurological involvement. 65-Milkmaid grasp is characteristically seen in which of the following conditions? Options A. Hemiballismus B. Sydenhams chorea C. Huntingtons disease D. Torticollis E. Gilles de la Tourette syndrome

Teaching Notes for Question 65 Theme: Milkmaid grasp Milkmaid grasp is a feature of Sydenhams chorea seen in Acute Rheumatic fever. Generally seen in children, this chorea has other clinical signs such as the spooning sign, pronator sign and darting motility of the tongue. 66-A 2-year old boy is admitted with fever. The presence of jaundice would make which one of the following diagnoses more likely? Options: A. Typhoid B. Malaria C. Toxocariasis D. Lyme disease E. Cat scratch fever

Teaching Notes for Question 66 Theme: Most likely diagnosis of fever and jaun dice. Typhoid usually presents with fever, malaise, headache and constipation. Malaria causes jaundice because of red cell haemolysis and should always be considered in a child with fever who has been abroad in the past 3 months. Toxocariasis (organism: the nematode Toxocara canis from dogs, or Toxocara catis from cats) presents in one of 2 forms: * Visceral larva migrans is a syndrome of fever, hepatomegaly, wheeze and eosinophilia caused by the immune response to migrating larvae. * Ocular larva migrans causes a granulomatous reaction in the retina where the larva dies and may be detected on fundoscopy in a child being investigated for strabismus or decreased visual acuity. Lyme disease (organism: Borrelia burgdorferi) usually presents with skin lesio ns (erythema chronicum migrans), fever, arthralgia, and headache. Disease can localise to heart, CNS or

185

PEDIATRICS EXAMINATION REVIEW joints. Cat scratch fever (organism: gram negative bacillus Bartonella henselae) presents with (usually cervical) lymphadenopathy and a history of a ca t scratch. 67-A two-month-old child had his first DTP injection 3 days previously and his mother has returned because his thigh is swollen. On examination, most of the anterolateral surface of the thigh is red and indurated. What will you recommend for t his child in the future? Options A. He should not have any further immunisations B. He should have all immunisations except pertussis C. He should receive the acellular pertussis vaccine when next immunised D. He should receive the oral pertussis vaccine when next immunised E. He should receive the whole cell pertussis vaccine (as DTP) as usual at his next immunisation

Teaching Notes for Question 67 Theme: Pertussis vaccination This degree of thigh swelling constitutes a severe local reaction (official definition: 'an extensive area of redness which becomes indurated and involves most of the anterolateral surface of the thigh or a major part of the circumference of the upper arm'). In the case of a severe local reaction, the acellular pertussis vaccine should be administered the next time. This is composed of a selection of proteins from pertussis and is said to have fewer side effects than the whole cell pertussis vaccine (which is composed of whole killed organism). A severe generalised reaction is defined as any of the following: * A temperature >39.5C within 48 hours of pertussis vaccination * Seizures or encephalopathy within 72 hours of pertussis vaccinatio n * Prolonged unresponsiveness or collapse * Anaphylaxis or bronchospasm * Inconsolable screaming for more than 4 hours In the case of a severe generalised reaction, diphtheria and tetanus ONLY should be given next time. There is no oral pertussis vaccine. Any oral vaccine will be a live vaccine (e.g. oral typhoid, oral polio (Sabin)). 68-You see a 15-year-old girl in clinic with the following results: * Hepatitis B surface antigen: positive * Antibodies to hepatitis B surface antigen: negative * Hepatitis B 'e' antigen: negative * Antibodies to hepatitis B core antigen: IgG positive * Antibodies to hepatitis B 'e' antigen: positive Which of the following statements best describes her clinical condition? Options: A. Acute hepatitis B infection B. Past hepatitis B infection C. Chronic hepatitis B infection (low infectivity) D. Chronic hepatitis B infection (high infectivity) E. Previously vaccinated against hepatitis B

Teaching Notes for Question 68 Theme: Hepatitis B This girl is HBsAg positive which means she is currently infected with hepatitis B (if she had cleared the infection she would be HBsAg negative and would have antibodies to HBsAg). She has antibodies to the core antigen, which will always be found in people who have had hepatitis B infection (either currently or in the past). If this was acute hepatitis B infection one would expect her to be IgM positive, or to not yet have formed antibodie s to the core

186

PEDIATRICS EXAMINATION REVIEW antigen; the fact that she is IgG positive suggests she has chronic infection. The 'e' antigen status determines the infectivity of the carrier. e -antigen positive (but antibody negative) means high infectivity; antibodies to the e antig en (but no e antigen detected) as in this case, signifies low infectivity. This has relevance if this girl becomes pregnant. Babies born to mothers with high infectivity require passive and active immunisation (HBIg and vaccine); babies born to mothers with low infectivity require vaccination alone. 69-In the syndrome of inappropriate ADH secretion (SIADH): Options: A. Plasma sodium is usually normal B. Plasma urate levels are elevated C. Oedema may occur due to fluid overload D. Urine osmolality is >500 mosm/kg E. Serum globulins are high

Teaching Notes for Question 69 Theme: Syndrome of inappropriate ADH secretion (SIADH) SIADH produces inappropriate water re tention despite low plasma osmolality. Therefore, serum levels of Na, K, urea, creatinine, and urate are all lowered. However, plasma protein levels are normal, so dependent oedema is rare. Urine is inappropriately concentrated. Causes include tumours sec reting ADH (small cell lung cancer, prostate, pancreas, thymus, lymphoma), drugs, CNS injury, including Guillain -Barr syndrome, lung infections, and acute intermittent porphyria (AIP). 70-A 14 year-old boy is seen in the Ambulance and Emergency Departmen t. He is unconscious and he has a rash around his mouth. The following blood test results are found. pH 7.29 , Pco2 4.5, HCO3 15, Cl 110, K 5, Na 135. What has he taken? Options: A. Methanol B. Ethylene glycol C. Opiates D. Glue E. MDMA

Teaching Notes for Question 70 Theme: Acidosis Drugs causing high anion gap acidosis include salicylates, methanol, ethylene glycol, and paracetamol. Impaired hepatic excret ion of lactate, or conversion to acids metabolically can also result in acidosis with high anion gap. Drugs causing normal anion gap acidosis include amphotericin, acetazolamide, and solvents (toluene) *glue*. Another cause of normal anion gap acidosis is renal tubular acidosis. Glue poisoning usually gives rise to a distal (type 1) RTA, therefore a hyperchloremic acidosis with a normal anion gap. 71-The most important cause of death in infectious mononucleosis is: Options: A. Liver failure. B. Kidney failure. C. Heart failure.

187

PEDIATRICS EXAMINATION REVIEW

D. Respiratory failure. E. Splenic rupture.

Teaching Notes for Question 71 Theme: Infectious mononucleosis The spleen is enlarged and sof t in consistency in this disease. If care is not taken while palpating the spleen, it may rupture and this is fatal in 25% of cases of infectious mononucleosis. 72-Acute attacks of porphyria: Options: A. Can be precipitated by paracetamol B. Can produce hyponatraemia due to Diabetes Insipidus C. Are a feature of porphyria cutanea tarda D. Are precipitated by the contraceptive pill E. Are associated with malabsorption

Teaching Notes for Question 72 Theme: Porphyria Acute attacks do not occur with Porphyria cutanea tarda (PCT), Congenital porphyria (CP) and Erythropoetic Protoporphyria (EPP). Hereditary coproporphyria (HCP) and Variegate porphyria (VP) have skin m anifestations and acute attacks. AIP is the only one without skin manifestations. Paracetamol, aspirin and opiates are safe analgesics in porphyria. Variegate porphyria is associated with pigmentation, hirsuitism and blisters as well as acute attacks. Abdominal pain, diarrhoea, constipation, and nausea can all occur in acute attacks. Porphyria rarely presents before puberty and occasionally first presents with an acute acttack triggered by the OCP. Malabsorption is not a feature of porphyria. 73-A 17-yr-old male medical student presents with a sudden onset of tunnel vision. On examination, he reports having only central vision over an area of approximately 1 m by 1 m. He reports this at a distance of 3 m and also at 6 m. Which of the following diagnoses b est fits his symptoms? Options: A. Optic neuritis B. Anterior uveitis C. Methanol poisoning D. Non -organic symptoms E. Retinitis pigmentosa

Teaching Notes for Question 73 Theme: Visual fields Tunnel vision should show an increasing area of vision with distance. A fixed area implies non-organic symptomatology. Retinitis pigmentosa will cause symptoms of tunnel vision. Optic neuritis will lead to blurring and ce ntral scotomas. Anterior uveitis tends to cause pain and blurred vision. Methanol poisoning begins with blurred vision, progressing to visual field contraction and sometimes complete blindness. 74-A middle aged lady presents to Casualty with a wrist dro p and pain radiating down her right arm after sustaining an injury to her neck, right arm and shoulder during a game of tennis. The probable site of the injury was the: -

188

PEDIATRICS EXAMINATION REVIEW

Options: A. Posterior cord of the brachial plexus B. Medial cord of the brachial plexus C. T1 spinal root D. C5 spinal root E. Ulnar nerve

Teaching Notes for Question 74 Theme: Brachial plexus lesions Wrist drop is indicative of a process affectin g the radial nerve. This arises from the posterior cord of the brachial plexus. The ulnar nerve innervates the small muscles of the hand while the medial cord gives rise to the median nerve. Accidental injection of spinal roots seems unlikely and would not cause a complete palsy. 75-A 26-year-old air hostess is brought to A&E following a witnessed grand mal seizure. She has been previously fit and well except for a history of mild intermittent arthralgia affecting both hands. Her only medication is the OCP , she smokes 20/day and drinks 15 units of alcohol per week. On examination she has no focal neurology. There is slight puffiness of the fingers and low -grade pyrexia of 37.6C. BP 160/98 mmHg. Fundoscopy reveals silver wiring and AV nipping bilaterally. Urine analysis tests show 3+ blood, 2+ protein. Results: Hb MCV WCC platelets CRP ESR Sodium potassium urea creatinine Albumin total protein LFTs BM Clotting screen: PTT APTT VDRL Drug screen of urine Plasma alcohol 9.9 g/dl 83 fl 3.1 X 109/l 117 X 109/l 2 g/l 71 mm in the first hour 134 mmol/l 5.1 mmol/l 10.1 mmol/l 178 mmol/l 30 g/l 80 g/l normal 7.8 mmol 11 seconds 39 seconds positive 1:320 negative not detected

The patient then develops acute dyspnoea and examinati on reveals decreased air entry at the right base, tachycardia, BP 105/55. O2 sats 94% on room air. She responds well to prednisolone and azathioprine, but at an OPD appointment 3 months later is complaining of worsening right hip pain poorly responsive to NSAIDs. X-ray is normal but bone scan shows increased uptake in the hip. What is the most likely cause of her hip pain? Options: A. Osteoporosis B. Avascular necrosis

189

PEDIATRICS EXAMINATION REVIEW

C. Septic arthritis D. Stress fracture E. Trochanteric bursitis

Teaching Notes for Question 75 Theme: SLE - Complications The presentation with a fit in the absence of structural or metabolic derangement accompanied with pancytopenia, arthralgia, a nd an elevated ESR with normal CRP is indicative of SLE. This patient has CNS and renal lupus. The positive VDRL, prolonged APTT and PE is compatible with associated anti -phospholipid syndrome. AVN is a well-recognised complication of steroid treatment. 76-A 35-year-old woman who complained of easy bruising, weight gain and striae is being worked up for Cushings syndrome. Her 24 -hour urinary free cortisol is 450 nmol. Which is the next investigation to confirm the diagnosis? Options: A. Low-dose dexamethasone suppression test B. High-dose dexamethasone suppression test C. Measuring cortisol levels D. Measuring ACTH levels E. CRH stimulation test

Teaching Notes for Question 76 Theme: Cushings syndrome * 48-hour low-dose dexamethasone suppression test is the investigation of choice to confirm the diagnosis in cases of Cushings syndrome in which urinary free cortisol is abnormally high. High -dose dexamethasone test is a valuable outpatient screening test, whereas the low dose dexamethasone test is conducted while the patient is admitted in the hospital. 0.5 mg of dexamethasone is given every 6 hours till 8 doses are completed and suppression results are noticed. I f it is not suppressed then the diagnosis of Cushings syndrome is made after taking the urinary cortisol results into account. * Cushings disease is the cause of Cushings syndrome in around 60% of cases, adrenal neoplasms are responsible for around 30% , and ectopic ACTH production is responsible for the remaining 10%. * Prognosis is poorest in patients where the cause is malignant adrenal neoplastic disease, those with surgically amenable lesions tend to do well. * Commonest clinical features of Cushi ngs include hypertension, central obesity with striae, hirsuitism, skin fragility, emotional lability, osteoporosis, diabetes mellitus and muscle wasting. 77-A 26-year-old woman is admitted to hospital for cadaveric renal transplantation. The operation is uneventful however she is admitted to the Intensive Therapy unit for 48 hours postoperatively. She is given treatment with prednisolone and cyclosporin, however 10 days after admission she develops a fever, diarrhoea, headaches and a non -productive cough. Examination reveals no focal signs of infection. Blood cultures are taken and a CXR is unremarkable. She is treated with cefotaxime, however after 48 hours the fevers continue and a repeat CXR shows left lower lobe consolidation. Options: A. Streptococcus pneumoniae B. Klebsiella pneumoniae C. Pneumocystis carinii D. Cytomegalovirus E. Cryptococcus neoformans

190

PEDIATRICS EXAMINATION REVIEW

Teaching Notes for Question 77 Theme: Nosocomial pneu monia Although Streptococcus pneumoniae should be considered as one of the most likely causes of postoperative pneumonia in an immunosuppressed patient, a response to Cefotaxime would be expected within 48 hours. This raises the possibility of atypical org anisms, however this soon after a transplant P. carinii, CMV and C. neoformans are very unlikely. Although coliforms are frequently isolated from the sputum of hospitalised patients, particularly those who may have had many previous courses of antibiotics, they should be considered as a cause of invasive infections, particularly in individuals who had been nursed in renal and high dependency units, resistance to 3rd generation cephalosporins is common in the noscomial setting. 78-A 6 week old male baby presents with a 2 day history of multiple seizures. He has been vomiting non-billious breast milk for the last 3 weeks. On examination the child is afebrile and dehydrated with a depressed anterior fontanelle. Ultrasound of the abdomen shows hypertrophic pyloric stenosis. The intravenous (iv) fluid of choice for administration would be: Options: A. iv 50% calcium gluconate B. iv isolyte P C. iv 5% dextrose D. iv sodium bicarbonate E. iv Hypertonic saline

Teaching Notes for Question 78 Theme: Treatment of electrolyte imbalance This child presents with a history of seizures that are of recent origin and vomiting for 3 weeks. He is dehydrated and afebrile with a depres sed anterior fontanelle (AF). Two major diagnostic considerations are:1.)Dyselectrolytaemia 2.)Septicaemia and meningitis A septic screen should be performed and antibiotics commenced. Obviously serum electrolytes will be measured. Hypochloraemic, hypokalaemic, metabolic alkalosis is the classic electrolyte and acid -base imbalance associated with the non -bilious vomiting of pyloric stenosis. The dehydration may result in hypernatraemia or hyponatraemia and there may be pre -renal renal failure. Metabolic alkalosis causes a decrease in ionised calcium because of increased binding of calcium to plasma proteins. Infusion of calcium gluconate would not correct the metabolic abnormality. Infusion of 5% dextrose, sodium bicarbonate or hypertonic saline woul d also not address the basic electrolyte imbalance. Intravenous (iv) Isolyte P consists of 5% dextrose with Na, K and Cl supplementation, and is used in the treatment of paediatric dehydration. Replacement should be guided by the clinical response and the electrolyte balance. 79-Regarding Legionnaire's disease:Options: A. Strict barrier nursing is essential to contain the infectious agent B. Preexisting immunocompromise is a risk factor C. Rifampicin alone is as effective as erythromycin for the treatment D. Pontiac fever is a rarer and milder form of the disease E. Diagnosis can be made from stool culture

191

PEDIATRICS EXAMINATION REVIEW Teaching Notes for Question 79 Theme: Legionnaire's dis ease Legionnaire's disease is usually due to infection with Legionella pneumophila type 1. It is a relatively common cause of community-acquired and nosocomial pneumonia in adults much less common in children. Clinical differentiation from the other caus es of community acquired pneumonia is not possible. For example, the incidence of diarrhoea is similar in Legionnaire's disease, pneumococcal pneumonia and mycoplasma pneumonia. However cerebellar ataxia, peripheral neuropathy and poor memory are recognise d neurological complications and are probably mediated by a cerebral toxin (in addition to the effects of hyponatraemia). Male sex, smoking, high alcohol intake, pre -existing immunocompromise and COPD are risk factors. Rifampicin alone increases resistance and is not as effective as 3 weeks of erythromycin. Pontiac fever is a 'flu -like' illness that is more common than full blown Legionnaire's disease, affects younger patients, has a shorter duration, epidemic tendency, no association with underlying diseas e and zero mortality. 80-A 15 year old boy is admitted with pneumonia. Investigations show hyponatraemia and mildly deranged LFTs. CXR shows hazy shadowing in the right mid and lower zones. Therapy is started and 5 days later he becomes acutely jaundiced with red discolouration of the urine. Which of the following drugs is the likely cause : Options: A. Cefotaxime B. Penicillin C. Rifampicin D. Ciprofloxacin E. Tetracycline

Teaching Notes for Question 80 Theme: Drug induced jaundice (1) The patient was started on rifampicin for suspected Legionella pneumonia. Rifampicin is a hepatic enzyme inducer and can lead to acute jaundice, and patient should be warned that secretions will turn red (particularly tears and urine). For Legionnaires' disease, rifampicin should be given with erythromycin as when given alone it tends to increase resistance. 81-Seven weeks after returning from rural Ghana, a 16 -year-old boy devel ops fever and rigors. The least useful test is: Options: A. Thick and thin blood film B. IgM for viral haemorrhagic fever C. Chest radiograph D. Blood cultures E. HIV antibody test

Teaching Notes for Question 81 Theme: Fever in a returning traveller Malaria is the most likely tropical diagnosis; therefore, multiple thick and thin blood films are essential. CXR and blood cultures are needed to rule out a respira tory infection or systemic sepsis. Viral haemorrhagic fever should be considered in the differential diagnosis if the patient has returned from an endemic area within the last 21 days; 7 weeks is too long, the incubation period is 221 days. 82-A 15-year-old is undergoing investigation for hirsuitism. Which of the following would not be implicated in the aetiology? Options:

192

PEDIATRICS EXAMINATION REVIEW

A. Cyclosporin A. B. Phenytoin. C. Moxonidine. D. Congenital adrenal hyperplasia. E. Cushing's syndrome.

Teaching Notes for Question 82 Theme: Hirsuitism Drugs causing hirsuitism include cyclosporin/phenytoin/minoxidil (not moxonidine -a centrally acting anti-hypertensive). Polycystic ovaries and congenital adrenal hyperplasia are associated with increased androgens and hirsuitism. Treatment is with anti-androgens (cyproterone, spironolactone), reducing free androgens (oestrogen therapy, OCP, weight loss), and cosmetic. 83-A family attends g enetic counselling. They have one child affected with Downs syndrome and are considering another pregnancy. Which of the following statements is most accurate? Options: A. Both parents should have karyotype analysis, but there is no need to karyotye the affected child B. Most children with Down's syndrome are born to women over the age of 35 years C. Amniocentesis should only be offered to confirm the diagnosis of Downs syndrome if the family will consider termination of the pregnancy D. A karyotype of 46XX excludes a diagnosis of Downs syndrome E. A nuchal scan together with bHCG and PAP (pregnancy associated protein) between 11 and 13 weeks of pregnancy gives an accurate risk of Downs syndrome in >80% of cases

Teaching Notes for Question 83 Theme: Trisomy 21 Although most cases of Downs syndrome occur because of a complete trisomy of chromosome 21, some will occur because of a balanced chromosomal translocat ion in one of the parents. The parents in these cases are phenotypically normal and have a complete complement of genetic material but the chromosomal arrangement is different. Because of this, they have a high recurrence risk for other children to be affe cted with Downs syndrome. Consequently, all families with a child with Downs syndrome should have basic chromosomal investigations performed. Women over 35 are at increased risk of having a child with Downs syndrome but because most children are born to mothers <35 years, most children with Downs syndrome are born to younger mothers. The triple test consists of measurements of alpha fetoprotein, bHCG and unconjugated estriol. It will identify about 6570% of those at high risk of carrying an infant with Downs syndrome. The nuchal scan together with bHCG and PAP identifies those at high risk of a Downs syndrome pregnancy in >80% of cases. 84-A 17 year old student develops a deterioration in vision in her left eye over three days. She complains of discomfort in the eye and thinks that difficulty with perception of colour was the first problem that she noticed, during a trip to an art gallery. On examination, visual acuity on the left is down to light perception. The pupil appears dilated and does not constrict to light, although does when a torch is shone in the right eye. What is the most likely diagnosis : Options: A. CMV retinitis B. Optic neuritis

193

PEDIATRICS EXAMINATION REVIEW

C. Anterior iscaemic optic neuropathy D. Amaurosis Fugax E. Central retinal artery occlusion

Teaching Notes for Question 84 Theme: Unilateral visual loss Causes of acute Unilateral loss of vision Acute angle glaucoma other eye is often affected Temporal arteritis Associated with headache, jaw claudication, scalp tenderness. High ESR. Usually affects both eyes. Central Retinal Artery Occlusion Acute, with maximal deficit at the onset and variable resolution. Red spot at macula. Amaurosis Fugax Transient blindness lasting minutes to hours. Associated with vascular risk factors as usually embolic. May be described as a curtain coming downAnterior Iscaemic Optic Neuropathy Painless, may be central field defect with colour disturbance a prominent feature. Arteriosclerosis, hypertension and diabetes are risk factors. Optic Neuritis Painful, colour vision affected. May not result in complete visual loss. May be the first manifestation of MS, therefore requires appropriate investigation. Infectious C auses CMV, Toxoplasmosis in association with immunodeficiency 85-A 29 year old Cypriot man gives a history of recurrent pain and swelling of the left knee. He also describes recurrent episodes of fever, pleuritic pain, and rash. He is then hospitalised with severe headache, photophobia and neck stiffness. He is lucid with a GCS of 15. His blood test results show: Hb 13.8g/dl plats 210 X 109/l WCC 4.9 X 109/l plasma glucose 6.5 mmol/l CRP 39g/l ESR 56 mm in the first hour RF, ANA, ENA and ANCA negative CSF analysis: opening pressure 14cmH2O protein 0.6 g/l RBC 2 /ml WBC 21/ml (100% lymphocytes) CSF glucose 4.1 mmol/l no organisms seen on microscopy 48 hour culture of CSF negative What is the diagnosis? Options: A. Familial Mediterranean fever B. Adult onset Still's disease C. Behcets syndrome D. Familial Hibernian fever E. Non Hodgkins lymphoma

Teaching Notes for Question 85 Theme: Familial Mediterranean fever (aseptic meningitis) Pleuritis can occur in Familial Mediterranean fever(FMF)whereas peritonitis occurs much more commonly. Aseptic meningitis is a recognised clinical manifestation of FMF. The splenomegaly and proteinuria is due to the development of amyloidosis, not an infrequent consequence of untreated FMF. Amyloidosis can complicate any condition characterised by a prolonged persistent acute

194

PEDIATRICS EXAMINATION REVIEW phase response. The Marie -nostrin gene is only found in 85% of patients with FMF. The SAP scan is a quantative investigation for the extent of amyloidosis. Colchicine given prophylactically in FMF is thought to offer some protection against the development of amyloidosis. 86-Which of the following do not tend to make the symptoms of myasthenia gravis worse? Options: A. Prednisolone B. Aminoglycosides C. Pyridostigmine D. Lithium E. Quinidine

Teaching Notes for Question 86 Theme: Myasthenia gravis Although prednisolone is used in the treatment of myasthenia, it may make the symptoms worse, initially. The main drugs to be regarded with caution are: * Aminoglycosides * Procainamide * Quinidine * Lithium * Phenytoin * Penicillamine * Contrast agents.

87-This 15-year-old was cyanosed on exertion. What is the diagnosis? Options: A. Pulmonary thromboembolic disease B. Fallots tetralogy

195

PEDIATRICS EXAMINATION REVIEW

C. Transposition of great arteries D. Eisenmengers syndrome E. Mitral stenosis

Teaching Notes for Question 87 Theme: TOF This is a classical boot-shaped heart with an absent pulmonary artery im pression and oligaemic lungs. 88-A patient has hypothyroidism. She develops a painful hand at night. In view of the probable aetiology, which muscle would you look for wasting? Options: A. Lateral two interossei B. Abductor pollicis brevis C. Medial two lumbricals D. Flexor digiti minimi E. Extensor pollicis

Teaching Notes for Question 88 Theme: Carpal tunnel. In the hand, the median nerve supplies the: * lateral 2 lumbricals, * opponens pollicis, * abductor pollicis brevis, and * flexor pollicis brevis - (LOAF). The other muscles of the hand are innervated by the ulnar nerve. 89-In a patient that has been previously diagnosed with Wilsons disease, which o f the following statements is true : Options: A. Serum cerruloplasmin levels are raised B. When penicillamine is administrated then urinary copper level rise C. Is inherited as an autosomial dominant trait D. May result in retinal abnormalities E. Most common presentation is with a manic depressive illness

Teaching Notes for Question 89 Theme: Wilsons disease Wilson's disease most commonly presents with extra -pyramidal features and dementia. Serum cerruloplasmin is low, Kaiser Fleisher rings appears at the periphery of the iris. Autosomal recessive inheritance, gene is on chromosome 13. 90-Which of the following statement(s) regarding childhood minimal -change nephrotic syndrome is/are true?: Options; A. On immunofluorescent studies, deposition is seen along the glomerular basement membrane B. Hypervolaemia is a common problem C. Albumin is routinely given to children with proteinuria D. Long-term cyclophosphamide therapy is beneficial in the steroid resistant group

196

PEDIATRICS EXAMINATION REVIEW

E. Minimal change disease is a histological diagnosis

Teaching Notes for Question 90 Theme: Nephrotic Syndrome * By definition this is minimal change; only Electron microscopy changes in the glomerular basement mambrane are seen in the form of increased spaces between the podocytes or foot processes. This makes the glomerulus more permeable and albuminuria occurs. * These children are far more likely to be hypovolaemic. * 4.5% albumin is given if the child is severely hypovolaemic (as resuscitation fluid).20% albumin is given for severe oedema, especially tense ascites, scrotal oedema. * Cyclophosphamide is beneficial but only as a short -term course over 8 weeks. The total cumulative (lifetime) dose of cyclophosphamide must not exceed 180mg/kg so as to avoid the side -effects of gonadal toxicity and late malignancies. * MCNS is a histological diagnosis the term steroid sensitive NS is used if a clinical diagnosis is made. PROTOCOL FOR MANAGEMENT OF NEPHROTIC SYNDROME: 1.Initial episode: Prednisolone 60 mg/m2/day (max 80 mg/day) x 4 -6 weeks, followed by 40 mg/m2 on alternate days for 4 wks 2.First two relapses: Prednisolone 60mg/m2/day (max 80 mg/day) until remission followed by 40mg/m2 on alternate days for 4 weeks 3.Frequent relapser: Maintenance Prednisolone 0.1 -0.5 mg/kg/alt days for 3-6 months, then slow reduction 4.Relapse on Prednisolone > 0.5 mg/kg/ alternate days Levamisole 2.5 mg/ kg /alternate days 5.Relapse on Prednisolone >0.5 mg /kg/alt days with steroid side effects or >1.0 mg/kg/alt days cyclophosphamide 2-3 mg/kg/day for 8 weeks 6.Post - cyclophosphamide relapses: As 2-3 above or Cyclosporin 5 mg/kg/day for one year Indications for renal biopsy in a child with nephrotic syndrome Age< 1yr (congenital nephrotic synd) or >16 yrs (adult disease pattern) Persistent HT Persistent or macroscopic hematuria Persistent low C3 or C4 Impaired renal function unresponsive to correction of fluid balance Before second line agents are started ie not steroid sensitive. What are the complications of Nephrotic Syndrome? Infectiondue to urinary loss of immunoglobulins loss of factor b of the alternate complement activation path loss of transferrin T cell abnormalities steroid immunosuppression Cellulitis (staph), peritonitis(pneumococcal), Septicaemia (gr negative ),chicken pox, measles Hypovolaemia- abdo pain, oliguria, cold extremities, increased toe core temp gap, poor cap refill, BP may be normal ,high (reactive HT) or low Raised Hb/Hct, low urinary Na (< 10 mmol/l) Thrombosis and embolism Hypercoagulability due to increased plasma fibrinogen, factor V & VII b y liver, decreased antithrombin III (lost in urine) + hypovolaemia, thrombocytosis, oedema & reduced mobility, diuretics Hypocalcaemia- due to loss of vitamin D binding protein in urine(also albumin loss - need corrected Ca++ Complications of treatment Cushingoid effects, poor growth, cataract - steroid(5% mortality), Bone marrow suppression (cyclophasphamide, cyclosporin) 91-A child has had his chromosomes analysed in the course of various investigations and the result is: 46 XY, t (2;5)(q35;p21. 3}.

197

PEDIATRICS EXAMINATION REVIEW

Which of the following statements is correct?: Options: A. He has more than 46 chromosomes B. The result shows all his genetic defects C. There is a translocation between the short arm of chromosome 2 and the short arm of chromosome 5 D. He is likely to be infertile E. There is an increased risk of him having a child with difficulties

Teaching Notes for Question 91 Theme: An abnormal karyotype The term karyotype is used to describe an individuals chromosomal complement. It will accurately describe visible abnormalities in chromosome number and arrangements such as translocations, deletions and duplications. It does not describe an individual's genetic defects. Every individual has two copies of over 40,000 genes and many of these will have mutations that are of no consequence to the individual. The karyotype 46 XY, t (2;5)(q35;p21.3) describes a male with a normal number of chromosomes but a translocation be tween the long arm of chromosome 2 (q) (sub band 35) and the short arm of chromosome 5 (p) (sub band 21.3). An individual with a balanced translocation such as this is likely to be asymptomatic but is at risk of having a child with difficulties. Y chromosome abnormalities are much more likely to result in infertility. 92-In leptospirosis which one of the following is true? Options: A. It is usually transmitted by rat bite B. It is a notifiable disease C. The patient may be jaundiced during the immune phase D. The organism may be isolated from urine during the septic phase E. Ciprofloxacin is the treatment of choice for severe forms

Teaching Notes for Question 92 Theme: Leptospirosis * Leptospirosis is a zoonosis transmitted by contact with water contaminated with rat urine. * Most patients suffer only a mild fever headache and muscle pains and the condition often goes undiagnosed. In a small number of cases t he disease presents as a septic illness 12 weeks after exposure. Weils disease is the eponym for the most severe presentation with shock and multi-organ failure. * The organism, Leptospira interrogans can be isolated from the blood or CSF during the septicaemic phase and from the urine during the 2nd week of illness. * Treatment options are IV penicillin/amoxicillin for 7 days in severely ill patients or doxycyline for milder cases. 93-A 9 months old baby girl is brought to you with failure to thrive a nd rickets. Her parents tell you that she has always been a very thirsty child, drinking 3 -4 bottles of water through the night, and having soaking wet napping, urine dipstick shows 1+ glucose, and 1+protein. Which of the following features are you likkely to find on examination and investigation. Options: A. Cataract. B. Corneal opacities. C. Hyperkalaemia. D. Metabolic acidosis with a wide anion gap.

198

PEDIATRICS EXAMINATION REVIEW

E. Nephrocalcinosis.

Teaching Notes for Question 93 Theme: Renal tubular acidosis. This is proximal renal tubular acidosis . Anion gap = Total number of cations in blood - Total numbers of anions. = (Na+ K ) (Cl +Hco3), Normal value = 8-12 Biochemical features: *Renal glycosuria,generalised aminoaciduria , bicarbonate wasting, reduced phosphate resorption, K wasting and tubular proteinuria, (B2 microglobulins, NAG,RBP). Renal tubular acidosis. *Proximal (Type II). -Bicarbonate wasting due t o low threshold. -Once plasma bicarbonate falls low enough urine PH < 5.5. -Can excrete acid in distal tubulee, don't get hypercalciuria or nephrocalcinosis. -Rx: large amounts of Nahco3 needed. *Distal (classical type). -Impaired h+ secretion in distal tubule can not produce acid urine, PH typically > 6, hypercalciuria, nephrocalcionosis. -Rx:small amount NaHco3 Hyperkalemic (TypeIV). -Aldosteron deficiency. -Pseudohypoaldosteronism. Fanconi syndrome. -Renal glycosuria. -Lwo Po4 bsorption -Hypophosphataemic rickets. -Generalized aminoaciduria (Tubular leak of all low molecular weight proteins). -bicarbonate wasting - renal tubular acidosis -Potassium wasting with hypokalaemia. -Large salt & water loss -polyuria +polydipsia. Fanconi syndrome Causes. -Cystinosis. -Galactosaemia. -Tyrosinaemia. -Hereditary fructosaemia. -Lows syndrome (Oculo -Cerebral- Renal syndrome). -Chemotherapy- esp 1 fosphamide. -Wilsons disease. -Heavy metal poisoning. -Drugs- esp amphetericin. -Idiopathic- large group. Cystinosis. -Autosomal recessive. -Lysosomal storage disorder - unable to transport cystine out of lysosome. -Diagnosis clinical suspicion, slit lamp, Wbc cystine level. -Antenatal diagnosis available. -All tissues in body involved. *Kidney-proximal renal tubular acidosis. *Eyes- corneal deposits. *Liver- impaired function. *Pancreas- diabetes. -Treatment , Phosphocyteamic. Increase cystine -transport out of lysosome, can delay onset of renal failure.

94-A 5 years old presents to casuality with a 2 months history of inguinal lymphadenopathy, his full blood count is normal , but there are small blasts in the blood film, a bone marrow aspirate confirm that he has acute lymphoblastic leukaemia. Which is the most appropriate initial course of action ? Options: A. Commonce chemotherapy straight away. B. Do a lymph node biopsy to exclude concomitant lymphoma. C. Referre him for a bone marrow transplantation.

199

PEDIATRICS EXAMINATION REVIEW

D. Start IV fluids & allopurinol. E. Discharge him and review in 1 week time as an out patient.

Teaching Notes for Question 94 Theme: Acute lymphoblastic leukaemia Acute lymphoblastic leukaemia is commoner in children, it generally presents with non specific symptoms but it is not uncommon to find lymphadenopathy on presentation . Poor prognostic factors include: Age (<1, >10), male sex, hypodiploidy and MLL gene rearrangements. Treatment, lasts for 3 years in boys & 2 years in girls and involves blocks of maintenance chemotherapy and intensive chemotherapy, all mostly given as outpatients . Tumoour lysis is common on induction of remossion and best prevented with IV fluids & allopurinol.

95- Which of the following is not a recognised complication of anticoagulation thera py? Options: A. Deep venous thrombosis. B. Thrombocytopaenia. C. Skin necrosis. D. Osteoporosis. E. Peripheral neuropathy.

Teaching Notes for Question 95 Theme: Anticoagulation. Although the main side effect of anticoagulation therapy is excessive bleeding, more unusual side effect exist. Heparin may be associated with both thrombocytopaenia and thrombosis as part of the disorder known as heparin -indused thrombocytopaenia (HITT). This is an acquired immune disorder in which heparin-dependant anti-p;atelet antibodies form, leading to sever thrombocytopaenia and occasionally catastrophic thrombosis. Treatment involves stopping heparin and anticoagulating with hirudin or danaparoids. Long-term heparin therapy may be associated with osteoporosis. Warfarin, especially induction therapy in patients with inherited protein C or S deficiency, may be associated with skin necrosis.

96-The mother of a baby in the postnatal ward has an antenatal screening test for sickle haemoglobin. This is positive deposite a normal Hb & MCV. The Hb -electrophoresis shows HbA 55% and Hbs 45%. Her partner has a microcytic anaemia, and Hb -electrophoresis shows an HbA band only with HbA2 quantita tion of 4.8%. Which diagnosis is impossible in the baby? Options: A. Sickle cell trait. B. Haematologically normal. C. B-thalassaemia trait. D. Sickle-B-thalassaemia. E. Sickle cell diseas(HbSS).

Teaching Notes for Question 96 Theme: Haemoglobinopathy.

200

PEDIATRICS EXAMINATION REVIEW The mother probably has a sickle cell trait ,the father has no evidence of Hbs & has a laboratory & Hb-electrophoretic profile compatible with B 0thalassaemia trait. The baby may therefore have : *Normal Hb. *Sickle cell trait. *B-thalassaemia trait . *Sickle-b-thalassaemia. It is not possible for the baby to have sickle cell disease (HbSS). 97-A 9 years old male has always wet the bed, he has no daytime enuresis, wh ich one of the following statement(s) regarding enuresis is not true? Options: A. Runs in families. B. There is a 30% chance that his father was a bed-wetter. C. Drinking tea causes bed-wetting D. At 15 years age, there is a 1% chance that he wi ll still bed-wet. E. If a child has tried many treatments over a number of years the prognosis is poor.

Teaching Notes for Question 97 Theme: Enuresis. Primary nocturnal enuresis. Some of the myths regarding enuresis are: *Medications are required in all cases. *Children with enuresis wet the bed deliberately. *The bladders of children with enuresis are smaller than other children.

98-A 18 years old student becomes un well with a sore throat, low grade fever and has some cervical lymphadenopathy, on consulting his GP 3 days after the onset of this illness, examination is otherwise unremarkable, investigations reveals: FBC normal ,Na=137 K=4.4 urea=10.1 ,creatinin =141 ,C3= 1.31g/l (0.55 1.20)C4= 0.42g/l (0.20-0.50),CH50= 87% (50-110) Urinalysis: blood 3+, protein +, few red cell casts. What is the most likely diagnosis? Options: A. post-streptococcal glomerulonephritis. B. Wegeners granulomatosis. C. Ig A nephropathy. D. Interstitial nephrosis. E. Henoch-Schonlein purpura.

Teaching Notes for Question 98 Theme: IgA nephropathy. IgA nephropathy (Bergers disease) typically affects young adult males, and immediately follows a viral infection or may have no clear precipitating factor. Prognosis is good, 10% develop renal failure, post -streptococcal GN normally follows a URTI after 2-3 weeks % is associated with a low C3 &coplement consumption where as C3 is normal/high in IgA nephropathy. Proteinuria rather than haematuria is more common in PSGN. There is no history of antibiotics or NSAIDs for interstitial nephritis and no history of arthralgia or purpura f or HSP

201

PEDIATRICS EXAMINATION REVIEW

99-An adolescent male is admitted with progressive limb weakness, on examination he has flacid weakness predominantly in the legs and absent reflexes. CSF examination shows markedly raised protein & a mild lymphocytosis, Which of the following does not imply a worse prognosis? Options: A. History of diarrhea in the preceding few weeks. B. presence of postural hypotension. C. Markedly reduced FVC. D. Evidence demyelination on nerve conduction studies. E. Sever weakness.

Teaching Notes for Question 99 Theme: Prognostic factors in Guillain Barre syndrome. The diagnosis in Guillain Barre syndrome. Denervation (rather thyan demyelination) show on nerve conduc tion studies implies a poor prognosis as do the remaining stems, in addition to old age. In arround 3% of cases death occurs .up to 10% of cases have sever residual disability.

100-Only one of the following is correct regarding physiology of the colon: Options: A. It is the site of most vitamin C absorption. B. Na and Cl are actively secreted. C. Mucous, K and bicarbonate are absorbed. D.Stomach distension increases colonic motility. E. Increased parasympathetic activity leads to decreased co lonic motility.

Teaching Notes for Question 100 Theme: Colonic physiology. Colonic functions include active absorption of Na and cl, and secretion of k, hco3 and mucous. Parasympathetic activity, distension (gastrocolic reflex), emotional factors and cholinergic agents increase colonic motility. Vitamin C is water -soluble (acscorbic acid) and most absorption is proximal to the colon.

202

PEDIATRICS EXAMINATION REVIEW

Questions
(PART 3)

SELECT THE MOST APPROPRIOT DIAGNOSIS

203

PEDIATRICS EXAMINATION REVIEW

1-Options : A. Osteomalacia B. Ankylosing spondylitis C. Avascular necrosis D. Rheumatoid arthritis E. Osteoarthritis F. Pagets disease Instructions: Choose the most appropriate disease for the radiological description. 1. Narrowing of joint space, marginal osteophytes and subchondral cysts. 2. Bamboo spine 3. Mixed osteoblastic and osteolytic lesions in the left tibia. There is also cortical thickening. 4. Joint space narrowing, juxtaarticular osteoporosis and bony erosions.
2-Options : A. Streptococcal throat infection B. Haemophilus influenza epiglottitis C. Pneumococcal infection D. Kawasakis disease E. Typhus F. Tuberculosis G. Infectious mononucleosis H. Erythema infectiosum I. Mumps J. Varicella zoster K. Rubella L. Measles Instructions: For each of the patients described below choose the single most likely option from the above list. Each option may be used once, more than once or not at all. 1. A 5 year old girl presents with conjunctivitis, fever and coryza 12 days after exposure to a sick sibling. On examination she has blue white punctate lesions on her buccal mucosa. 2. A 16 year old boy presents with fever, sore throat, anorexia, malaise. He has enlarged cervical lymph nodes. On investigation there is lymphocytosis with atypical lymphocytes. 3. A 6 year old girl presents with fever and a slapped cheek erythematous eruption on her cheeks. 4. A 8 year old boy presents with malaise, fever, and has painful swelling of both parotid glands. 5. A 6 year old boy presents with fever, cervical lymphadenopathy and erythema of palms and soles. 3-Options : A. Full blood count B. Marrow biopsy C. EEG D. CT scan E. INR F. Thyroid function tests

204

PEDIATRICS EXAMINATION REVIEW

G. Liver function tests H. Liver biopsy I. Prolactine levels J. Drug serum levels K. Breast ultrasound Instructions: Drug side effects-investigations; give the most appropriate investigation for each scenario. Each answer may be used once, more than once or not at all. 1. Patient on phenytoin, with diplopia, tremor, and low mood 2. Patient on Sodium Valproate, with vomiting, anorexia, jaundice, drowsiness 3. Patient on carbamazepine feeling unwell, with sore troat, and temp. 37.2 C 4. Male patient on phenothiazines comes with discharge from his breast 4-Options A. Down's syndrome B. Turner's syndrome C. Pataus syndrome D. Cri-du-chat syndrome E. Cystic fibrosis F. Kleinfelter's syndrome G. Kallmanns syndrome H. Silver Russell syn drome Instructions: From the given list of options, choose the single best answer for the given set of questions. Each option maybe used once, more than once or not used at all. 1. A young mother brings her child to the paediatric OPD as she noticed h e has absence of skin on his scalp area. O/E- micrognathia, microcephaly, umbilical hernia and a pansystolic murmur of VSD are present. 2. A 5-year-old child is brought to the hospital with complaints of repeated chest infections and GI problems since birth. His sweat chloride levels are diagnostic. 3. A tall adult male, with absence of secondary sexual characters, anosmia, cleft lip and palate and mostly normal mental development. 4. A young female, with delayed puberty, cubitus valgus, heart defe cts and mild mental retardation. 5. A mother brings her 2 year old with respiratory infection to the OPD. O/E he has simian crease, retarded physically and mentally, large tongue, and a VSD. 5-Options A. Duodenal atresia B. Jejunal atresia C. Oesophageal atresia D. Necrotising enterocolitis E. Malrotation F. Malrotation with volvulus G. Hirschprung's disease H. Intussusception Instructions: Please match the most likely diagnosis with the scenario described below 1. A baby girl born at 26 weeks is transferred to NICU on day 3 with abdominal distention, blood in the stools and increasing ventilatory requirements

205

PEDIATRICS EXAMINATION REVIEW

2. A baby girl born at 36 weeks is transferred to NICU on day 1 with cyano sis and respiratory distress after attempted feeding 3. A baby boy born at term is transferred to NICU on day 2 with abdominal distension and failure to pass meconium. 4. A baby boy born at term is transferred to NICU on day 10 with sudden collapse. He has bilious aspirates, some blood in the stool and a silent abdomen. 5. A baby boy born at 32 weeks is transferred to NICU on day 1 with bilious vomitting. On examination, he has features of Trisomy 21. 6-Options A. Atrial septal defect B. Ventricular septal defect C. Patent ductus arteriosus D. Coarctation of aorta E. Ebstein anomaly Instructions: Match the correct physical sign/presenting symptoms with the lesion of congenital heart disease:1. A 17 year old presents with headache, leg fatigue and claudication. On examination there is an absence of femoral pulses with a mid -systolic murmur heard over the back. 2. A 18 year old presents for a health checkup with no symptoms. On examina tion there is a widely split fixed 2nd heart sound. 3. An 18 year old presents with palpitations. On auscultation there is a quintuple cardiac cadence. 4. A 7 year old presents with breathlessness. On auscultation there is a continuous murmur. 5. A 3 year old presents with breathlessness and pedal oedema. On auscultation there is a sharp holosystolic murmur heard along the left sternal border. 7-An 18-year-old man was admitted to casualty at 3 am having collapsed at a nightclub. A friend, who h ad accompanied him in the ambulance but subsequently left A and E before speaking to a doctor, told the paramedics that his friend had been well earlier in the evening and that he usually did not drink much alcohol or take any drugs. Investigations revealed: Haemoglobin 8.6 x 109/L White cell count Neutrophils Lymphocytes Monocytes Eosinophils Basophils Platelets Reticulocyte count Prothrombin time Fibrinogen D-Dimer screen Serum sodium Serum potassium Serum urea Serum creatinine Lactate dehydrogenase 25.8 x 109/L 22.5 x 109/L 2.0 x 109/L 0.8 x 109/L 0.4 x 109/L 0.1 x 109/L 32 x 109/L 12% 32 s 0.3 g/L 3.0 mg/L 140 mmol/L 6.3 mmol/L 17.5 mmol/L 200 mmol/L 540 IU/L

Activated partial thromboplastin time 90 s

The blood film showed red cell fragmentation with polychromasia, toxic granulation of neutrophils and platelet anisocytosis. What is the diagnosis?

206

PEDIATRICS EXAMINATION REVIEW

Options : A. Autoimmune haemolysis B. Autoimmune thrombocytopaenia C. Disseminated intravascular coagulation D. Drug-induced haemolysis E. Thrombotic thrombocytopenic purpura

Choose 1

8-Options A. Intussusception B. Mesenteric adenitis C. Meckel's diverticulum D. Appendicitis E. Strangulated inguinal hernia F. Hypertrophic pyloric stenosis G. Constipation H. Nephroblastoma I. Riedel's lobe of liver Instructions: Choose the single most likely diagnosis from the options listed above for each of the clinical scenarios described below. Each option may be used once, more than once or not at all. 1. A 4 year ol d boy is brought into hospital by his mother. For the past 2 weeks he has had a cold, a sore throat, headache and nausea. Today he has vomited twice, is not tolerating fluids, and complains of central abdominal pain. 2. A 2 year old girl is referred urg ently to clinic having presented to her general practitioner with haematuria on two occassions, and an abdominal mass. She is now asymptomatic. There is a firm mass with a poorly defined smooth lower border in the left upper quadrant of her abdomen. On percussion of the liver there is no dullness, and on palpation the liver is non -tender. 3. A 13 year old girl presents with a complaint of abdominal pain that she has experienced for the past two weeks, which has worsened acutely today. History reveals a b right red loss of blood per rectum with defaecation, which she notes on the toilet paper. Her abdomen is soft on palpation and on auscultation there are normal bowel sounds. A per rectum examination is too tender to perform and an anal fissure is noted at 6 o'clock. 4. A 1 year old child has been noted to have bouts of screaming where he is doubled -up in agony from abdominal pain with a 60 minute pain -free period in between. When you ask to see his nappy, you note a redcurrant jelly stool, with a trace o f blood on his anus. 9-A 18-years-old adolescent had a bone marrow transplant from his sister for treatment of acute myeloid leukemia. Six weeks later he presented with a rash, general malaise, poor appetite and chronic watery, green diarrhoea. His med ications included cyclosporin, penicillin V, co-trimoxazole and aciclovir. On examination, a diffuse macular rash was visible, mainly affecting the palms of his hands and the soles of his feet. His pulse was 104 beats per minute (bpm) regular. Blood press ure 112/74 mmHg. Heart sounds were unremarkable and his chest was clear. His abdomen was soft apart from mild tenderness in the right upper quadrant. No organs were palpable. Bowel sounds were active. Neurological examination was normal. Investigations re vealed: Haemoglobin 12.4 g/dL Mean corpuscular volume (MCV) White cell count Neutrophils Lymphocytes Monocytes 90 fL 3.65 x 109/L 1.7 x 109/L 1.0 x 109/L 0.8 x 109/L

207

PEDIATRICS EXAMINATION REVIEW

Eosinophils Basophils Platelets Serum albumin Serum total bilirubin Serum alanine transferase Serum alkaline phosphatase Serum gamma glutamyl transferase What is the diagnosis? Options:

0.05 x 109/L 0.1 x 109/L 147 x 109/L 36 g/L 64 mol/L 90 IU/L 135 IU/L 40 IU/L

Choose 1

A. Acute graft versus host disease B. Clostridium difficile infection C. Cytomegalovirus infection D. Side-effect of immunosuppression E. Transfusion-associated graft versus host disease 10-A 14 year old girl was referred to the local child psychiatrist by her GP who was concerned about her weight loss. The GP reported that she has lost about 2 stones over the past 2 months and that although she was preoccupied by preparing food, she did not seem to be eating well. Her mother reported that she has been hording food in her bedroom. The mother also informed the GP that despite losing weight her daughter was spending long time every day undertaking heavy exercises. Which of the following supports a diagnosis of anorexia nervosa in this patient: Options: A. Loss of over 15% of body weight B. Binging on fatty food C. Increased levels of growth hormone D. Increased levels of cortisol E. Absence of depressive features F. Amenorrhea G. Use of diuretics 11-Options A. Spinal X-ray and HLA profile B. Bone scan C. MRI D. Anti dsDNA and complement E. Thomas test F. Pelvic spring test G. Sigmoidoscopy, biopsy and enema H. Joint aspiration and microscopy I. Clinical history alone, joint X-ray J. Joint aspiration, swab and blood cultures K. Rheumatoid factor, joint X-ray L. ECHO M. Neurophysiology Instructions: For each of the patients described below choose the single most likely option from the above list of options. Each option may be used once, more than once or not at all. 1. A 5-year-old Asian refugee child develops high fevers, organomegaly, joint pains and a rash. He also has a 4/6 murmur. 2. A 45-year-old man who was on a fast developed acute pain in his toe. Choose 5

208

PEDIATRICS EXAMINATION REVIEW

3. A 45-year-old woman presents with wrist pain, which is worse at night and relieved by dangling the hand over the edge of the bed and shaking it. 4. A 45-year-old man presents with swollen, painful and stiff hands and feet and the symptoms are worse in the mornings. 5. A man has a bike accident and injures his leg. There is a small open wound. The wound is treated but 2 days later he develops pain in his knee. 12-Options A. Varicella B. Scabies C. Pityriasis rosea D. Psoriasis E. Meningitis F. Measles G. Kawasakis disease H. Herpes simplex virus I. Ebstein-Barr virus J. Roseola infantum K. Impetigo L. Erythema toxicum Instructions: For each of the patients described below choose the single most likely option from the above list of options: Each option may be used once, more than once or not at all. 1. A 6-month-old boy presents with a generalised rash after a bout of fever. 2. A 3-year-old boy presents with oedema, congested conjunctivae, erythema of the oral cavity, lips, and palms. It does not respond to antibiotics. 3. A mother rings A&E saying that her 3 -year-old boy has fever for 1 day and is drowsy. He also has a rash which does not blanch on pressure. 4. A 17-year-old African male presents with sore throat, fever, anorexia, malaise, lymphadenopathy, palatal petechiae, and splenomegaly. 5. A non-immunised child of 5 years presents with catarrh, conjunctivitis, fever. He has some bluish white spots on his buccal mucosa. 13-Options : A. Deformation B. Disruption C. Malformation D. Dysplasia E. Teratogen F. Dystocia Instructions: Match the above options with the be st descriptive statement from the choices given below:1. Chemical agent with a malign influence on development 2. Abnormal development of a structure caused by extrinsic forces 3. Intrinsic defects in the pattern of development 4. Destruction of normally formed tissue 5. Abnormal organisation of cells in tissue 14-Options :

209

PEDIATRICS EXAMINATION REVIEW

A. Herpes virus B. Streptococcus C. Staphylococcus D. Brucellosis E. Tuberculosis F. Poliomyelitis G. Candidiasis H. Infectious mononucleosis I. Rubella J. Diphtheria K. Coryza L. Meningitis M. Hepatitis N. Influenza Instructions: For each of the patients described below choose the single most likely option from the above list of options: (Each option may be used once, more than once or not at all.) 1. A 4-year-old boy presents to A&E with shock. He is found to have myocarditis. His mother gives a history of tonsillitis. 2. A 15-year-old patient presents with a rash and suboccipital ly mphadenopathy. He was not immunised as a child. 3. A 35-year-old lady presents with myalgia, nausea, and fever. She also has abdominal pain. Her bilirubin is raised. 4. A veterinarian presents with fever, malaise, anorexia, vomiting, hepatosplenomega ly, depression, arthritis, sacro -ileitis and rash. 5. A man who is on steroids for rheumatoid arthritis for a long period of time now presents with white patches in his mouth.

210

PEDIATRICS EXAMINATION REVIEW

Answer &Teaching Notes for Questions


(PART 3)

211

PEDIATRICS EXAMINATION REVIEW

1-Options : A. Osteomalacia B. Ankylosing spondylitis C. Avascular necrosis D. Rheumatoid arthritis E. Osteoarthritis F. Pagets disease Instructions: Choose the most appropriate disease for the radiological description. 1. Narrowing of joint space, marginal osteophytes and subchondral cysts. 2. Bamboo spine 3. Mixed osteoblastic and osteolytic lesions in the left tibia. There is also cortical thickening. 4. Joint space narrowing, juxtaarticular osteoporosis and bony erosions.
Teaching Notes for Question 1 Theme: Radiographic abnormalities in rheumatological disease Rheumatoid arthritis and osteoarthritis features are as what given in the question. Ankylosing spondylitis affects the spine in young adults and causes restricted spinal mobility. In late stages X-ray shows bamboo spine where vertebrae are fused. Pagets disease is due to a disorder of bone remodelling. It is usually monoostotic (affects one bone). Most important biochemical abnormality is elevated alkaline phosphatase and normal calcium levels.

2-Options : A. Streptococcal throat infection B. Haemophilus influenza epiglottitis C. Pneumococcal infection D. Kawasakis disease E. Typhus F. Tuberculosis G. Infectious mononucleosis H. Erythema infectiosum I. Mumps J. Varicella zoster K. Rubella L. Measles Instructions: For each of the patients described below choose the single most likely option from the above list. Each option may be used once, more than once or not at all. 1. A 5 year old girl presents with conjunctivitis, fever and coryza 12 days after exposure to a sick sibling. On examination she has blue white punctate lesions on her buccal mucosa. 2. A 16 year old boy presents with fever, sore throat, anorex ia, malaise. He has enlarged cervical lymph nodes. On investigation there is lymphocytosis with atypical lymphocytes. 3. A 6 year old girl presents with fever and a slapped che ek erythematous eruption on her cheeks. 4. A 8 year old boy presents with malaise, fever, and has painful swelling of both parotid glands.

212

PEDIATRICS EXAMINATION REVIEW

5. A 6 year old boy presents with fever, cervical lymphadenopathy and erythema of palms and soles. Teaching Notes for Question 2 Theme: Diagnosis of childhood illness. Q1. This is a case of measles caused by an RNA paramyxovirus spread by droplets. Incubation period is 8 to 14 days. Measles infection presents with catarrh, conjunctivitis and fever. Koplik spots are pathognomonic 'grain of salt like' spots on the buccal mucosa located adjacent to the posterior molar. They are seen early in the illness. A maculopapular rash typically first appears behind the ears and spreads to involve the face and then the trunk and limbs. Complications are due to infection by the measles virus itself -respiratory- laryngotracheobronchitis, bronchitis, giant celll pneumonia - ear - otitis media -CNS - postinfectious encephalomyelitis. This severe complication usually manifests 7 -14 days after initial presentation with drowsiness, seizures, focal neurological signs and coma. Mortality is 10% and 15% of patients who recover will have residual neurological deficit. - subacute sclerosing panencephalitis (SSPE) is a rare complication caused by latent measl es virus infection which on reactivation 5 -10 yrs later results in a degenerative CNS disorder. Secondary bacterial infection is an important complication of measles infection particularly in malnourished children. Those wishing to update themselves on the current debate about measles vaccine and autism should read the section and refer to the journals about measles vacccine at web address http://www.doh.gov.uk/mmr/index.html. Q2. This is a case of infectious mononucleosis caused by Epstein Barr virus - a virus of the herpes group which infects B-lymphocytes. It presents usually in young adults with fever, sore throat, anorexia, malaise, lymphadenopathy, palatal petechiae, splenomegaly, hepatitis and haemolytic anaemia. A mild erythematous maculopapular rash occurs in 3-15% of patients - this number increases to 80% of patients if there has been previous treatment with amoxicillin or ampicillin. Peripheral blood count reveals lymphocytosis, and the smear will show atypical lymphocytes. These are CD8+ T-lymphocytes which are produced in response to EBV infected B -cells. Q3. This is a case of erythema infectiosum also known as slapped cheek syndrome. It is caused by parvovirus B19 a small single -strand DNA virus. Common symptoms include a mild prodromal illness consisting of fever, malaise, headache, myalgia, nausea, and rhinorrhea. On day 5 -7 after symptom onset, a bright red rash appears on the cheeks, often followed by an erythematous maculopapular rash on the trunk and legs. Resolution of the rash revea ls a lacy reticular pattern on the limbs. Complications include -arthritis- typically asymmetrical and transient -aplastic crises - seen in children with conditions which decrease red cell survival such as sickle cell anaemia, thalassaemia major, other h aemolytic anaemias. Q4. This is a case of mumps caused by a single stranded RNA paramyxovirus. The patient is infective 7 days before and 9 days after parotid swelling starts. Incubation period is 14 - 21 days. The infection presents with malaise, fever and painful swelling of the parotids. Complications are orchitis, arthritis, meningitis, pancreatitis and myocarditis. Q5. This is a case of Kawasaki disease Kawasaki disease is a vasculitis of unknown aetiology which generally occurs in those under 9 y rs of age. Diagnostic criteria include fever; bilateral non -exudative conjunctivitis; oral changes -(swollen red lips, strawberry tongue); erythema of the palms and soles (which desquamate in the second or third week of illness); polymorphous rash and cervi cal lymphadenopathy. The pyrexial phase of the illness lasts up to 10 days and is usually accompanied by intense irritability and fractiousness. Cardiovascular complications occur in the subacute phase of illness (+/_ day 10 to day 30). These include peric arditis, myocardititis and coronary artery aneurysm. Thrombocytosis is a late finding. There should be no delay in giving IVIG once the diagnosis (which is a clinical one) is made; it should be given within 10 days of the start of the illness. Mortality is around 1% for Kawasaki disease due to acquired cardiovascular disease.

213

PEDIATRICS EXAMINATION REVIEW

3-Options : A. Full blood count B. Marrow biopsy C. EEG D. CT scan E. INR F. Thyroid function tests G. Liver function tests H. Liver biopsy I. Prolactine levels J. Drug serum levels K. Breast ultrasound Instructions: Drug side effects-investigations; give the most appropriate investigation for each scenario. Each answer may be used once, more than once or not at all. 1. Patient on phenytoin, with diplopia, tremor, and low mood 2. Patient on Sodium Valproate, with vomiting, anorexia, jaundice, drowsiness 3. Patient on carbamazepine feeling unwell, with sore troat, and temp. 37.2 C 4. Male patient on phenothiazines comes with discharge from his breast Teaching Notes for Question 3 Theme: Drug(s) Q1. (J) Phenytoin levels vary in individuals; plasma concentration monitoring is advised on starti ng the treatment. Q2. (G) Liver failure is more common in children < 3 years old , but may occur in any age, LFT should be monitored in first 6 months of therapy Q3. (A) Blood dyscrasia with low WBC and frequent infections is a severe side effect. Q4. (I)Neuroleptics by blocking dopamine D2 receptors may give extrapyramidal effects and hyperprolactinemia. 4-Options A. Down's syndrome B. Turner's syndrome C. Pataus syndrome D. Cri-du-chat syndrome E. Cystic fibrosis F. Kleinfelter's syndrome G. Kallmanns syndrome H. Silver Russell syndrome Instructions: From the given list of options, choose the single best answer for the given set of questions. Each option maybe used once, more than once or not used at all. 1. A young mother brings her child to the paediatric OPD as she noticed he has absence of skin on his scalp area. O/E- micrognathia, microcephaly, umbilical hernia and a pansystolic murmur of VSD are present. 2. A 5-year-old child is brought to the hospital with complaints of repeated chest infections and GI problems since birth. His sweat chloride levels are diagnostic. 3. A tall adult male, with absence of secondary sexual characters, anosmia, cleft lip and palate

214

PEDIATRICS EXAMINATION REVIEW

and mostly normal mental development. 4. A young female, with delayed puberty, cubitus valgus, heart defects and mild mental retardation. 5. A mother brings her 2 year old with respiratory infection to the OPD. O/E he has simian crease, retarded physically and mentally, large tongue, and a VSD. Teaching Notes for Question 4 Theme: Chromosomal disorders 1. Pataus syndrome is trisomy 13. 2. Cystic fibrosis occurs due to abnormality in long arm of chromosome 7. 3. Kallmans syndrome is characterised with the features given in the question. 4. Turners syndrome is 45 XO karyotype. 5. Downs syndrome is trisomy 21. 5-Options A. Duodenal atresia B. Jejunal atresia C. Oesophageal atresia D. Necrotising enterocolitis E. Malrotation F. Malrotation with volvulus G. Hirschprung's disease H. Intussusception Instructions: Please match the most likely diagnosis with the scenario described below 1. A baby girl born at 26 weeks is transferred to NICU on day 3 with abdominal distention, blood in the stools and increasing ventilatory requirements 2. A baby girl born at 36 weeks is transferred to NICU on day 1 with cyanosis and respiratory distress after attempted feeding 3. A baby boy born at term is transferred to NICU on day 2 with abdominal distension and failure to pass meconium. 4. A baby boy born at term is transferred to NICU on day 10 with sudden collapse. He has bilious aspirates, some blood in the stool and a silent abdomen. 5. A baby boy born at 32 weeks is transferred to NICU on day 1 with bilious vomitting. On examination, he has features of Trisomy 21. Teaching Notes for Question 5 Theme: Neonatal intestinal obstruction 1. NEC is a disease of prematurity and impaired gut perfusi on is thought to be responsible for the clinical picture. 2. Oesophageal atresia has an incidence of about 1 in 2500 live births. There is usually an associated tracheo-oesophageal fistula (90%). There is often polyhydramnios in pregnancy. The child prese nts with frothing from the mouth, choking, coughing with episodes of cyanosis exacerbated by feeding. There is a high incidence of aspiration pneumonia. 3. Hirschprung's disease is caused by the absence of ganglion cells in the myenteric plexuses of the distal bowel. Aganglionosis may be restricted to the rectum or rectosigmoid region - 'short segment Hirschsprung's disease', or may extend above the sigmoid - 'long segment Hirschsprung's disease'. In 'short segment Hirschsprung's disease', there is a highe r incidence in males (4:1) with an equal sex incidence in 'long segment Hirschsprung's' disease. The disorder shows a familial tendency, and there is an association with Down's syndrome (5 -15%) and cardiac malformation (2 -5%). Clinical symptoms are usually present within the first few days of life with delayed passage of meconium, abdominal distension, bilious vomiting, poor feeding and failure to thrive. Enterocolitis is a severe complication and presents with diarrhoea, gross abdominal distension and ci rculatory collapse. Children with 'short segment Hirschsprung's disease' can present later in life, with a history of chronic constipation since birth and poor weight gain. Barium enema shows a narrowed distal colon with proximal dilation. A second enema 2 4 hrs after the first shows retention of contrast and may reveal the transition zone between normal bowel and the distal aganglionic intestine. Rectal biopsy confirms the diagnosis.

215

PEDIATRICS EXAMINATION REVIEW

Treatment usually involves an initial colostomy proximal to the aganglion ic bowel. This is followed by a definitive pull through procedure at 3 -12 months of age. 4. During embryological development the intestines herniate into the umbilical cord at 4 - 10 weeks gestational age. They then return to the abdomen and rotate in an a nticlockwise direction. Failure of this normal process results in malrotation which results in a narrow mesenteric stalk which predisposes to midgut volvulus and intestinal obstruction. Babies present with vomiting, and in cases of volvulus with abdominal distension, rectal passage of blood and peritonitis. 5. Duodenal atresia occurs in 1 in 10,000 live births.The site of obstruction is most commonly in 2nd part of duodenum with hypertrophy of the proximal duodenum. There is an association with Down's syndrome (30%) as well as cardiac anomalies, malrotation and biliary atresia. Babies present with bilious or non-bile stained vomiting. X-ray shows the typical 'double -bubble'.Treatment is surgicalduodenoduodenostomy. Causes of bowel obstruction in the neon ate are: -atresia of any part of the gastrointestinal tract -duodenal, jejunoileal atresia, large bowel atresia -mechanical obstruction -meconium ileus, meconium plug -volvulus -malrotation -imperforate anus -failure of normal peristalsis - Hirschsprung's disease -intussusception may occasionally occur in the neonate Later in the neonatal period other conditions should be considered, including pyloric stenosis. 6-Options A. Atrial septal defect B. Ventricular septal defect C. Patent ductus arteriosus D. Coarctation of aorta E. Ebstein anomaly Instructions: Match the correct physical sign/presenting symptoms with the lesion of congenital heart disease:1. A 17 year old presents with heada che, leg fatigue and claudication. On examination there is an absence of femoral pulses with a mid -systolic murmur heard over the back. 2. A 18 year old presents for a health checkup with no symptoms. On examination there is a widely split fixed 2nd heart sound. 3. An 18 year old presents with palpitations. On auscultation there is a quintuple cardia c cadence. 4. A 7 year old presents with breathlessness. On auscultation there is a continuous murmur. 5. A 3 year old presents with breathlessness and pedal oedema. On auscultation there is a sharp holosystolic murmur heard along the left sternal border. Teaching Notes for Question 6 Theme: Congenital heart diseases Q1. This is a case of coarctation of the aorta. Narrowing or constriction of the lumen of the aorta may occur anywhere along its length but is most common distal to the origin of the left subclavian artery near the insertion of the ligamentum arteriosum. Most children and young adults with isolated, discrete coarctation are asymptomatic. Headache and epistaxis are symptoms. The extremities can feel cold and claudication with exercise may occur due to diminishe d blood flow from the aorta to the lower extremities. Attention is usually directed to the cardiovascular system when a heart murmur or hypertension in the upper extremities and absence, marked diminution, or delayed pulsations in the femoral arteries are detected on physical examination. Enlarged and pulsatile collateral vessels may be palpated in the intercostal spaces anteriorly, in the axillae, or posteriorly in the interscapular area. The upper extremities and thorax may be more developed than the low er extremities. A midsystolic murmur over the anterior part of the chest, back, and spinous processes may become continuous if the lumen is narrowed sufficiently to result in a high-velocity jet across the lesion throughout the cardiac cycle. CXR shows a figure 3 sign in the left upper mediastinum secondary to hypoplasia of the aortic arch with poststenotic dilation of the aorta infra -coarctation.

216

PEDIATRICS EXAMINATION REVIEW

Q2. Atrial septal defect is usually asymptomatic until middle age. Right ventricular lift, widely split and fixed S2, systolic ejection murmur at pulmonary area are some of the examination findings. ECG shows right axis deviation and features of right ventricular hypertrophy. Prognosis depends on the size of shunt - large defects cause disability by the age of 40. Usually, during inspiration, there is an increase in venous return to the right ventricle which delays closure of the pulmonary valve. In an ASD, the atria are linked via the defect, therefore inspiration produces no net pressure change between them the timing of the pulmonic valve closure does not change with regards to aortic valve closure and has no effect on the splitting of S2 - a fixed split S2. Q3.Ebstein anomaly (congenitally malpositioned tricuspid valve) is a cardiac defect in which the leaflets of the tricuspid valve are displaced into the right ventricle - a portion of the right ventricle is atrialized. The S1 is split. It is common in this condition for an atrial septal defect to be present and a fixed S2 is heard together with an S3 and an S4 . Hence a quintuple cadence is heard on auscultation. Q4. Patent ductus arteriosus is usually asymptomatic but can present with left ventricular failure or pulmonary hypertension. In this condition, pressure in the aorta is higher than the pressure in the pulmonary artery throughout systole and diastole thus a continuous murmur is heard. Features of left ventricular hypertrophy are found on examination and ECG. Large shunts cause high mortality from cardiac failure. Infective endocarditis is more common. S urgical ligation of the ductus gives good results. Q5. In ventricular septal defect the left ventricular pressure is higher than that of the right ventricle during systole resulting in the characteristic holosystolic murmer. 7-An 18-year-old man was admitted to casualty at 3 am having collapsed at a nightclub. A friend, who had accompanied him in the ambulance but subsequently left A and E before speaking to a doctor, told the paramedics that his friend had been well earlier in the evening and that he usually did not drink much alcohol or take any drugs. Investigations revealed: Haemoglobin White cell count Neutrophils Lymphocytes Monocytes Eosinophils Basophils Platelets Reticulocyte count Prothrombin time Fibrinogen D-Dimer screen Serum sodium Serum potassium Serum urea Serum creatinine Lactate dehydrogenase

8.6 x 109/L 25.8 x 109/L 22.5 x 109/L 2.0 x 109/L 0.8 x 109/L 0.4 x 109/L 0.1 x 109/L 32 x 109/L 12% 32 s 0.3 g/L 3.0 mg/L 140 mmol/L 6.3 mmol/L 17.5 mmol/L 200 mmol/L 540 IU/L

Activated partial thromboplastin time 90 s

The blood film showed red cell fragmentation with polychromasia, toxic granulation of neutrophils and platelet anisocytosis. What is the diagnosis? Options : A. Autoimmune haemolysis B. Autoimmune thrombocytopaenia C. Disseminated intravascular coagulation D. Drug-induced haemolysis

Choose 1

217

PEDIATRICS EXAMINATION REVIEW E. Thrombotic thrombocytopenic purpura Teaching Notes for Question 7 Theme: DIC and Ecstasy The coagulopathy, low fibrinogen, raised D-dimers, anaemia, high neutrophil count and thrombocytopenia are all indicative of disseminated intravascular coagulation (DIC). In addition the renal failure with high potassium levels (K) and lactate dehydrogenase (LDH) are suggestive of associated rhabdomyolysis. The causative agent is likely to be ecstasy/3,4 methylenedioxymethamphetamine (MDMA). 8-Options A. Intussusception B. Mesenteric adenitis C. Meckel's diverticulum D. Appendicitis E. Strangulated inguinal hernia F. Hypertrophic pyloric stenosis G. Constipation H. Nephroblastoma I. Riedel's lobe of liver Instructions: Choose the single most likely diagnosis from the options listed above for each of the clinical scenarios described below. Each option may be used once, more than once or not at all. 1. A 4 year old boy is brought into hospital by his mother. For the past 2 weeks he has had a cold, a sore throat, headache and nausea. Today he has vomited twice, is not toler ating fluids, and complains of central abdominal pain. 2. A 2 year old girl is referred urgently to clinic having presented to her general practitioner with haematuria on two occassions, and an abdominal mass. She is now asymptomatic. There is a firm mass with a poorly defined smooth lower border in the left upper quadrant of her abdomen. On percussion of the liver there is no dullness, and on palpation the liver is non -tender. 3. A 13 year old girl presents with a complaint of abdominal pain that she has experienced for the past two weeks, which has worsened acutely today. History reveals a bright re d loss of blood per rectum with defaecation, which she notes on the toilet paper. Her abdomen is soft on palpation and on auscultation there are normal bowel sounds. A per rectum examination is too tender to perform and an anal fissure is noted at 6 o'cloc k. 4. A 1 year old child has been noted to have bouts of screaming where he is doubled -up in agony from abdominal pain with a 60 minute pain -free period in between. When you as k to see his nappy, you note a redcurrant jelly stool, with a trace of blood on his anus. Teaching Notes for Question 8 Theme: Abdominal conditions in children 1. Mesenteric adenitis is caused by inflammation of the mesenteric lymph nodes. It is most often associated with a viral infection, but an association with streptococcal infections of the upper respiratory tract has been reported. Its clinical presentation mimics that of acute appendicitis. The child with appendicitis is usually quiet with facial flushing. Enlarged cervical lymph nodes on examination with a history of a previous URTI points towards mesenteric adenitis. On examination, the site of abdominal tenderness may shift with a change in patient position, as opposed to appendicitis, where the location of tenderness tends to be fixed. Pyrexia usually resolves rapidly with mesenteric adenitis. Unfortunately (for diagnostic purposes) appendicitis has few signs of peritonitis in the early stages. The longer history of this case suggests mesenteric adenitis as the single most likely diagnosis. If the child is not admitted for observation, there should be careful discussion with the parents to observe for worsening symptom s despite analgesics and fluids. This will prevent a diagnosis of appendicitis being missed. 9-A 18-years-old adolescent had a bone marrow transplant from his sister for treatment of acute myeloid leukemia. Six weeks later he presented with a rash, gener al malaise, poor appetite and chronic watery, green diarrhoea. His medications included cyclosporin, penicillin V, co-trimoxazole and aciclovir.

218

PEDIATRICS EXAMINATION REVIEW

On examination, a diffuse macular rash was visible, mainly affecting the palms of his hands and the soles of his feet. His pulse was 104 beats per minute (bpm) regular. Blood pressure 112/74 mmHg. Heart sounds were unremarkable and his chest was clear. His abdomen was soft apart from mild tenderness in the right upper quadrant. No organs were palpable. Bowel sounds were active. Neurological examination was normal. Investigations revealed: Haemoglobin Mean corpuscular volume (MCV) White cell count Neutrophils Lymphocytes Monocytes Eosinophils Basophils Platelets Serum albumin Serum total bilirubin Serum alanine transferase Serum alkaline phosphatase Serum gamma glutamyl transferase What is the diagnosis? Options: A. Acute graft versus host disease B. Clostridium difficile infection C. Cytomegalovirus infection D. Side-effect of immunosuppression E. Transfusion-associated graft versus host disease Teaching Notes for Question 9 Theme: Graft versus host disease He has all three of the main manifestations of acute graft -versus-host disease (GVHD) dermatitis (macular rash affecting palms and soles), hepatitis (tender right upper quadrant (RUQ) and raised alanine transaminae (ALT) and bilirubin) and gastro -enteritis (diarrhoea usually described as green and watery). Choose 1

12.4 g/dL 90 fL 3.65 x 109/L 1.7 x 109/L 1.0 x 109/L 0.8 x 109/L 0.05 x 109/L 0.1 x 109/L 147 x 109/L 36 g/L 64 mol/L 90 IU/L 135 IU/L 40 IU/L

10-A 14 year old girl was referred to the local child psychiatrist by her GP who was concerned about her weight loss. The GP reported that she has lost about 2 stones over the past 2 months and that although she was preoccupied by preparing food, she did not seem to be eating well. Her mother reported that she has been hording food in her bedroom. The mother also informed the GP that despite losing weight her daughter was spending long

219

PEDIATRICS EXAMINATION REVIEW

time every day undertaking heavy exercises. Which of the following supports a diagnosis of anorexia nervosa in this patient:Options: A. Loss of over 15% of body weight B. Binging on fatty food C. Increased levels of growth hormone D. Increased levels of cortisol E. Absence of depressive features F. Amenorrhea G. Use of diuretics Teaching Notes for Question 10 Theme: Anorexia Anorexia nervosa is a seriou s psychiatric disorder affecting girls and young women. It may also affect males who make less than 10% of all patients. The onset is usually in the teen years. It is very rare in middle age and beyond. Patients usually present with 15% or more body weight loss. Patients routinely avoid fattening food and indulge in excessive exercises, self -induced vomiting and use of diuretics. The main psychopathology behind the illness is distortion of body image and morbid fear of fatness. In addition to amenorrhoea, patients may also have elevated growth hormone and cortisol levels. Treatment of the condition may include hospital admission to restore weight and psychotherapy. Prognosis is variable. 11-Options A. Spinal X-ray and HLA profile B. Bone scan C. MRI D. Anti dsDNA and complement E. Thomas test F. Pelvic spring test G. Sigmoidoscopy, biopsy and enema H. Joint aspiration and microscopy I. Clinical history alone, joint X-ray J. Joint aspiration, swab and blood cultures K. Rheumatoid factor, joint X-ray L. ECHO M. Neurophysiology Instructions: For each of the patients described below choose the single most likely option from the above list of options. Each option may be used once, more than once or not at all. 1. A 5-year-old Asian refugee child develops high fevers, organomegaly, joint pains and a rash. He also has a 4/6 murmur. 2. A 45-year-old man who was on a fast developed acute pain in his toe. 3. A 45-year-old woman presents with wrist pain, which is worse at night and relieved by dangling the hand over the edge of the be d and shaking it. 4. A 45-year-old man presents with swollen, painful and stiff hands and feet and the symptoms are worse in the mornings. 5. A man has a bike accident and injures his leg. There is a small open wound. The wound is treated but 2 days later he develops pain in his knee. Choose 5

220

PEDIATRICS EXAMINATION REVIEW

Teaching Notes for Question 11 Theme: Differential diagnosis of joint pains 1. This is rheumatic fever. He should have a full cardiac assessment. 2. Joint aspiration and microscopy is needed here as this is a case of gout which is due to de position of sodium monourate crystals in the joints and may be precipitated by trauma, surgery, starvation, infection and diuretics. Diagnosis depends upon finding urate crystals in tissues and synovial fluid. 3. Neurophysiology is needed here as this is a case of carpal tunnel syndrome and neurophysiology to find the level of the lesion and assess axonal degeneration and the likelihood of improvement after surgery is essential. 4. Rheumatoid factor and joint X -ray is needed here as this is a case of rheumatoid arthritis which presents with swollen, painful and stiff hands and feet and the symptoms are worse in the mornings. 5. Joint aspiration, swab and blood culture is needed here as this is a case of infection of the joint which may have gained entry from the wound. 12-Options A. Varicella B. Scabies C. Pityriasis rosea D. Psoriasis E. Meningitis F. Measles G. Kawasakis disease H. Herpes simplex virus I. Ebstein-Barr virus J. Roseola infantum K. Impetigo L. Erythema toxicum Instructions: For each of the patients described below choose the single most likely option from the above list of options: Each option may be used once, more than once or not at all. 1. A 6-month-old boy presents with a generalised rash after a bout of fever. 2. A 3-year-old boy presents with oedema, congested conjunctivae, erythema of the oral cavity, lips, and palms. It does not respond to antibiotics. 3. A mother rings A&E saying that her 3 -year-old boy has fever for 1 day and is drowsy. He also has a rash which does not blanch on pressure. 4. A 17-year-old African male presents with sore throat, fever, anorexia, malaise, lymphadenopathy, palatal petechiae, and splenomegaly. 5. A non-immunised child of 5 years presents with catarrh, conjunctivitis, fever. He has some bluish white spots on his buccal mucosa. Teaching Notes for Question 12 Theme: Fever with rash Q1: This is a case of Roseola infantum. Infection with HHV -6 frequently develops during infancy as maternal antibody wanes. Although HHV -6A has not yet been associated with disease, HHV-6B can cause exanthe m subitum (roseola infantum), a common illness characterised by fever with subsequent rash. HHV-6B is also a major cause of febrile seizures without rash during infancy. In older age groups, HHV-6B has been associated with mononucleosis syndromes, focal en cephalitis, and (in immunocompromised hosts) pneumonitis and disseminated disease Q2: This is a case of Kawasaki disease (mucocutaneous lymph node syndrome. It is an acute, febrile, multi-system disease of children. It is characterised by unresponsiveness to antibiotics, nonsuppurative cervical adenitis, and changes in the skin and mucous membranes such as oedema, congested conjunctivae, erythema of the oral cavity, lips, and palms, and desquamation of the skin of the fingertips. Q3: This is a case of meningitis as the child has a fever and has been drowsy. He also has the characteristic non-blanching rash of meningitis. The organism can and must be isolated. Q4: This is a case of infection caused by Epstein Barr virus which preferentially infects B lymp hocytes. It presents with sore throat, fever, anorexia, malaise, lymphadenopathy, palatal petechiae, and

221

PEDIATRICS EXAMINATION REVIEW

splenomegaly. Q5: This is a case of measles caused by RNA paramyxovirus spread by droplets, incubation period is 7 21 days. It presents with catarrh, conjunctivitis, fever. Koplik spots are pathognomonic grain of salt like spots on the buccal mucosa. 13-Options : A. Deformation B. Disruption C. Malformation D. Dysplasia E. Teratogen F. Dystocia Instructions: Match the above options with the best descriptive statement from the choices given below:1. Chemical agent with a malign influence on development 2. Abnormal development of a structure caused by extrinsic forces 3. Intrinsic defects in the pattern of development 4. Destruction of normally formed tissue 5. Abnormal organisation of cells in tissue Teaching Notes for Question 13 Theme: Dysmorphogenesis Types of problems in morphogenesis include: a. defective formation of tissue - malformation. e.g. congenital heart defects b. Unusual forces acting on normal tissues - deformation e.g. breech deformat ion sequence c. Breakdown of normal tissue - disruption. e.g. amniotic bands d. Abnormal organisation of cells in tissue - dysplasia e.g. abnormal ear shape and size in many syndromes 14-Options : A. Herpes virus B. Streptococcus C. Staphylococcus D. Brucellosis E. Tuberculosis F. Poliomyelitis G. Candidiasis H. Infectious mononucleosis I. Rubella J. Diphtheria K. Coryza L. Meningitis M. Hepatitis N. Influenza

222

PEDIATRICS EXAMINATION REVIEW

Instructions: For each of the patients described below choose the single most likely option from the above list of options: (Each option may be used once, more than once or not at all.) 1. A 4-year-old boy presents to A&E with shock. He is found to have myocarditis. His mother gives a history of tonsillitis. 2. A 15-year-old patient presents with a rash and suboccipital lymphadenopathy. He was not immunised as a child. 3. A 35-year-old lady presents with myalgia, nausea, and fever. She also has abdominal pain. Her bilirubin is raised. 4. A veterinarian presents with fever, malaise, anorexia, vomiting, hepatosplenomegaly, depression, arthritis, sacro -ileitis and rash. 5. A man who is on steroids for rheumat oid arthritis for a long period of time now presents with white patches in his mouth. Teaching Notes for Question 14 Theme: Diagnosis of infections Q1: This is a case of diphtheria which is caused by corynebacterium diphtheriae. Diphtheriae usually presents with a brassy cough, tonsillitis and a false membrane over the fauces. The toxin may cause polyneuritis. Shock may occur from myocarditis, toxaemia or conducting system inv olvement. Q2: This is a case of Rubella caused by a RNA virus. The incubation period 14 -21 days. It presents with a mild macular rash and suboccipital lymphadenopathy. Small joint arthritis may also be present. Q3: This is a case of hepatitis. This is evident from the fact that the patient has jaundice (yellow sclera) and pain in the region of his right upper abdomen. Q4: This is a case of brucellosis which is common in the Middle East. Brucellosis is more commonly found amongst vets and farmers. The following signs and symptoms are present: fever, malaise, anorexia, vomiting, hepatosplenomegaly, depression, arthritis, sacro -ileitis and rash. A diagnosis is made using blood cultures. Q5: This is a case of candidiasis which is commonly found in patien ts who have low immunity.

Signature not verified!

kamel hassan

Digitally signed by: kamel hassan DN: CN = kamel hassan ORG = msd OU = 1 C = AD Date: 2007.04.30 19: 59:22 +02'00' Reason: I am the author of this document

223

You might also like